Final Exam (GI, Neuro/Psych, EENT, Pulm, Renal/GU, WH, Cardio, Heme/Endo, ID/Derm, Ortho)

अब Quizwiz के साथ अपने होमवर्क और परीक्षाओं को एस करें!

A 16 y.o. male presents with a history of injury to his left pinna while competing in a wrestling match. The superior outer portion of the pinna is edematous and fluctuant to palpation. There is minimal tenderness to palpation. What is the risk of this injury?

"Cauliflower ear"

Complications of chronic pancreatitis include :

1. Chronic intractable abdominal pain 2. Jaundice 3. Diabetes mellitus 4. Vit B 12 malabsorption

A 24 yr old female c/o a palpable, firm, mobile, 2 cm mass to her right breast at the 12 o'clock position. She reports it has been present for 6 months, enlarges with her menses and is painful at the onset of her menses. She smokes 12 cigarettes a day; drinks 4-5 cups of caffeinated coffee daily. She denies any alcohol intake. There is no FHx of breast cancer. You order a breast ultrasound and a fibroadenoma is found. What is your treatment plan?

1. Provide reassurance 2. Refer for FNA 3. Advise to decrease or limit her caffeine intake 4. Decrease dietary fat intake 5. Apply warm compresses prn 6. OTC acetaminophen or ibuprofen prn discomfort 7. Wear a bra with firm support (ie: sports bra) & at HS prn 8. Demonstrate technique for SBE Explanation: Females under 30 can begin with an ultrasound Ultrasound can evaluate dense breast tissue & distinguishes between fluid-filled cysts and solid masses Females at high risk or those with a suspicious lump should have a mammogram Refer for FNA May collapse the cyst Have pt f/u in 1 mo Fluid removed is sent for cytology Have pt f/u in 1-2 mos If no fluid, or bloody fluid, is removed - refer for breast biopsy

What is the recommended target LDL to reduce the risk of coronary artery disease in a diabetic patient?

100 mg/dL The National Cholesterol Education Program clinical practice guidelines have designated diabetes as a coronary risk equivalent and have recommended that patients with diabetes should have an LDL cholesterol goal of 100 mg/dL

Gestational DM of pregnancy is screened by a 50-g glucose load, with blood drawn 1 hour after administration of the load. What is the usual value considered the upper limit of normal for this test?

140 mg/dl The screening test for gestational DM is as follows: A. Time: 24-28 weeks B. Glucose load 50 g C. Time sample drawn: 1 hour D. Screening cut-off: 140 mg/dl

Normal hemoglobin A is made of what combination of heme and globin chains?

2-alphas and 2-betas Hemoglobin A1 is composed of two alpha and two beta chains.

A 46 year-old male is brought to the emergency department after sustaining burns to his entire right arm and leg in a house fire. Which of the following percentages best classifies the extent of his burns?

27 According to the "rule of nines" the percentages can be calculated as follows: entire arm = 9% of body surface area (BSA). Anterior surface of each leg = 9% BSA, posterior surface of each leg = 9% BSA. The patient has a total of 27% BSA with burns

For which patient below would drug therapy most likely be considered for initial treatment

50-year-old male diabetic smoker on antihypertensive meds; LDL-C: 180 mg/dL

A patient sustained a 6 cm laceration on his anterior tibia that was primarily closed in the emergency department. What is the most appropriate time frame for removal of these sutures?

7-14 days Suture removal is based upon the area of the body that was sutured. Facial sutures are placed for 3-4 days, scalp sutures for 5-7 days, trunk sutures are placed for 6-8 days, and sutures on the extremity are placed for 7-14 days. Sutures on the extremities can stay for longer periods of time if the area is under maximal tension

A 55-y/o male presents complaining of episodic substernal CP that occurs especially during strenuous exercise. Suspecting CAD, an exercise stress test is ordered. The test is considered to be abnormal if which of the following occurs?

A 2 mm ST-segment depression is seen on the ECG at peak exercise. A 2 mm ST-segment depression is suggestive of cardiac ischemia and is considered to be an abnormal finding.

Which of the following historical factors differentiates post-traumatic stress disorder from acute stress disorder? A. The inability of the person to recall an important aspect of the event. B. Avoidance of stimuli that invokes recollections of the event. C. A belief that their future has been foreshortened because of the event. D. The presence of sleep disorder.

A belief that their future has been foreshortened because of the event. Post-traumatic stress disorder and acute stress disorder have many of the same characteristics. A sense of a foreshortened future, such as not expecting a normal life span or a career due to the trauma, distinguishes post-traumatic stress disorder from an acute stress disorder. The other answers are common to both disorders.

Four weeks postpartum, a 32-year-old woman develops palpitations, heat intolerance, and nervousness. She is diagnosed with hyperthyroidism. Her thyroid is not enlarged or tender. The 24-h uptake of radioactive iodine is 1%. The most appropriate treatment for this woman is

A beta blocker This patient has postpartum thyroiditis, which occurs in 5 to 9% of all postpartum women. Appropriate treatment is symptomatic because the hyperthyroidism is caused by the release of preformed thyroid hormone from a damaged thyroid gland. Therapies, such as beta blockers, aimed at treating symptoms are the most effective treatment.

A 44 year-old male construction worker fell on his outstretched hand while at work. He complains of right wrist pain. On PE, there is swelling and reduced range of motion secondary to pain. He is point tender over the anatomic snuffbox of the right wrist. X-rays are negative. Based on the exam, you should be suspicious of

A fracture of the scaphoid bone Fracture of the scaphoid bone Fall on outstretched hand ; Most common carpal fracture Tenderness & swelling in anatomical "snuffbox" Fractures are often "occult"--not seen on initial Treatment: Thumb-spica splint, analgesics and surgery when displaced

A patient had an acute inferior, transmural myocardial infarction 4 days ago. A new murmur raises the suspicion of MR due to papillary muscle rupture. Which of the following murmur descriptions describes this condition?

A grade IV/VI systolic murmur heard best at the apex with radiation to the left axilla. This is a classic description of mitral regurgitation. The papillary muscle rupture is a complication of an acute inferior transmural myocardial infarction, and results in a failure of the mitral valve leaflets to close. The direction of regurgitant flow of blood is toward the left axilla.

In which of the following patients would PTCA most likely be used?

A patient with one vessel disease

A 53-year-old man presents to the ED after the acute onset of CP . The episode occurred 8 h prior to his arrival and lasted for a total of 20 to 30 min. The CP is now resolved. The patient has a long Hx of DM and hypercholesteremia and has smoked approximately 1 to 1 1/2 packs per day of cigarettes for past 30 years. On PE, BP of 84/52, and HR is 54. He has JVD to angle of mandible and clear lung fields. His rhythm strip reveals a Wenkebach pattern. Given his hypotension, a SG catheter is placed. RA pressure is estimated at 16 mmHg (normal 0-5), PA pressure at 20/10 mmHg (normal 12-28/3-13), and PCWP s 8 mmHg (normal range 3-10). Which of the following is most consistent with this clinical picture?

A right ventricular infarction Suspect Right Ventricular Infarct (40%): V1R,V3R-V6R

Which type of anti-hypertensive agent is NEVER recommended in pregnancy?

ACE inhibitors

A 71-y/o man complains of occasional lower back pain. His BP is 150/85 mm Hg and his HR is 80/min. Cardiac exam reveals an S4 gallop. Abdominal exam reveals a pulsatile mass approximately 5.0 cm in diameter palpable in the epigastric area. Peripheral pulses are normal. Which of the following is the most likely diagnosis?

Abdominal aortic aneurysm

A patient is concerned about being exposed to condyloma acuminata. Which of the following tests is most appropriate to perform in order to better identify these lesions?

Acetowhitening Acetowhitening is used to facilitate the diagnosis of condyloma acuminata lesions. A 3 to 5% acetic acid solution is applied to these suspected genital warts for five to ten minutes. Condyloma lesions will whiten and appear as circumscribed macular or papular lesions with a granular surface.

A 32 year-old man heard a "pop" when running across the court while playing tennis. He experienced pain and immediately fell. He is unable to plantar flex his foot. The most likely cause is rupture of the:

Achilles tendon

A 21-year-old female who is sexually active presents to your office with the recent onset of burning with urination and frequency. She reports her urine is cloudy and rust colored. She denies fever, abdominal pain, nausea or vomiting. On exam she appears well hydrated but uncomfortable requesting to excuse herself to the restroom. There is no vaginal discharge by history or exam, nor is there costovertebral angle tenderness. Mild suprapubic discomfort is elicited with abdominal palpation. What is the MOST likely diagnosis?

Acute cystitis

An ELISA assay detects the presence of antibodies to Entamoeba histolytica; cysts fro m the same organism are found in the stool specimen. A trichrome stain shown here highlight Which of the following is the most appropriate next step in management?

Administration of metronidazole (Flagyl) Treatment Colitis or liver abscess Metronidazole(750 mg PO or IV tid for 5-10 d) followed by paromomycin 500 mg PO tid for 10 days Asymptomatic cyst carriers Iodoquinol 650 mg PO tid for 20 d or paromomycin 500 mg PO tid for 10 d Except in the case of rupture, amebic liver abscesses rarely require drainage Diagnostic Studies Demonstration pf trophozoites or cysts of E. histolytica on (1) wet mount, (2) iodine-stained concentrates of stool, or (3)trichrome stains of stool or concentrates 3 fresh stool specimens should be examined A combination of these procedures is positive in 75-90% of cases

A 30-y/o male has experienced a syncopal episode lasting approximately 45 seconds. He states he is athletic and that this is the first episode of "passing out". The nurse hands you his EKG below What would be your next step?

Admit the patient to a monitor bed with a consult to a cardiologist for placement of a AICD - stat Brugada Syndrome Autosomal dominant disorder Diagnostic EKG pattern of incomplete RBBB with coved ST segment elevation in leads V1-V3 may be transient or apparent with drug challenge Arrhythmias commonly occur during sleep & risk of SCD is highest in patients with a spontaneous Brugada ECG pattern or prior syncope Patients' should undergo ICD implantation Tx AICD/Avoid V-Arrhythmia Syncope in younger male(must rule out)

A 51 year-old diabetic female is brought to the emergency department with complaints of fever and chills for the past 24 hours. Physical examination reveals a BP 100/70 mm Hg, P 110bpm, oral T 102 F, R 18 and pulse ox 98% on room air. On her right foot you note an area between the 4th and 5th digits that is erythematous, moist and tender. Lymphadenopathy is present behind the right knee. Which of the followings is the next best step in treatment?

Admit the patient, obtain blood and wound cultures and treat with IV antibiotics This is a diabetic foot. Failure to recognize and treat appropriately in a timely fashion could cause further morbidity and mortality. Early admission, cultures and aggressive IV antimicrobial treatment is the standard of care

What is the most common cause of nosocomial pneumonia?

Aerobic gram-negative bacteria & S. aureus

A 65-year-old male with a history of BPH develops HTN . Which of the following would be the best treatment option for this patient's HTN ?

Alpha blocker

A 35-year-old woman presents with fever, diarrhea, and right upper quadrant pain. She has recently returned from a 2-month business trip in Mexico. PE reveals no jaundice. She has point tenderness over the liver and has a positive FOBT. CT scan of the abdomen reveals several oval lesions in the liver. Which of the following is the most likely diagnosis in this patient?

Amebic Liver Abscess

Which classification of medication would most likely contribute to a patient's hearing loss?

Aminoglycosides

Which of the following is used to follow the progression of macular degeneration?

Amsler grid

A 60 year-old patient presents with elevated blood pressure and peripheral edema. Laboratory testing reveals a BUN of 58 mg/dl and a creatinine of 4.5 mg/dl, these results are unchanged from six months ago. Urinalysis today is negative except for the following, specific gravity of 1.002, 2+ protein, and trace glucose. Which of the following laboratory findings would be most consistent for this patient?

Anemia

A 58-year-old female has a decreased serum Fe and total iron binding capacity (TIBC) in association with an increased serum ferritin. These findings are most indicative of:

Anemia of Chronic Disease Most common type of anemia in hosp pts Can be seen in pts with infections, malignancy (mets, Hodgkins, MM), chronic inflammatory ds (SLE, RA), and diabetes, CHF Signs & Symptoms: fatigue, pallor, dizziness, SOB Labs: H&H, RBC are decreased, MCV usually normocytic but can be microcytic, retic is normal Dx: low serum Fe, TIBC and transferrin, ferritin levels are HIGH (remember: it is an acute phase reactant) Tx: Treat the primary disorder

A 56-year-old male came to the ED with complaints of palpitations and SOB since 1 week ago. He has a longstanding history of poorly controlled HTN PE reveals an elevated BP of 190/98 , elevated JVP, mild hepatomegaly, bilateral pedal edema, and rales at the lung bases. Diagnostic studies reveal concentric LVH without significant valvular abnormalities on echocardiogram. Which of the following drugs is beneficial in the treatment of the patient's condition by virtue of both afterload and preload reduction?

Angiotensin-converting enzyme inhibitor- enalapril

A 59 year-old male with longstanding uncontrolled HTN presents with nausea and a 5 pound weight gain in the last 2 days. He states "my belly is getting bigger and I can't fasten my pants." Which of the following PE findings would be most likely in this patient?

Ankle edema and elevated JVP This patient's symptoms indicate right heart failure with bloating.

A 33 year-old male presents to your office with a complaint of right knee injury associated with pain and swelling. He states he was running after his loose dog and suddenly stopped, hyperextended his knee, heard a pop and noticed immediate swelling. On physical examination, the Lachman test and anterior drawer test demonstrates joint laxity. Which of the following ligaments is most likely injured?

Anterior cruciate Anterior cruciate ligament injuries occur with sudden deceleration injuries. Patients often hear a pop and the diagnosis is aided by assessing the anterior drawer test and Lachman test.

A 59 year-old otherwise healthy female develops acute dyspnea and chest pain one week post total abdominal hysterectomy. Which of the following is the next step in the management of this patient?

Anticoagulation Anticoagulation is the treatment of choice in patients with pulmonary embolism with normal ventricular function and no absolute contraindications.

You are evaluating a 21 year-old male patient with a history of drug problems and multiple arrests. He has had many jobs in the past, but does not keep them long, due to repetitive fights at work. He has a long history of violating and disregarding the rights of others. Which of the following is the most likely personality disorder in this patient?

Antisocial Antisocial personality disorder patients often disregard the rights and feelings of others. They often engage in criminal behavior.

During a routine physical exam, a 16-year-old male complains of a painful lesion on the inside of his cheek. A round whitish-gray ulcer that has a red surrounding halo is noted on exam. The lesion is tender to the touch. The patient denies any medical, surgical, sexual, or social history. What is the most likely diagnosis?

Aphthous ulcer

A 56 year-old male with a known history of polycythemia suddenly complains of pain and paresthesia in the left leg. PE reveals the left leg to be cool to the touch and the toes to be cyanotic. The popliteal pulse is absent by palpation and Doppler. The femoral pulse is absent by palpation but weak with Doppler. The right leg and upper extremities have 2+/4+ pulses throughout. Given these findings what is the most likely diagnosis?

Arterial thrombosis Arterial thrombosis has occurred and is evidenced by the loss of the popliteal and dorsalis pedis pulse. This is a surgical emergency. Venous occlusion and thrombophlebitis do not result in loss of arterial pulse

Which of the following physical examination findings is most likely in a patient with anemia, low factor VIII and a prolonged PTT?

Arthropathy Hemarthrosis is likely in Factor VIII deficiency (Hemophilia A).

A 59-year-old male smoker complains of severe substernal squeezing chest pain of 30-minute duration. The paramedics have given him SL NTG and oxygen by nasal cannula. His BP is 110/70 mm Hg and HR is 90 bpm on arrival to the ER. The EKG is normal. Which of the following is the best next step? A. Echocardiography B. Thallium stress test C. Aspirin D. Coronary angiography E. CABG

Aspirin

A 50 year old presents after an apparent suicide attempt from drug ingestion. History is unobtainable. The patient's blood pressure is 130/70 mm Hg. He is tachycardic and displays rapid deep breathing. Laboratory results are as follows: Na+ 140 mEq/dL, K+ 4.0 mEq/dL, Cl- 105 mEq/dL, HCO3- 15 mEq/dL, Creatinine 1.0 md/dL, Glucose 120 md/dL, Serum pH 7.2 What is the Anion Gap and which of the following drugs is most likely taken?

Aspirin Anion Gap 20 (Na-Cl+HCO3)

A 6-year-old boy who had a mild respiratory tract infection for 2 days awakens in the middle of the night with shortness of breath and difficulty breathing, and his parents bring him to the emergency room. He is alert, afebrile,, O2 saturation of 89%, His respiratory rate is 36/min and his heart rate is 150/min. he has a prolonged expiratory phase when breathing. He is afebrile. Lung auscultation reveals high-pitched, squeaky, musical breath sounds in all lung fields during inspiration and expiration. Which of the following is the most likely diagnosis?

Asthma

Which of the following is the most common presentation for an elderly female patient with primary hyperparathyroidism?

Asymptomatic Patients with primary hyperparathyroidism are most commonly found to have this disease by an incidental finding of hypercalcemia on routine laboratory testing as a result of screening. Up to 0.1% of the adult population has this condition which is most commonly seen in females over age 50

A patient known to have allergic rhinitis and asthma presents with chronic pruritic inflammatory lesions of the flexor surfaces, wrists, and dorsal areas of the feet. The lesions are excoriated, lichenified, and crusted patches and plaques. The most likely diagnosis is

Atopic dermatitis One aspect of atopic triad of allergic rhinitis (hay fever), asthma, and eczema Usually an intermittent, chronic, severely pruritic eczematous dermatitis This classic and common rash is also known as "eczema." Common Distribution Most commonly on flexures, with prominent involvement of antecubital and popliteal fossae; may be widespread with generalized erythroderma in severe cases Usual Morphology Scaly erythematous patches, vesiculation, crusting, fissuring & lichenification Hyperpigmentatio, lichenification, and scaling in the antecubital fossae Most pts are chronic carriers of Staphylococcus aureus in anterior nares and on skin

A patient is traveling to Africa on a church mission trip. The area he is traveling to has a high incidence of chloroquine-resistant malaria. What drug is indicated for prophylaxis in this patient?

Atovaquone (Malarone) Malarone is used for prophylaxis for malaria in areas with chloroquine- or mefloquine-resistant malaria

Which of the following is considered the antibiotic of choice in the treatment of human bite wounds?

Augmentin Augmentin is considered to be the antibiotic of choice for human bites that may be contaminated with Eikenella corrodens, strep viridans, and staph aureus

How is secondary HTN different from essential HTN?

BP is often refractory to antihypertensives

A 46-year-old woman with a history of sinusitis presents with a severe headache. She complains of neck stiffness and photophobia. On physical examination she ahs a temperature of 103.4 F. Blood pressure is normal and a heart rate is 110/min. She has a normal fundoscopic examination and no focal Neurologic defect. She has nuchal rigidity. Brudzinski and Kernig signs are positive. Which of the following is the most likely diagnosis?

Bacterial meningitis

The patient in the previous question is placed empirically on parenteral cefotaxime, erythromycin, and TMP/SMX. Five days later, fever and nonproductive cough continue, pulmonary infiltrates have progressed, and the patient is clinically worse. What is the most appropriate intervention?

Begin empiric antituberculosis regimen, schedule bronchoscopy

Which of the following is associated with normal grief?

Begins within two months Normal grief has an onset within two months of loss, lasts less than two months, and is associated with normal self-perception.

A 30 year-old patient presents with a three year history of episodic of wheezing, dyspnea and dry to minimally productive cough. These episodes are typically worse at night and during the fall and spring seasons. The patient admits to a long history of allergies and tobacco use. Exam reveals numerous nasal polyps, xerotic skin and expiratory wheezes in bilateral lung fields. There is no cyanosis or abnormal heart sounds noted. His pulmonary function test reveals a low FEV1/FVC ratio. Which of the following class of medications would be best to treat an acute exacerbation of this disorder (asthma)?

Beta agonists

A 30 year-old patient presents with a three year history of episodic of wheezing, dyspnea and dry to minimally productive cough. These episodes are typically worse at night and during the fall and spring seasons. The patient admits to a long history of allergies and tobacco use. Exam reveals numerous nasal polyps, xerotic skin and expiratory wheezes in bilateral lung fields. There is no cyanosis or abnormal heart sounds noted. His pulmonary function test reveals a low FEV1/FVC ratio. Which of the following class of medications would be best to treat an acute exacerbation of this disorder (asthma)?

Beta agonists All of these medications may play a role in the long-term management of asthma however, only beta agonists are indicated in the management of an acute exacerbation

Which of the following treatment modalities has been proven to lower mortality and should be instituted for all patients having a MI (unless contraindications exist)?

Beta blockers Beta blockers are proven to reduce morbidity and mortality associated with acute MI and should be prescribed to all patients unless a significant contraindication exists.

Which of the following is most appropriate to delay the progression of osteoporosis?

Bisphosphonate Bisphosphonate therapy is considered the current first line oral therapy for treatment. These medications reduce vertebral, hip, and nonvertebral fractures by nearly 50%, especially in the first year of treatment

Which of the following is the most common serious consequence of untreated giant cell arteritis?

Blindness Symptoms include headache, jaw/tongue claudication, scalp tenderness, fever, musculoskeletal symptoms(PMR) Sudden blindness from involvement of optic vessels dreaded complication

A 55-year-old presents with orthopnea and PND. On PE, JVD and pulmonary rales are noted. Which of the following laboratory tests would most likely be elevated in this patient?

Brain natriuretic peptide Produced in response to ventricular dilation Counter-regulatory hormone Natriuresis Diuresis Vasodilatation Causes of elevated BNP levels Non-cardiac Elderly, women Renal failure, cirrhosis, PE, cor pulmonale Cardiac Systolic dysfunction, diastolic dysfunction, HTN, valvular heart disease Causes of falsely low BNP levels Recent decompensation (< 4 hours) Level < 50 pg/ml: Neg predictive value 96% Level > 500 pg/ml: predictive of CHF Levels of 100-500 pg/ml: maybe CHF or: Pulmonary embolus; Primary PHTN; ESRD; Cirrhosis; Hormone replacement

A 54-year old male presents with hemoptysis and cough. He c/o intermittent episodes of wheezing, diarrhea, flushing and tachycardia. A bronchoscopy reveals a purple, well-vascularized central lesion. Which of the following is the most likely diagnosis?

Bronchial carcinoid tumor

A 74 year-old female with mild hypertension is brought to the ED with right hemiparesis after a fall two days ago. The family reports that the patient has had some confusion this morning. Examination reveals left pupillary dilation and decreased strength of the upper and lower extremity. Which of the following diagnostic studies should be ordered first?

CT scan of the brain This patient presents with a history of minor trauma and progressive neurological abnormalities consistent with subdural hematoma. The diagnosis would be confirmed by CT scan, which is less expensive and more sensitive for blood than an MRI.

A 4 year-old female presents with a headache and new onset confusion. She was diagnosed with a viral illness 2 weeks ago. Examination reveals petechiae and mild scant purpura on her legs bilaterally. The CBC demonstrates a platelet count of 8,000/uL. Which of the following is the most appropriate next step in the assessment of this patient?

CT scan of the head The concern for this patient is low platelets and internal bleeding. In this scenario, the patient complaining of a headache would warrant a CT scan to evaluate for an intracranial bleed

Your patient is an 18 year-old male with a family history of multiple endocrine neoplasia I (MEN I). Which of the following screening electrolyte levels would you recommend for this patient?

Calcium MEN 1 is a familial multiglandular endocrine tumor syndrome. The initial biochemical manifestation of MEN 1 is usually hypercalcemia and can be detected as early as 14-18 years old in patients with the gene mutation, although clinical symptoms do not usually manifest until the third or fourth decade. The other electrolytes are not necessarily affected.

65 y/o man presents to the ER with severe crampy abdominal pain and inability to defecate. He is diagnosed with a mechanical obstruction of the colon. This is most commonly caused by

Carcinoma Carcinoma of the colon is the most common cause of mechanical obstruction of the colon and is followed in frequency by sigmoid diverticulitis and volvulus. These three causes account for 90% of cases of colonic obstruction.

A 65 year-old homeless male with a history of pancreatitis is seen in the emergency department for vomiting, upper abdominal pain, back pain and weakness. He is cachetic, pale and jaundiced. A 4-5 cm mass is palpable in the mid to right hypochondrium. What is the most likely diagnosis?

Carcinoma of head of pancreas Seventy-five percent of pancreatic cancers are in the head. Risk factors include age, tobacco use, obesity, chronic pancreatitis, family history and previous abdominal radiation.

A 68-year-old man with a history of hypertension, diabetes, and urinary retention awoke feeling nauseated and light-headed. . He did not respond to questions from his wife. When the emergency medical technicians arrived, his blood pressure was 60 by palpation. IV fluids and oxygen were administered. Vital signs obtained in the ER were blood pressure 60, heart rate 120 and regular, temperature 38.9°C (102°F), and respiratory rate 30. A brief physical examination revealed coarse rales approximately halfway up in the chest bilaterally and inaudible heart sounds An indwelling urinary catheter was placed with drainage of 10 to 20 mL of dark urine. ECG was unremarkable except for sinus tachycardia. Antibiotics were administered, and the patient was transferred to the ICU, where a right heart catheterization was performed. Pulmonary capillary wedge pressure was 28 mmHg. Cardiac output was 1.9 L/min. Right atrial mean pressure was 10 mmHg These findings are most consistent with which of the following types of shock?

Cardiogenic Hemodynamic Profile in Cardiogenic Shock PCWP increased PCW normal or decreased in RV infarction CO decreased SVR increased

Which of the following is not a correct description of vasovagal syncope

Caused by excessive sympathetic nervous system tone Vasovagal syncope is the most common cause of syncope in healthy young people. It often has a preceipitating event, prodromal symptoms and an excellent prognosis

A 60-year-old man is involved in a head-on motor vehicle accident and sustains significant head trauma. He is awake and oriented to person, place, and time but complains of dizziness. Physical examination reveals normal vital signs, no orthostasis, and no neurologic findings. Overnight in the surgical intensive care unit, the patient develops excessive thirst, polydipsia, and polyuria. He develops orthostatic changes on physical examination. His serum sodium rises to 160 meq/L and his serum glucose is normal. Which of the following is the most likely diagnosis?

Central DI Man had trauma to his posterior pituitary stalk from a car accident resulting in central diabetes insipidus(DI) due to lack of vasopressin Diagnosis may be made by raising the patient's serum osmolality through water restriction then observing the urine osmolality response to injected vasopressin Neprogenic DI will not respond to the stimulation by vasopressin Patients with SIAD present with hyponatremia

A 45 year-old type 2 diabetic female with history of cholelithiasis presents to the clinic with 2-3 episodes of sudden, severe epigastric pain that radiates to her shoulder. She has associated nausea and vomiting. Temperature is 101 degrees F and she is experiencing chills. Today her eyes appear yellow in color. Which of the following is the most likely diagnosis for this patient

Cholangitis Cholangitis is characterized by a history of biliary pain, fever, chills, and jaundice associated with episodes of abdominal pain.

Which of the following is considered to be the treatment of choice for the pruritus that occurs with primary biliary cirrhosis?

Cholestyramine (Questran) Cholestyramine, a bile salt sequestrant, is able to decrease the pruritus that occurs from the bile stasis and granulomas.

In which of the following populations is the pneumococcal conjugate vaccine (Pneumovax) indicated for a patient younger than 65 years of age?

Chronic alcohol abusers Pneumococcal vaccination is not indicated in pregnant women, healthcare workers or children receiving aspirin therapy. Vaccination is indicated in chronic alcohol abusers.

A 56- year-old diabetic smoker describes progressive "tiredness" and "cramping" in his left calf for 6 to 8 months. Two years ago he jogged 1 to 2 miles every day. Now walking 5 to 6 blocks or climbing stairs produces leg discomfort that is relieved with 2 to 3 minutes of rest. These symptoms are MOST consistent with

Chronic arterial insufficiency

A 55-year-old male comes to your office for assessment of a chronic cough. He complains of "coughing for the last 10 years", the cough has become more bothersome lately. The cough is productive of sputum that is usually mucoid; occasionally it becomes purulent. He has a 35-year history of smoking two packs of cigarettes a day (70 pack/year history). He quit smoking approximately 2 years ago. On physical examination his blood pressure is 160/85 mm Hg. His pulse is 96 bpm and regular. He has a body mass index of 34, and he weighs 280 pounds. He wheezes while he talks. On auscultation, adventitious breath sounds are heard in all lobes. His chest x-ray reveals significant bronchial wall thickening. There are increased markings at both lung bases. What is the most likely diagnosis in this patient?

Chronic bronchitis

Which of the following symptoms is more commonly seen in the initial presentation of diabetes mellitus type 2 over type 1?

Chronic skin infections Chronic skin infections, including candidal vaginitis in women, are a common presenting symptom in patients with diabetes mellitus type 2. Patients are often overweight or obese. Polyphagia, nocturnal enuresis and weight loss are more consistent with the presentation of diabetes mellitus type 1.

65-year-old woman, with a history of HTN , presents with a 3 week history of an ulcer on her left ankle. Patient is a non-smoker. On physical examination the ulcer is located over the left medial malleolus. The edges are rough and the surrounding skin is darkly pigmented, edematous, and atrophic. Pedal pulses are 2+ and the feet are warm. Sensory exam is normal. Which of the following is the most likely diagnosis?

Chronic venous insufficiency

Which of the following coronary arteries is typically involved in a lateral wall myocardial infarction?

Circumflex artery

Which of the following types of infection is most likely to benefit from hyperbaric oxygen therapy?

Clostridial infection Patients with clostridium myonecrosis (gas gangrene) will benefit from hyperbaric oxygen therapy, penicillin therapy, and radical surgical excision

A patient presents complaining of new onset of repeated mottled and numb fingers when exposed to previously tolerated cold weather. The patient denies color changes or itching. ANA is negative. CBC reveals a mild anemia and rouleaux. Which of the following is the most likely diagnosis?

Cold agglutinin disease Cold agglutinin disease presents with mottled and numb fingers after repeated cold exposure. CBC reveals a mild anemia and rouleaux.

A patient presents with wrist pain after falling on an outstretched hand. X-ray shows a transverse fracture of the radius with dorsal placement of distal fragment. This best describes what type of fracture.

Colles' Fracture Mechanism: Fall on an outstretched hand; Most common fracture of distal radius Signs and Symptoms: "Silver Fork" deformity seen, with dorsal angulation of distal radius

A 62 year-old male presents with a right hilar mass. Needle-biopsy of the mass reveals the presence of small-cell carcinoma and a bone scan reveals the presence of scattered hot spots throughout the skeleton. Which of the following is the most appropriate treatment?

Combination chemotherapy Combination chemotherapy is the treatment of choice for a patient with small-cell carcinoma of the lung

Otosclerosis causes which type of hearing loss?

Conductive

Which of the following is the most common cause of a transudative pleural effusion?

Congestive heart failure Congestive heart failure is the most common cause of a transudative pleural effusion. Lung cancer and pneumonia only cause an exudative effusion. Pulmonary embolism can cause either type of effusion but most commonly it is exudative.

An 80-y/o male presents with a 6-month history of SOB with exertion. He states his condition has worsened over the past two days and has also noted an 8 pound weight gain. PMH is also positive for a MI 10 years ago and a 40-pack-year smoking history. PE reveals bilateral basilar rales and 2+ pitting edema in the lower extremities. Which of the following is the most likely diagnosis?

Congestive heart failure People with congestive heart failure present with worsening shortness of breath and weight gain. Physical exam reveals pulmonary rales, positive jugular venous distention and peripheral edema. Pericarditis presents with a pericardial friction rub. Patients with pneumonia and COPD do not present with JVD or edema.

A 37-year-old female comes to your office because she was found to be "hypertensive" by a nurse in a shopping mall screening program. She tells you that when she thinks about it, she really has not felt well for a couple of months. Her major complaint has been generalized fatigue and weakness. She also has been increasingly thirsty, urinating more frequently at night. On PE, her BP is 190/110 mm Hg. Laboratory abnormalities include a mildly increased serum Na+ level(150 mEq/L), hypokalemic metabolic alkalosis, a low serum rennin level, and increased urine potassium level. What is the most likely diagnosis based on the information presented

Conn's Syndrome(Primary Hyperaldosteronism)

A 80 y/o man was at church and briefly blacked out. He has recently had several episodes of dizziness and near syncope. A 24 hour holter monitor has recorded the following. EKG shows: Sick Sinus Syndrome What is the most appropriate treatment?

Consult for a pacemaker

A 72-year-old man with a long history of tobacco use presents to the emergency room with shortness of breath. He is mildly cyanotic and hypotensive. On exam he has poor air movement bilaterally and has evidence of an elevated jugular venous pressure. A Swan-Ganz is placed revealing RA pressures of 15 mmHg (normal range 0-5), a PA pressure of 70/28 (normal range 12-28/3-13), and a PCWP of 10 (normal range 3-11). Which of the following is most consistent with this clinical picture?

Cor pulmonale

A 67 year-old patient with a 15 year history of chronic obstructive pulmonary disease (COPD) presents with worsening respiratory symptoms and abdominal pain. On examination, you observe peripheral edema, jugular venous distention, epigastric pulsations, and tender hepatomegaly. You auscultate a systolic ejection murmur. Echocardiogram reveals a dilated right ventricle with decreased ejection fraction and a normal appearing left ventricle. Which of the following is the most appropriate diagnosis for this patient?

Cor pulmonale Cor pulmonale denotes RV hypertrophy and eventual failure resulting from pulmonary disease. It is most commonly caused by COPD. The signs and symptoms presented are classic for the diagnosis.

Which of the following is the most important medication to initiate for chronic therapy in this patient (A-Fib patient)?

Coumadin

A 6 year-old male presents with hemarthrosis of the left knee. Coagulation studies reveal the following results: PT 12.5 seconds (normal range 12-14 seconds), INR 1.0, aPTT 58 seconds (normal range 18-28 seconds), platelet count 430,000/microliter (normal range 150,000-450,000/microliter), and bleeding time 4 minutes (normal range 2-12 minutes). Which of the following is the best treatment option for this patient?

Cryoprecipitate Hemophilia A presents with a prolonged aPTT and normal platelet count and function. Hemophilia A is treated with factor VIII concentrate or cryoprecipitate

Which of the following interventions is the treatment of choice for actinic keratosis?

Cryotherapy Cryotherapy is the treatment of choice for isolated superficial actinic keratosis

A 33-year-old HIV-positive patient presents with headache, confusion, low-grade fever, and sinus fullness for 1 week. There is no history of an opportunistic infection but a recent CD4 count is 55 cells per cubic millimeter. On examination there is a moderate bilateral papilledema. The patient is mildly confused, but there are no other focal neurologic findings. CT scan of his brain with contrast is negative. A lumbar puncture reveals an opening pressure of 320 mm H2O and the fluid is noted to be slightly turbid. CSF protein and glucose levels are normal, and there are 10 white blood cells (WBCs) per cubic millimeter. India. What is the most likely diagnosis?

Cryptococcal meningitis Etiology Cryptococcus Neoformans Large thick-walled budding yeast, encapsulated in host Found in pigeon and other bird dropping in soil No specific geographic location Pathology Infection usually acquired through inhalation causing asymptomatic & self-limiting pulmonary infection May disseminate & spread to CNS Risk factors for disseminated disease AIDS (CD-4 < 100) Long term, high dose glucocorticoid use Autoimmune, & transplant Lymphoreticular malignancies (Hodgkin's) Sarcoidosis

A 13 year-old female presents to the office with right knee and thigh pain and the inability to bear weight since waking yesterday morning. The mother states the child had a fever of 100.9 degrees F this morning and continues to be non- weight bearing. Examination reveals a warm, erythematous, swollen knee. Which of the following tests would be most beneficial in the diagnosis and treatment of this patient?

Culture of joint aspirate A culture of the joint fluid will confirm the diagnosis and offer information regarding infectious agent.

A 30-year-old woman is noted to have blood pressure in the 160/100 mm Hg range. She has abdominal obesity which abdominal striae. She has easy bruisability and has increased hair growth on her face and chest. Which of the following is the most likely diagnosis?

Cushing Syndrome The central obesity, abdominal striae, hirsutism, and easy bruisability are consistent with Cushing syndrome, a disease of adrenal steroid overproduction

A 26 yr old nulliparous woman presents for evaluation of irregular menstrual cycles x 1 yr. Her periods occur every 2-3 months although she has gone as long as 4 months w/o a cycle. Menstrual flow varies from moderate to heavy. Her LMP was 11 wks ago. Menarche occurred at age 13. She denies any use of alcohol, does not smoke and has never been on hormonal contraception. She is in a sexually monogamous relationship, using condoms for birth control. She admits to a 30 lb weight gain over the past yr which she attributes to her sitting at her desk for most of the day. She does not exercise. She offers no other c/o. PE: 132/80; 84; 16; 98.2F BMI - 36 General: obese, well dressed Caucasian female, in NAD Skin: velvety thickening noted to posterior neck Heart: RRR, S1 S2 noted, no M/R/G Lungs: CTA bilat, no W/R/R Lymph: no LAD Abd: obese, non-tender, no striae or scars noted, BS+ x 4; no HSM Extremities: no C/C/E. DTRs 2+ throughout, pulses 2+ to BUE & BLE GYN: normal genitalia w/o vaginal or cervical discharge, no cervical motion tenderness, no uterine or adnexal masses What are your DDx? What is the Dx? What is your plan? What is your advice? What to prescribe?

DDx: Pregnancy, Anovulatory cycles/ amenorrhea (DUB also works here), Menometrorrhagia, Polycystic ovarian syndrome (PCOS), Leiomyomata, Endocrine polyps, Inherited coagulopathy (von Willebrand's disease is most common), Asherman syndrome (scarring within the uterus), Dx: PCOS Plan: 1. Order pregnancy test 2. Transvaginal pelvic ultrasound (US) 3. Labs - TSH; serum prolactin & testosterone; FSH & LH levels; CMP; CBC 4. F/u in 1-2 wks to review US & lab results Advice: 1. Lifestyle modifications - low calorie diet, increase exercise 2. Vaniqa, electrolysis, or laser hair removal can aid in hair removal 3. Discuss insulin resistance & infertility as possible complications Prescribe: 1. Birth control pills (OCPs decrease androgen production that can cause excessive hair growth) 2. Spironolactone (Aldactone) which blocks the effects of androgen on the skin. Spironolactone can cause birth defect, so effective contraception is required while taking this medication.

What is the most likely compensation for the previous acid-base disturbance (metabolic alkalosis)?

Decreased respiration rate

A 34 year-old female status-post trans-sphenoidal resection of pituitary adenoma presents with worsening polydipsia of 10-12 liters daily and polyuria within four days of discharge. A urinalysis reveals a specific gravity of 1.004 (1.001- 1.035) and shows decreased urine osmolality but is otherwise normal. Labs reveal mild hypernatremia. What is the treatment of choice for this patient?

Desmopressin The main treatment for diabetes insipidus is Desmopressin

Which of the following is most likely to cause a false negative PPD?

Diagnosis of AIDS Any process that results in reduced immune response can cause a false negative PPD. Past immunization with BCG would typically result in a false positive while aspirin allergy and pregnancy generally have no effect in and of themselves on PPD results.

65-y/o man with a long history of untreated HTN C/O recurrent SOB on minimal exertion. Cardiovascular system exam is normal except for a prominent precordial impulse. CXR normal except for a prominent LV shadow. An exercise tolerance test with thallium scanning reveals no evidence of myocardial ischemia. Two-dimensional echocardiography reveals LVH. Radionuclide ventriculography reveals normal R & L EF's. What is the most likely explanation for the patient's symptoms?

Diastolic heart failure Not measurable at bedside Resistance to filling directly relates to ventricular diastolic pressure Resistance increases with age reflecting myocyte loss &increased interstitial collagen deposition Associated with prolonged ventricular relaxation time (time between AV closure & MV opening when ventricular pressure falls rapidly) Etiologies HOCM Marked LVH 2nd to HTN, & advanced AS Amyloid infiltration of myocardium

Which of the following is the most likely physical exam findings in a patient with COPD?

Diffuse expiratory wheezing COPD is characterized by chronic airway obstruction with most airflow resistance occurring in small airways of the lower respiratory tract producing expiratory wheezing.

Which of the following is the most effective outpatient treatment for Mycoplasma pneumonia?

Doxycycline (Doryx) Mycoplasma is an intracellular parasite which would require an antibiotic, like doxycyline, to penetrate the cell membrane and eradicate the infection. The other antibiotic selections do not achieve this action.

A 65 year-old male 5 days status-post MI is evaluated for anterior CP . The pain improves with sitting up. The patient has associated fever, leukocytosis and a pericardial friction rub. Which of the following is the most likely diagnosis?

Dressler syndrome Pericarditis may occur 2-5 days after infarction due to an inflammatory reaction to transmural myocardial necrosis. This is known as postmyocardial infarction pericarditis or Dressler syndrome.

Which of the following physical exam findings would be most likely in a patient with a pleural effusion?

Dull to percussion with decreased tactile fremitus Due to the increased density presented by the effusion the percussion note will be dulled and transmission of speech vibration will be reduced. In contrast, emphysema with its increased lung volume will cause hyperresonance and increased fremitus.

Which of the following has been associated with exposure to asbestos? A. Pulmonary fibrosis B. Bronchogenic carcinoma of the lung C. Pleural and peritoneal based mesotheliomas D. Pleural plaques E. All have been associated

E. All have been associated

Which of the following clinical findings are predictive of more serious disease in patients with CAP? A. Hypotension B. Tachycardia and impaired oxygen saturation C. Multi-lobar involvement D. Elevated blood urea nitrogen and altered level of consciousness E. All of the above

E. All of the above

Which of the following conditions produces marked proteinuria (Excretion of > 3.5 grams daily)? A. Focal segmental glomerulosclerosis B. Lupus nephritis C. Diabetic nephropathy D. Amyloidosis E. All of the above

E. All of the above

Which of the following radiographic findings is suggestive of this silicosis?

Eggshell calcification of enlarged hilar lymph nodes

A 13-year-old boy who has a VSD with a large left to right shunt is at increased risk of developing which of the following?

Eisenmenger syndrome ↑PVR resulting in bi-directional flow Redirection of blood flow through VSD, ASD, or PDA from L-to-R to R-to-L shunt Hallmarks: Cyanosis + Pulmonary HTN + Erthyrocytosis Possible complications Hyperviscosity Cerebral hemorrhage Poor blood flow to the brain CHF Heart attack Stroke Sudden death

A 45-year-old man is diagnosed with essential hypertension based on two blood pressure readings of 150/100 and 156/102 mm Hg during two separate visits. Which of the following would most likely provide prognostic information regarding this patient?

End-organ effects from his hypertension Prognosis in hypertension depends on the patient's other cardiovascular risks and observed end organ effects such as LVH. And renal insufficiency.

A 76 y/o female with known chronic AR has a 3-week history of arthralgia, weakness, and low-grade fever. BP is 135/65, pulse is 90, and resp. is 22. Axillary temp is 100.2 degrees. Exam showed an alert, oriented patient with conjunctival petechiae and a 2/6 LSB diastolic murmur. Hgb is low at 9.4 g/dL and WBC count is high at 18.2 K/UL. Which of the following is the most likely diagnosis?

Endocarditis A 76 y/o female with known chronic AR has a 3-week history of arthralgia, weakness, and low-grade fever. BP is 135/65, pulse is 90, and resp. is 22. Axillary temp is 100.2 degrees. Exam showed an alert, oriented patient with conjunctival petechiae and a 2/6 LSB diastolic murmur. Hgb is low at 9.4 g/dL and WBC count is high at 18.2 K/UL. Which of the following is the most likely diagnosis? A. Endocarditis B. Multiple myeloma C. Ankylosing spondylitis D. Waldenstrom's macroglobulinemia E. Idiopathic thrombocytic purpura

A 25 year-old patient presents complaining of a sore throat and swollen glands in the back of her neck. Upon exam, erythema and white purulent exudates are noted in the tonsillar region. Results of streptococcal antigen tests were negative and the patient's monospot test is pending. CBC with differential normal blood values with the exception of 60% lymphocytes on the differential, 30% of which are "atypical." Which is the most likely cause of the patient's condition?

Epstein-Barr virus Epidemiology Transmitted primarily in saliva & not highly contagious Infections in infants and young children generally asymptomatic or presents as mild pharyngitis with or without tonsillitis Primary EBV infection among adolescents and young adults accounts for most cases of infectious mono. Clinical Manifestations Incubation period 4-6 weeks. Prodromal symptoms of malaise, fatigue, and myalgia frequently precede the onset of pharyngitis, fever and lymphadenopathy by a few days Splenomegaly, hepatomegaly, rash, periorbital edema, palatal exanthem & jaundice less common

A 42 year-old male with chronic complaints of heartburn and regurgitation presents for follow-up after undergoing endoscopic evaluation which shows evidence of Barrett's esophagus. Which of the following is the most serious potential complication in this patient?

Esophageal adenocarcinoma Esophageal carcinoma is the most serious complication of Barrett's esophagus.

A 62 year-old male is brought to the emergency department with acute hematemesis. The patient denies a previous history of vomiting. His wife states he has chronic liver disease. Physical examination reveals a distended abdomen without rebound, guarding or organomegaly. There is a fluid wave. Which of the following is the most likely diagnosis?

Esophageal varices Esophageal varices are dilated submucosal veins that develop in a patient with underlying portal hypertension. The most common cause of portal hypertension is cirrhosis.

An 18 month-old female presents to the Emergency Department having possibly swallowed a hearing aid battery within the past hour. She is drooling and appears anxious but parents have noticed no stridor or dyspnea. She has no history of previous esophageal injury. Physical examination is unremarkable. Chest radiograph reveals a radiopaque round object at the distal esophagus. Which of the following is the most appropriate treatment option?

Esophagoscopy for removal Esophagoscopy is the procedure of choice for acutely ingested foreign bodies.

A 48 year-old male presents with a mild tremor that seems to increase with stressful situations and subsides when he has a glass of wine with dinner. The patient does not demonstrate the tremor at rest but it reappears when he reaches for a pen. Which of the following is most likely the cause of this patient's symptoms?

Essential tremor Essential tremor can be familial and cause action related hand tremor, head tremor, or voice tremor. The lower extremities are spared and generally no further neurologic findings are present.

A 16-year-old boy was at his team basketball practice when a team member accidentally jabbed his fingers into the 16-year-old's left eye while trying to block his shot. He felt sharp, blinding pain and his eye is tearing. Which examination should be done in the side lines first following direct eye trauma?

Evaluate visual acuity with corrected external lenses in place

A 13 year-old girl reports two weeks of worsening right knee pain with no history of antecendent injury or recent trauma. She reports frequent episodes of nighttime awakening with knee pain in the past two weeks. Examination of the knee reveals edema and a tender mass over the anterior proximal right tibia. Her knee exam is otherwise within normal limits. Radiographs of the right knee show a lytic mass with a multi-laminated periosteal reaction involving the proximal anterior tibia. Biopsy of the mass is seen here What is the most likely diagnosis?

Ewing sarcoma The distinctive feature of Ewing sarcoma is the radiographic appearance of a periosteal "onion skin" reaction. A 'small round blue cell' tumor of bone is Ewing's sarcoma, commonly seen at this age.

A 36 year-old female comes to the office because a mole on her left calf has changed. On physical examination of the left posterior lower leg, there is a 12 mm, asymmetrical, variegated blue-black macule with raised pink plaque in the upper half of the lesion. Which of the following is the most appropriate clinical management of this lesion?

Excisional surgery Surgical excision of suspected melanoma is necessary for histologic diagnosis and treatment of the lesion

A 52- year-old woman with long-standing rheumatoid arthritis is hospitalized for total knee replacement. On her admission chest x-ray, a 4-cm nodule is noted near the right hilum. She has smoked one pack of cigarettes daily for the last 32 years. What would be the MOST appropriate management of this patient?

Exploratory thoracotomy

A 72-year-old female has become increasingly jaundiced. Her total serum bilirubin is 10.2 mg/dl with direct bilirubin of 8.5 mg/dl. Her serum alkaline phosphatase is elevated to 224 IU/L. The AST and ALT are normal. These findings suggest that she has:

Extrahepatic biliary obstruction

What is the most common hypercoagulable state?

Factor V Leiden Factor V Leiden is the most common inherited hypercoagulable states with about 2-5% of the population affected.

A 50 year-old female presents with constipation following an episode of tearing pain associated with bleeding while defecating 4 days ago. She admits to a residual throbbing pain and she is afraid to have a bowel movement. On examination you note what appears to be a crack in the epithelium of the anal verge. Which of the following is the most appropriate initial management for this patient?

Fiber supplementation and sitz baths This patient has signs and symptoms consistent with an anal fissure. Most patients will respond to fiber supplementation and sitz baths.

A 46-year-old woman arrives in your clinic for routine examination. She has no specific complaints, and a full review of systems is unrevealing. On PE she has normal vital signs and a 1.5-cm thyroid nodule is palpated in the right lobe of her thyroid; there are no other abnormal findings. A laboratory test reveals a serum TSH level of 2.3 mU/L. Which of the following would be the most appropriate recommendation?

Fine-needle aspiration biopsy A clinically solitary thyroid nodule is a discrete swelling within an otherwise palpably normal thyroid gland. With time, many solitary thyroid nodules may enlarge, shrink, or eventually disappear spontaneously. However, most do not change appreciably. The majority of these nodules are benign colloid nodules. Other causes of benign solitary nodules are thyroid adenomas, cysts, and thyroiditis. Approximately 5% of all solitary thyroid nodules are thyroid carcinomas. Fine-needle aspiration biopsy is indicated in all patients with solitary thyroid nodules and is especially helpful in patients with no other obvious signs of carcinoma. When this technique is used, approximately 70% of nodules are found to be cytologicaly benign. The false-negative rate is <5%. Autonomously functioning nodules are rarely malignant. A negative iodine 123 thyroid scintigraph should be followed up with surgery

An otherwise healthy 23 year-old female presents to the student health office complaining of 3 days of frequent, watery, non-bloody stools. She denies significant abdominal pain, vomiting, fever or dark urine. Others on campus have been seen with the same presentation this week. Initial choice of treatment includes which of the following?

Fluid intake and bowel rest Most mild diarrhea will not lead to dehydration with adequate fluids and comfort with rest to the bowel.

Which of the following therapies is recommended for a 13 month-old child with sickle cell disease?

Folic acid and penicillin V Patients with sickle cell disease should receive prophylactic penicillin V starting at 2 months of age and folicacid starting at 1 year of age. Ferrous sulfate is not globally recommended for patients with sickle cell disease.

Which of the following signs and symptoms is typically noted in patients with acute cystitis?

Frequency and dysuria

A 32 year-old male with history of tobacco abuse presents with an intermittent burning sensation in his chest for six months, worsening over the past 2 weeks. His wife has noticed episodes of coughing at night. He denies dysphagia, weight loss, hematemesis, or melena. His vital signs are all normal and physical examination is unremarkable. Which of the following is the most likely diagnosis?

Gastroesophageal reflux disease Gastroesophageal reflux disease presents with at least weekly episodes of heartburn and typically occurs after meals and upon reclining. Patients may complain of regurgitation, chronic cough, laryngitis, or sore throat.

In a patient with hypercholesterolemia, which of the following would not be an appropriate treatment to lower the serum cholesterol concentration?

Gemifrozil

Which of the following medications is most likely to cause acute tubular necrosis?

Gentamicin

An 18 year-old male high school basketball player comes to clinic for a routine physical exam. His height is 193 cm (76 in.); arm span is 201 cm (79 in.). He has long fingers and toes. BP is 146/62mmHg and HR 64/min. Which of the following exam findings is most consistent with the diagnosis?

Grade 2/6 high-frequency diastolic murmur at the third right intercostal space This murmur is most consistent with aortic regurgitation which can be present in patients with Marfans syndrome and a dilated aortic root.

Which of the following would be consistent for a person who has a successful response to the hepatitis B immunization series?

HBsAg negative; anti-HBc negative; anti-HBs positive

Which of the following clinical pictures best defines acquired immune deficiency syndrome (AIDS)?

HIV +, community acquired pneumonia (CAP) Mycobacterium avium complex with or without evidence of HIV infection is considered a definitive AIDS diagnosis

Anti-thyroglobulin and anti-microsomal autoantibodies are detected at high titer in a 43-year-old female. These findings are most often features of which of the following diseases:

Hashimoto's thyroiditis

A 26 year-old male presents with increased dyspnea with exercise. He has noted a decrease in his exercise tolerance over the past several months. He denies chest pain or skipped heart beats. Echocardiogram reveals LVH with asymmetric septal hypertrophy. EF is 65%. Which of the following is the most likely presenting history or physical exam finding?

He has an older brother with the same diagnosis Hypertrophic cardiomyopathy can be genetic and present in 25% of first degree relatives.

A 56-year-old man is admitted to the hospital for chest pain of 2-hour duration. His heart rate is 42 bpm, with sinus bradycardia on ECG, as well as ST-segment elevation in leads II, III, aVF. Which of the following is the most likely diagnosis?

He likely has suffered an IWMI Sinus bradycardia is often seen with IWMI, because the RCA supplies the inferior wall of the LV and the sinoatrial node. The ischemic changes in leads II, III, and aVF are in the inferior leads.

A 40 year-old male presents with several months of abdominal pain and nausea. Endoscopy reveals an irregular 1 x 2 cm area of loss of rugal folds near the antrum. Biopsies show extensive mucosal and submucosal infiltration by B lymphocytes. The patient is given appropriate treatment and the lesion resolves. Which infectious agent is most likely to be associated with these findings?

Helicobacter pylori H. Pylori causes gastric mucosal inflammation with PMN's and lymphocytes. Infection causes nausea and abdominal pain. Inflammation may be confined to the superficial gastric epithelium or may extend deeper resulting in varying degrees of gland atrophy. Eradication of H. Pylori with appropriate therapy leads to resolution of the chronic gastritis.

Which of the following is most frequently associated with renal cell carcinoma?

Hematuria

Clinical findings that would help differentiate nephritic syndrome from nephrotic syndrome include:

Hematuria & red blood cell casts

A 25-year-old schoolteacher comes to a walk-in-clinic with nausea, vomiting, anorexia, aversion to her usual two-pack-a-day tobacco habit, and right upper quadrant pain. She has been sick for the past 3 days. Two of her students in her class have come down with similar symptoms. She has had no exposure to blood products and has no other significant risk factors for sexually transmitted disease. On examination, her sclerae are icteric and her liver edge is tender. She looks acutely ill. What is the most likely diagnosis in this patient?

Hepatitis A Fecal-oral route, contaminated food or drink sources, daycare outbreaks No long term effects - self-limiting course

A 65 year-old man with long-standing, stable biopsy-proven postnecrotic cirrhosis develops right upper quadrant abdominal pain and abdominal swelling. He is afebrile. Palmar erythema, spider telangiectasias, and mild jaundice are noted on physical examination. His abdomen is distended, shifting dullness is present, a tender firm liver edge is felt 3 fingerbreadths below the right coastal margin, and a spleen tip is palpable. A faint bruit is heard over the liver. Paracentesis reveals blood-tinged fluid. Lab reveals a serum albumin: 2,6g/dL; Serum calcium: 11.2 mg/dl; Platelet count 104,000/mm3; Hematocrit: 34%; WBC count: 4300/mm3 His most likely diagnosis now is:

Hepatocellular Carcinoma

Which of the following conditions is caused by a disorder in the red blood cell membrane?

Hereditary spherocytosis The cell membrane defect of spherocytosis leads to hemolysis due to trapping of the cells by the spleen

A 3 year-old child playing in an abandoned shed is bitten by a black widow spider. The mother rushes the child to the emergency department within 20 minutes of the incident. Which of the following if the best initial intervention?

Hospital admission for symptomatic care Hospital admission for symptomatic care should be considered in children, pregnant women, and patients with preexisting cardiovascular disease.

A patient presents with recurrence of severe depressive symptoms. Previous episodes have occurred on at least five occasions, with one episode responding to ECT. The current episode has lasted for over six months. After a two-month trial of fluoxetine (Prozac), followed by six weeks of amitriptyline (Elavil), the patient still has significant symptoms, including suicidal ideation. The patient is not eating, not sleeping, and is so nonfunctional that she has been fired for missing work. She has not kept appointments for psychotherapy. Which of the following is the most appropriate intervention?

Hospitalize for electroconvulsive therapy Indications for ECT include failure to respond to several antidepressant medication trials, previous response to ECT, severe symptoms with need for rapid therapeutic response.

Which of the following is the most important factor in determining this patient's risk of sun reactivity and therefore higher risk of developing skin cancer?

How easily they tan Sun reactivity is greatest in individuals who are classified with a "white" skin color and who have a limited ability to tan after exposure to ultraviolet radiation

A 52 year-old male presents with a severely painful, swollen right great toe. He denies recent trauma, but reports several similar episodes of toe pain and swelling over the past two to three years. He has a history of alcohol abuse and hypertension, for which he "takes medication" of unknown type. Examination reveals bright erythema and edema associated with the right first MCP joint. Which of the patient's antihypertensive medications may be contributing to this condition?

Hydrochlorothiazide (HCTZ) Thiazide diuretics, such as hydrochlorothiazide, are associated with increased risk of hyperuricemia and gout.

A 25 year-old woman complains of bloating, abdominal cramps, and loose stools with flatulence. Her symptoms seem to occur 1-2 hours after meals containing any dairy products. Which of the following would be the most appropriate diagnostic test to confirm your suspicion?

Hydrogen breath test This woman most likely has lactase deficiency as her symptoms occur after dairy ingestion. Hydrogen breath test after administration of lactose will confirm the diagnosis.

The most likely lung finding on physical examination of the traumatized side is which of the following (tension pneumothorax)?

Hyperresonant percussion

Which of the following conditions is most closely associated with DM.

Hypertriglyceridemia Hypertriglyceridemia is most closely associated with DM. TG levels of 2-3 x normal are often the first sign of undiagnosed DM. All individuals with a significant elevated serum TG level should be screened for DM

Which of the following conditions is most closely associated with DM.

Hypertriglyceridemia Hypertriglyceridemia is most closely associated with DM. TG levels of 2-3 x normal are often the first sign of undiagnosed DM. All individuals with a significant elevated serum TG level should be screened for DM

An adult presents with a three month history of progressive severe muscle cramps, extremity paresthesias and lethargy which began shortly after a thyroidectomy for a malignant thyroid lesion. Which of the following is the most likely diagnosis?

Hypoparathyroidism Hypocalcemia secondary to hypoparathyroidism is commonly seen as a complication of thyroidectomy

A 56 year-old male with a history of hypertension and alcohol use presents with mid-epigastric pain and coffeeground emesis since early this morning. The patient denies diarrhea, constipation or blood in his stools. Endoscopic evaluation done after admission revealed peptic ulcer disease with active bleeding. Which of the following is the most appropriate therapy to reduce the risk of rebleeding in this patient?

IV proton pump inhibitors reduce the risk of rebleeding after endoscopic treatment in patients with peptic ulcers that have high risk features such as active bleeding.

A 2 year-old male presents with a four day history of fever and general malaise. On examination the vitals reveal an oral temperature of 102 degrees F. The child appears to have rubor on the trunk which started one day prior to this visit. Physical examination reveals a maculopapular rash with defervescence. Which of the following is the most appropriate management at this time?

Ibuprofen (Motrin) Motrin is indicated for management of the fever in Roseola infantum caused by the herpesvirus

A 9 year-old boy who has had cold-like symptoms for the past few days is brought to the clinic by his mother who states that her son had gross hematuria this morning. Prior to the cold-like symptoms the boy has been in excellent health. He is up-to-date on all of his immunizations. The patient does not have any edema, hypertension or purpura. Urinalysis reveals the urine to be cola-colored with a 2+ positive protein and 2+ hemoglobin. Microscopic analysis reveals 50-100 RBCs/HPF, no WBCs, bacteria, casts or crystals. What is the most likely diagnosis?

IgA nephropathy

Which immunoglobulin is the first to respond during the primary immune response for a gram positive bacterial infection?

IgM IgM is the first immunoglobin to respond during the acute exposure. This immunoglobulin promotes opsonization and phagocytosis. IgG is the immunoglobulin that responds during the secondary exposure. IgE is the immunoglobulin that responds during an allergic response as well as during a parasitic infection. IgA is an antibody found in colostrums and GI secretions.

A 50 y/o man is admitted to the hospital CCU after sustaining a IWMI. Eight hours after this event his BP is 70/50 HR is 110/min. His 12 lead EKG is seen above. Which of the following is the best intervention now?

Immediate synchronized cardioversion 100J (biphasic)

The pathologic process responsible for the renal damage in post-streptococcal glomerulonephritis is which of the following?

Immunologic The antigen-antibody complex that occurs as a result of streptococcal infection is the result of an abnormal immunologic response.

During a history taking, a mother describes the honey-colored crusts on her son's face She states that these crusts began as flat, red spots. Which condition is consistent with the above findings?

Impetigo Impetigo Thick, "honey" crusted lesions Caused by Streptococcus pyoderma or Staph Laboratory: Culture

A 46 year old house painter presents to your clinic with pain, swelling and limited range of motion of the shoulder. The pain began abruptly with swelling and focal tenderness. Deep palpation increases the pain in the area near the distal end of the acromium. Aspiration of the joint reveals a very small amount of fluid with low to normal WBC count and a clear to translucent color. There is no history of fever, chills or night sweats. You suspect which of the following?

Impingement Syndrome Etiologic factor repetitive overhead activity causing trauma and inflammation to supraspinatus tendon (most common), subacromial bursa, long head of biceps tendon & greater tuberosity Hx: Excessive overhead motion of the shoulder as with painters, swimmers and in the throwing arm of the baseball player, quarterback and javelin thrower

A patient has been in the CCU with an acute AWMI. He develops the abnormal rhythm shown below that does not resolve after revascularization. EKG Strip shows Which of the following is the best option?

Implant a temporary pacemaker Treatment: Stable patients require observation with ECG monitoring until transvenous pacing is performed. Unstable patients require emergent pacing, either transcutaneous or transvenous. Atropine usually not helpful in wide complex block because of infranodal location of lesion.

A 68 year-old patient presents after a syncopal episode. PMH of CAD and ischemic cardiomyopathy. Echocardiogram shows an EF of 20%. Electrophysiology study reveals inducible sustained VT from the LV . Which of the following is the best intervention?

Implantable cardiac defibrillator(ICD)

A 26 year-old female comes to the office for evaluation of a painful lump on her right buttock for the past week. Initially, it was a firm, tender nodule that has increased in size and tenderness in the past two days. On physical examination of the right buttock, there is a 3-cm fluctuant tender red nodule. Which of the following is the most appropriate initial intervention?

Incision and drainage Incision and drainage is the mainstay of therapy for abscesses

A 45 year-old female patient with history of type 2 diabetes mellitus presents with complaints of a red painful lump in her right groin. She relates that she first noticed a fairly deep, firm red lump that was tender approximately 4 days ago, which suddenly became much larger and more painful. A large fluctuant erythematous mass in the right groin area is noted on examination. Which of the following is the most appropriate initial intervention?

Incision and drainage Once an abscess forms, an incision and drainage is the initial treatment of choice.

A 55 year-old male with Class D NYHA stage IV HF with a severely depressed ejection fraction is brought to the emergency room via ambulance and is difficult to arouse, diaphoretic, and has had no urine output for the last 24 hours. He is hypotensive (BP 80/palp) and tachycardic (125/min). Physical exam reveals elevated JVP to the angle of the mandible, s1, s2 +s3 heart sound, pulmonary rales throughout auscultation of the lungs, and 4+ b/l lower extremity edema. Which of the following would best describe the hemodynamic profile in this patient?

Increased pulmonary capillary wedge pressure

Which of the following is the primary pathophysiologic abnormality in Eisenmenger syndrome?

Increased pulmonary vascular resistance Eisenmenger's Syndrome ↑PVR resulting in bi-directional flow Redirection of blood flow through VSD, ASD, or PDA from L-to-R to R-to-L shunt

How is the definitive diagnosis of a retinal detachment made?

Indirect fundoscopic exam with dilated pupil ocular coherence tomography

26 year-old female arrives in the emergency department with friends who say she was standing in front of her church, dressed in a white bathrobe, claiming to be the Virgin Mary and handing out $100 bills to all passers-by. Her friends noted that she had been depressed lately, but now seems completely euphoric. She had a similar episode two years ago. Which of the following is the most appropriate treatment?

Inpatient olanzapine (Zyprexa) therapy Psychotic Disorder

A 21 year-old female graduate student is brought to the ED by her friend who found the patient sitting in her car in the garage with the motor running. The friend reports that since she broke up wit her fiance 2 months ago the patient has become increasingly withdrawn, has had difficulty concentrating, and has lost 10 pounds. What is your recommendation for treatment of this patient?

Inpatient treatment with medication and psychotherapy Admission is indicated after suicide attempt or after aborted suicide attempt if patient still at increased risk for suicide.

A 23 year-old male being treated for an acute bacterial prostatitis has been taking antibiotics for less than 24 hours. He presents to the emergency room today with acute urinary retention for 12 hours. Which of the following is the most appropriate next step?

Insert a percutaneous suprapubic tube.

Which of the following is the preferred initial therapy in the previous patient (tension pneumothorax)?

Insert needle in right 2nd intercostal space in midclavicular line

Which of the following complications of Crohn's disease most frequently requires surgery?

Intra-abdominal abscess Surgery is the treatment of choice for an abscess, which is a common complication of Crohn's disease.

A 2-year-old baby girl is brought to the ED with a history of abdominal pain and diarrhea. Mother states that the child was playing normally and then "doubled over" with what appears to be abdominal pain. Your physical exam reveals a slightly distended moderately tender abdomen but no palpable masses or rigidity. You notice a jelly-like stool in the diaper. Which of the following is the most likely diagnosis?

Intussusception

A 35 year-old female presents with fatigue and dyspnea on exertion. She has a history of increasingly heavy menstrual periods for the last 8 months. She denies change in stool. Examination reveals HR 102 bpm, TEMP 97.6 F, RESP 20, BP 100/60 mmHg, pallor and cheilosis. Peripheral smear demonstrates a hypochromic microcytic anemia and few target cells. Which of the following is the most likely diagnosis?

Iron deficiency anemia Iron deficiency anemia is characterized by pallor and cheilosis and is commonly associated with menorrhagia. The peripheral smear will demonstrate hypochromic microcytic anemia and few target cells.

A 22 year-old graduate student presents with a six-month history of abdominal pain relieved with defecation which seems to coincide with her starting PA school. She describes alternating constipation and diarrhea as well as bloating. She denies any recent weight changes. Her only medication is citalopram (Celexa) What is the most likely diagnosis for this patient?

Irritable bowel syndrome The patient's symptoms are consistent with irritable bowel syndrome. More than 50% of patients who seek medical attention for symptoms of IBS also have a diagnosis of depression, anxiety, or somatization.

Most epistaxis originates at which site?

Kiesselbach's plexus

An intoxicated patient is brought to the ED with a two day history of acute onset of severe fever, chills and chest pain with frequent cough and dyspnea. His past history is significant for a 30 pack year history of tobacco use, alcoholism for the past 20 years and hypertension. A chest radiograph reveals a left lower lobe infiltrate. Which of the following is the most likely causative organism?

Klebsiella Klebsiella is the most likely organism based on the rapidity of onset, the patient's history of alcoholism and the chest radiograph findings. Chlamydia and Mycoplasma typically have a more subacute onset. Legionella's presentation can be variable but typically results in patchy infiltrates on radiograph.

A 34-year-old woman contemplating pregnancy is diagnosed with stage 1 hypertension, and after an evaluation is noted to have no complications. Which of the following antihypertensive classes may be appropriate for this individual?

Labetolol Labetolol is widely used in pregnant women, and is considered safe for the fetus. ACE inhibitors or ARB's and direct renin inhibitors are contraindicated in all stages of pregnancy. HCTZ can cause thrombocytopenia in the fetus and is not recommended in pregnancy. Methyldopa is an older agent and not as effective as labetolol

A 17 year-old female presents to the ED after injuring her knee in a soccer match earlier today. The injury occurred while suddenly decelerating and cutting to run in another direction. She heard a pop in the knee and complains the knee now gives out with any twisting motion. Which of the following tests is the most useful in eliciting the most likely diagnosis?

Lachman Test Noncontact anterior cruciate ligament (ACL) injuries occur with deceleration and rotational injury while running, cutting or jumping. Contact ACL injuries involve hyperextension and/or valgus forces from a direct blow to the knee. The Lachman test is considered the most valuable test to assess an ACL injury.

A 60 year-old male with mental status changes presents with his wife reporting his cognitive decline that has worsened over the last several months. Examination reveals that he is alert and oriented x 2, has a shuffling gait, a reduced arm swing and masked facies. He has no delusions but is reacting to visual hallucinations. His affect is flat, thought process reveals blocking. He has deficits in short- and long-term memory and confabulates when answering questions. A previous medical provider gave the patient an antipsychotic medication which caused catatonia. Which of the following is the most likely etiology of his symptoms?

Lewy body dementia (LBD) is a dementia similar to Alzheimer's and is characterized by hallucinations, parkinsonian features, and extrapyramidal signs. Patients with LBD lose cholinergic neurons which cause a loss in cognitive functioning and a loss in dopaminergic neurons causes the loss in motor control. Patients with LBD are very sensitive to neuroleptic and antiemetic medications that effect dopaminergic and cholingergic systems. They respond with catatonia, loss of cognitive function and/or develop life-threatening muscle rigidity.

A patient presents with these flat-topped, violaceous, polygonal, sharply defined papules on the flexor aspect of the wrist. Which of the following is the most likely diagnosis?

Lichen planus Clinical features: Variable onset /Disorder of unknown cause Can follow administration of certain drugs and in chronic graft-versus-host disease theories include stress, genetics, infective (viral hepatitis C), and immunologic (autoimmune) factors appears to be a reaction to more than one provoking factor certain medications can produce lichenoid reactions: Antihypertensive beta-blockers. Diuretics, NSAIDs/DMARDs & metformin Usual Morphology Lesions are pruritic, polygonal, flat-topped and violaceous papules and plaques Common Distribution Wrists, ankles, mouth (may be widespread) can occur anywhere on the skin, but often found on the inside of the wrists and ankles, the lower legs, back, and neck; thick patches may occur, especially on the shins Mucosal form mouth, GI tract, genital region Treatment Topical glucocorticoids Variable course, but most patients have spontaneous remissions 6-24 months after onset of disease

Immediate swelling of the knee after injury suggests hemarthrosis, which is usually due to fracture or

Ligamentous tear

In order to prevent the progression of diabetic nephropathy which of the following medications should be instituted?

Lisinopril

In amebiasis, what is the primary site of extraintestinal disease?

Liver

An 8-year-old boy faints while running. He spontaneously awakens. Vital signs are normal. The patient is alert and pink. Cardiac and neurologic exams are normal. Which of the following is the most likely diagnosis?

Long QT interval syndrome Genetic, inherited Structurally normal heart Normal cardiac exam when patient is asymptomatic Clinical Presentation When Sympathetic NS Is Activated Syncope Sudden Cardiac Death / Crib Death Due to Torsades de Pointes (vent. tachy) Acquired QT Interval May occur at any age Occurs with low serum K, Mg, Ca Associated with intake of many medicines There is increasing evidence that acquired long QT syndrome is related to a genetic defect with low penetrance Family history of youthful SCD must dictate caution in medicines taken by other family members

In which of the following conditions would a low serum thyroid-stimulating hormone (TSH) level be expected?

Long-term levothyroxine suppression therapy

Which of the following antihypertensive drugs is incorrectly matched with the indication for therapy?

Loop diuretic—gout

What is the appropriate initial intravenous drug therapy for a patient in status epilepticus?

Lorazepam (Ativan) A benzodiazepine (lorazepam) is first line in the treatment of status epilepticus, followed by phenytoin or fosphenytoin.

A 66-year-old male presents to the clinic with a complaint of not being able to hear the beeping of his microwave oven when it finishes heating. Hearing loss in this patient is typically associated with: Fibrosis of the tympanic membrane Hypersecretion of cerumen in the external auditory meatus Ankylosis of the stapes at the oval window Loss of cochlear hair cells Loss of otoconia in the otolithic membrane

Loss of cochlear hair cells

A 6 year-old boy is brought to the pediatric clinic by his mother for an evaluation of his asthma. He coughs about 3 days out of the week with at least 2-3 nights of coughing. Which of the following would be the most appropriate treatment for this patient?

Low dose inhaled corticosteroid Low dose inhaled corticosteroids are the preferred treatment for mild persistent asthma.

Which of the following best characterize the restrictive ventilatory defects seen in interstitial lung diseases?

Low lung volumes Restrictive impairment Reduced lung volumes Decreased TLC Reduced RV Reduced VC Normal or increased FEV1/FVC FEV1 a measure of expiratory capability may be low simply because VC is low

A 60 YO man came to the outpatient clinic complaining of bad mouth odor that has been occurring for months. The patient has a 40 pack year history of smoking. He has chronic bronchitis with colorless sputum. He has been visiting his dentist who assured the absence of caries and sent him to be examined for the lesion shown. The patch can't be rubbed or removed by cotton tipped applicator. What is the most likely diagnosis?

Lukoplakia

A 16 year-old female presents to the office with a rash. She was healthy until one week ago when she developed a fever, headache, generalized lymphadenopathy and a rash on her trunk. On physical examination you note an erythematous rash with central clearing. Her left knee is swollen with painful range of motion. She recently returned from Girl Scout Camp. Which of the following is the most likely diagnosis?

Lyme disease This presentation is classic for stage 2 (disseminated infection) Lyme disease. Commonly patients have skin involvement, headache, mild stiffness of the neck, fever, chills, migratory musculoskeletal pain, arthralgias, malaise, and fatigue. Lyme disease is the most common vector borne disease in the US and Europe.

Which of the following is considered to be the modality of choice for the identification of a pituitary macroadenoma that is suspected on the basis of a visual field deficit?

MRI of the brain MRI of the brain provides the best visualization of pituitary tumors.

You are working in the cardiac ICU and are called to deal with a cardiac arrest. The patient is a 36-y/o female who was admitted with dehydration in the setting of a GI illness. She has had profound vomiting and diarrhea for the last 4 days with 10 recorded stools. In addition, the patient is known to be an alcoholic who drinks a pint of vodka daily. On arrival, the patient is in full cardiac arrest without a pulse, and the nursing staff has initiated basic cardiopulmonary life support. The patient has been intubated. The patient's rhythm is shown below. EKG Shows Torsade de Pointes Which of the following drugs would be most helpful in correcting this rhythm ?

Magnesium sulfate

What test is the single most useful test in establishing the diagnosis of multiple sclerosis?

Magnetic Resonance Imaging The presence of plaques on MRI is a key finding in establishing the diagnosis of MS.

A 26 year-old male was lifting a heavy object two weeks ago when he felt a sudden onset of low back pain. He describes pain in the low mid back at the belt line aggravated with movement. Radicular symptoms are noted in the left buttock down the leg to the dorsal aspect of the foot. He denies any urine or bowel complaints His examination demonstrates an inability to stand on his toes and a positive straight leg raise. Which of the following is most appropriate diagnostic study in this patient?

Magnetic resonance imaging (MRI) MRI is the diagnostic study of choice in a patient with suspected disc herniation. Most common level is L4-L5, L5-S1 Signs and Symptoms: Sharp pain radiating down back of leg in the distribution of the affected nerve Tests: Positive Straight leg raise (SLR), X-ray (negative usually), MRI is necessary for evaluation of soft tissue Not all pain is a herniation, not all herniations cause pain!

A 38-year-old man arrives at the emergency room with the chief complaint of hematemesis for 2 hours. He does not drink alcohol, take NSAID's or ASA and has no significant past medical history. He spent the previous night vomiting approximately 10-12 times after eating some "bad chicken". The patient is squirming on the stretcher and is retching. He is afebrile with a heart rate of 120/min and a BP of 90/60 mm Hg. Abdominal exam is positive for diffuse tenderness but the patient has no rigidity, guarding, or rebound tenderness. There is no hepatosplenomegaly. Rectal exam is negative for occult blood. A nasogastric tube is inserted and reveals bright red blood. Which of the following is the most likely diagnosis?

Mallory-Weiss tear

Essential tremor can be familial and cause action related hand tremor, head tremor, or voice tremor. The lower extremities are spared and generally no further neurologic findings are present.

Masked facies The patient symptoms are consistent with Parkinsonism. Physical exam findings include masked facies, micrographia, decreased arm swing, and monotonous speech.

Neoplasms of the islets of Langerhans:

May produce insulin, glucagon, or gastrin

A patient presents with intermittent vertigo after a recent URI. The patient denies tinnitus or hearing loss. On physical examination, it is noted that the vertigo is present with the patient's eyes open or closed. Which would be the best management of this condition?

Meclizine, steroids po Dx: Vestibular neuritis

A 34-year-old male presents with complaints of increasing swelling and pain in his left knee since a basketball game yesterday. He states that during the game he twisted his knee and had some sharp discomfort, but was able to keep playing. This morning he awoke with increased swelling and pain as well as difficulty bending the knee. He states that the knee appears to "lock up" on him. On PE, there is moderate swelling of the left knee with tenderness to palpation along the medial joint line with limited range of motion, especially in extension. Pain and swelling prohibit any further examination at this time. The most likely diagnosis is

Medial meniscus tear

A 47-year-old white male comes to you for a routine examination. He is physically fit and has been a sun worshipper all his life. He is blond and has blue eyes, but states that he tans easily and does not burn. Examination reveals a nonpruitic, nonpainful, hyperpigmented lesion with red and brown coloration and irregular borders on his upper back. There are no palpable nodes. The skin lesion seen here grossly is most likely to be a(an):

Melanoma An irregularly shaped cutaneous lesion that is hyperpigmented or variegated in color is highly suspicious of melanoma. Early recognition of such a lesion is imperative if a cure is to be possible, and biopsy is necessary to confirm the diagnosis.

A 45 year-old patient with type 1 diabetes mellitus is being screened for diabetic nephropathy. Which of the following urinalysis findings is most consistent with early diabetic nephropathy?

Microalbuminuria

An 8-year old girl is brought in because of increasing weight gain and frequent mild colds. On physical exam she has 2+ edema to the knees and elbows bilaterally. Lab tests reveal a decreased serum albumin of 2.4g/dL and a decreased total serum protein of 5.3 g/dL. Urinalysis shows 4+ protein. Her symptoms dramatically improve after she is given 4 weeks of prednisone. What is the most likely diagnosis?

Minimal change disease

A 13 year-old patient is hospitalized with a fever of 102.5 F and a rash. After 36 hours the rash has rapidly progressed to enlarging macules that appear ring or crescent shaped with central clearing. He also complains of multiple arthralgias involving his ankles, knees, and now his elbows. The EKG shows evidence of a 1st AV block. Labs were significant for an elevated ESR and leukocytosis. Which of the following physical examination findings would be most likely in this patient?

Mitral regurgitation murmur Jones criteria of rheumatic fever include 1 major (erythema marginatum) and 3 minor (fever, polyarthralgies, prolonged PR interval). Cardiovascular manifestations typically associated with rheumatic fever include mitral regurgitation.

A 56 year-old female four days post MI presents with a new murmur. On examination the murmur is a grade 3/6 pansystolic murmur radiating to the axilla. She is dyspneic at rest and has rales throughout all her lung fields BP 108/68, HR 7. Which of the following would be the definitive clinical intervention?

Mitral valve replacement MVR is the definitive intervention to correct MR caused by papillary muscle rupture.

A 19 year-old college student presents to the student health center complaining of a sore throat and mild flu-like symptoms for 2 days. A rapid strep screen and culture are negative. Symptomatic therapy is instituted. The patient returns 3 weeks later complaining of continued sore throat with increasing fatigue and onset of fever. Physical examination reveals an exudative pharyngitis, tender enlarged posterior cervical lymphadenopathy, and the spleen is palpable. Which of the following is most appropriate as the next step in diagnosis?

Mono spot With the presence of exudative pharyngitis associated with posterior cervical lymphadenopathy and splenomegaly, infectious mononucleosis is the most likely diagnosis. A mono spot would most likely be positive since the patient has had symptoms for more than 3 weeks

A person with type 2 diabetes who requires insulin brings in a home monitoring record for review. The current insulin regimen is 15 units of NPH with 5 units of regular in the morning, and 10 units of NPH with 5 units of regular in the evening. A trend of elevated blood sugar readings occurring at noon should prompt an insulin increase of which of the following?

Morning regular dose The elevated noon blood sugar reading indicates a need to increase the morning regular dose

Which of the following Rh genotypes in a mother and father would represent a risk for hemolytic disease of the newborn?

Mother Rh-negative, father Rh-positive If an Rh-negative woman carries an Rh-positive fetus, she may develop antibodies against Rh when fetal blood cells enter her circulation

A 58 year-old male presents with a 3 month history of spastic paresis in his legs. Today, he has experienced diplopia on lateral gaze and scotoma in his right eye. Examination reveals decreased visual acuity of the right eye. Which of the following MRI findings is most likely in this patient?

Multifocal white matter disease The patient is experiencing symptoms of multiple sclerosis (MS). MRI findings demonstrate multifocal white matter disease.

What physical examination finding is most specific for acute cholecystitis?

Murphy's sign A sharp increase in tenderness with a sudden stop in inspiratory effort constitutes a postive Murphy's sign and is most specific for cholecystitis.

A 24-month-old child is not walking but does scoot across the room on her bottom; otherwise, she is doing well. During her well care visit, additional testing for the following should be recommended?

Muscular dystrophy Inherited myopathies resulting from defects in a number of genes required for normal muscle function: characterized by progressive muscle weakness and wasting

A 26-year-old woman presents with the chief complaint of weakness that worsens throughout the day. She especially notices weakness and feeling tired when chewing food. The patient states that she feels strong upon arising in the morning but the weakness develops over the course of the day. She also complains of her eyelids drooping and occasional diplopia. Neurologic examination reveals ptosis after 1 min of sustained upward gaze. Which of the following is the most likely diagnosis?

Myasthenia Gravis

Five months following exposure to a household case of active pulmonary tuberculosis, a 28-year-old HIV-positive man presents with fever, chills, rash, weight loss, and nonproductive cough for 2 weeks. A CD4 count 3 months ago was 380 cells per cubic millimeter; A CXR reveals basilar interstitial infiltrates without cavities, adenopathy, or pleural effusion. A PPD test with controls reveals cutaneous anergy. Which of the following is the most likely explanation for this illness?

Mycobacterium tuberculosis

You are called to see a 25-year old recently immigrated Nicaraguan male who is in the Emergency department. He reports that he was well till 3 months ago when he developed night sweats, cough, fatigue and a low-grade temperature. He also reports a 20 lb weight loss, and several episodes of hemoptysis in the last week. The patient denies IV drug use, and has never received a blood transfusion. His CXR shows upper lobe bilateral cavitation. You suspect

Mycobacterium tuberculosis

The patient above (OSA) is found to have recurrent episodes of arterial desaturation-about 15 events per hour-with evidence of obstructive apnea. The initial treatment of choice for this patient is

Nasal continuous positive airway pressure In this patient with multiple episodes of desaturation, continuous positive airway pressure would be the recommended therapy. Weight loss is often helpful and should be recommended as well, but would probably not be sufficient. Uvulopalatopharyngoplasty has also been used in obstructive sleep apnea, but when applied to unselected patients is effective in less than 50%. Tracheostomy is a course of last resort that does provide immediate relief

A 45 year-old male presents to the clinic complaining of morning sluggishness, daytime fatigue, headaches. He admits to drinking two cocktails each evening. His bed partner reports his loud cyclical snoring, breath cessation and thrashing movements of his extremities during sleep. BMI is 40. Heart examination reveals regular rate and rhythm without S3, S4, or murmur and lungs are clear to auscultation. Polysomnography shows apneic episodes lasting as long as 60 seconds. Which of the following clinical interventions would most likely provide for the acute cessation of apneic episodes?

Nasal continuous positive airway pressure Nasal CPAP is curative in many patients with sleep apnea.

A 30 year-old male presents with a 1 week history of intermittent left orbital headache. He states that his episodes occur at night and last approximately 1 hour. Examination reveals ptosis, miosis and dyshidrosis. Which of the following would most likely be present in this patient?

Nasal lacrimation This patient is exhibiting signs and symptoms of cluster headache which often includes nasal lacrimation.

A 52-year-old woman with a 20 year history of insulin-dependent diabetes mellitus presents complaining of a rash that has developed on her legs and ankles. Physical examination reveals several oval-shaped plaques with demarcated borders and a glistening yellow surface located on the anterior surface of the legs and dorsum. Which of the following is the most likely diagnosis?

Necrobiosis lipoidica diabeticorum Classically well-circumscribed, firm, depressed, waxy, yellow-brown, lesions on the skin. 60% have DM, another 20% have glucose intolerance or FHx. Women are 3 times more affected Tx: stop smoking and optimize diabtic control, Intralesional and/or topical corticosteroids

An afebrile 50 year-old male presents with an acute onset of an exquisitely painful first metatarsophalangeal joint. Examination reveals a red and swollen joint with tenderness on palpation. Which of the following test results will most likely be found in this patient?

Negative birefringent crystals Negative birefringence of the needle-shaped crystals associated with gout are seen by a yellow colorization on polarized light microscopy.

Hypertension, hematuria, and dependent edema are the classic symptomatic triad of what type of kidney disease?

Nephritic syndrome

You are on hospital rounds and you evaluate a 50 year old male with known glomerulonephritis. Today, the patient has worsening generalized edema, hypotension, proteinuria in excess of 3 grams per day and hypoalbuminemia. All of these findings are consistent with:

Nephrotic syndrome

A 50-year-old man was diagnosed at age 15 with type 1 diabetes mellitus. His disease has been poorly controlled, as evidenced by elevated hemoglobin A1C levels. He develops a non-healing ulcer of his foot at age 35. At age 45, he has an increasing serum urea nitrogen and a urinalysis shows sp gr 1.012, pH 6.5, 1+ protein, no blood, 1+ glucose, negative leukocyte esterase, negative nitrite, and no ketones. Which of the following renal diseases is he most likely to have?

Nodular glomerulosclerosis

A 72-year-old woman has had increasing fatigue with a 3 kg weight loss over the past 7 months. Her hands become purple and painful upon exposure to cold. On physical examination, she has a palpable spleen tip. Laboratory studies show Hgb 10.5 g/dL, Hct 31.7%, MCV 99 fL, platelet count 193,600/microliter, and WBC count 5390/microliter. The direct Coombs test is positive at 4 C and negative at 37 C. Which of the following underlying diseases is this woman most likely to have?

Non-Hodgkin lymphoma Large group of malignancies involving the lymphocytes with a wide range of severity and Px Poorer Px than Hodgkin's Signs & Symptoms: Painless lymphadenopathy, fever, wt loss, night sweats Labs: CBC can be normal may show anemia, or increased WBC Dx: Lymph node Bx Tx: XRT and chemo survival rates depends on pathology

Which of the following is indicated as the initial management of spinal stenosis?

Nonsteroidal anti-inflammatory A nonsteroidal anti-inflammatory agent and exercise produces a good response in many patients and is indicated as the initial management option.

Which is characteristic in patients with bulimia but not in patients with anorexia nervosa?

Normal Menstruation

A 57 year-old man is being evaluated for shortness of breath. The following spirometric data are obtained: VC 4.90 L (predicted), 5.15 L (observed) 105% predicted FRC 3.99 L (predicted), 4.37 L (observed) 110% predicted RV 2.47 L (predicted), 3.17 L (observed) 128% predicted FEV1 3.50 L (predicted), 2.35 L (observed) 67% predicted These findings are consistent with which of the following?

Obstructive lung disease Spirometry findings in obstructive lung disease typically show normal or increased total lung capacity, decreased vital capacity, prolonged FEV1, and increased residual volume.

For patients undergoing highly emetogenic chemotherapy regimens, which of the following would be the first-line antiemetic medication?

Ondansetron (Zofran) A 5HT3 receptor blocker such as ondansetron is the agent of choice, given its high efficacy and low risk of side effects (headache being the major, and usually easily manageable, one).

An 8 year-old female presents with a 5 day history of a honey crusted rash beneath her nose. Which of the following is the best treatment for this condition?

Oral cephalexin (Keflex) Impetigo is an infection of the skin, often caused by S. aureus or S. pyogens, characterized by honey crusted erosions. Treatment includes topical mupirocin followed by oral cephalosporin.

Which of the following is the best treatment for this child (bullous impetigo)?

Oral dicloxacillin Bullous impetigo that is localized may be treated with topical Bactroban mupirocin, but extensive involvement is best treated with oral antibiotics that are penicillinase-resistant, such as dicloxacillin. Etiology caused by S. aureus or Strep pyogenes = Group A beta-hemolytic streptococcus Extension into the dermis is called Ecthyma Primary infections are common in children secondary at any age bullous in children and young adults May persist for days to months Treatment: oral antibiotics: dicloxacillin, cephalexin, Augmentin macrolides topical benzoyl peroxide 5% or 10% mupirocin (Bactroban) 2% cream or ointment eliminates S. aureus (including MRSA) and Group A strep apply TID x 10 days

A 16 year-old female comes to the office because her acne has become more severe and she would like to discuss treatment. She was originally prescribed topical clindamycin gel 1 year ago to treat open and closed comedones on her face. Despite regular use of the medication, she now has several comedones and papulopustules across her nose, cheeks, chin and forehead. Which of the following would be an appropriate additional treatment for moderate acne?

Oral doxycycline For moderate acne, oral antibiotics are added to the topical medications treatment regimen

The major hazard associated with antihypertensive therapy in the elderly is:

Orthostatic Hypotension

An 85 year-old nursing home resident presents with abrupt onset of cough, sore throat, headache, myalgias, and malaise. On examination the patient's temperature is 102 degrees F; the rest of the exam is unremarkable. Nasal smear is positive for Influenza B. Which of the following is the treatment of choice in this patient?

Oseltamivir (Tamiflu) Oseltamivir is used to treat both influenza A and B.

A 14-year old male active in sports, has been complaining of intermittent anterior right knee pain for several months. He denies any specific injuries. On examination, there is no erythema, swelling, deformities, joint laxity, or crepitus. Palpation reveals tenderness over the tibial tubercle and bursa of the right knee. X-Ray seen here This finding is characteristic of which of the following disorders?

Osgood-Schlatter disease Osgood-Schlatter disease causes pain at the tibial tubercle and it is caused by fragmentation of the tip of the proximal tibial physis

Bony and cartilaginous enlargement of distal interphalangeal joints is commonly seen in which of the following medical conditions?

Osteoarthritis Heberden's nodes are commonly seen in primary osteoarthritis

A 37-year-old Caucasian woman swims regularly for exercise. She starts to notice severe right ear pain and itching. She sees her family doctor and mentions her complaints. When he goes to insert the otoscope, he gently pulls on her ear which causes her much pain. He notes an inflamed external ear canal, but the tympanic membrane is normal. What is the most likely diagnosis

Otitis externa

A 75-year-old presents with a history of dizziness and syncopal episodes. EKG is below (shows 2nd or 3rd Degree AV Block) Which of the following is the treatment of choice for this patient? Which of the following is the treatment of choice for this patient?

Pacemaker placement

What is the most typical presentation of new-onset Crohn's disease

Pain, nonblood diarrhea, weight loss and malaise Most commonly involves terminal ileum and adjacent colon. Transmural (whole thickness of the colon) involvement with skip lesions. Diarrhea (most are non-bloody) RLQ pain; Low grade fever; Weight loss Perianal disease with fistula and abscess. Almost never involves the Rectum. (+) ASCA (antisaccharomyces cervisiae antibody)

In a patient with dyshidrotic eczema on which area of the body would associated vesicles be found?

Palms Vesicles associated with dyshidrotic eczema are most commonly found on the palms with the dorsum of the hands spared. The soles may also be affected in a similar fashion.

During a physical exam of a 55 y/o woman, who is a heavy smoker c/o of left eye swelling, dry eyes and left shoulder pain, you notice that the patient's left eyelid is drooping, the conjunctiva is dry, the left pupil is smaller than the right and she has left shoulder pain with certain movements Considering these findings, you suspect?

Pancoast's syndrome Due to tumor invading apex and superior sulcus and invasion of brachial plexus and cervical sympathetic nerves. Patients present with arm and shoulder pain and signs of Horner's syndrome. Horner syndrome, typified by a constricted pupil, partial ptosis and loss of sweating is associated with apical lung cancers

A 68 year-old male presents with jaundice, weight loss, and boring abdominal pain which radiates to the back. The gallbladder is palpable on physical examination. This finding is most consistent with which of the following?

Pancreatic tumor A large palpable gallbladder resulting from pressure from a tumor in the pancreatic head is known as Courvoisier's sign.

A patient presents with pain and swelling of the distal middle digit. Pus is noted at the lateral nail fold. Which of the following is the most likely diagnosis?

Paronychia Etiology inflammation of the nail fold may extend to the eponychium may be acute or chronic caused by C. albicans or S. aureus increased incidence: hands are in a wet environment repeated minor trauma to cuticle thumb sucking

What is the most common examination finding in a patient diagnosed with mumps?

Parotid gland tenderness

A 5 year-old male is brought to the office by his father who reports a 2 day history of low grade fever and coryza. The child awoke this morning with bright red cheeks. Physical examination reveals edematous, confluent plaques over the malar region of the face and reticular rash over the child's extensor surfaces. Which of the following is the most likely diagnosis?

Parvovirus B19 This is a classic presentation of Fifth Disease, a childhood exantham associated with human parvovirus B19.

A 37 year-old right-handed landscaper comes to the office for evaluation of recurrent itching and stinging of the skin on his right hand. Physical examination reveals confluent papules, vesicles, erosions and crusts on the dorsum of his right hand. Which of the following is the most appropriate initial diagnostic study in this patient?

Patch testing In patch testing, substances are applied to the skin in shallow cups and left in place for 24-48 hours. Sensitivity to an allergen is confirmed by a papular vesicular reaction that occurs within 48-72 hours of the laboratory testing.

A 51 year-old patient presents for follow-up after a recent stroke. Cardiac examination reveals a moderately loud SEM in the 2nd and 3rd interspaces parasternally. S2 is fixed and widely split. Which of the following is the most likely diagnosis in this patient?

Patent foramen ovale A patent foramen ovale is present in 25% of the population and can lead to paradoxic emboli and cerebrovascular events. Suspicion should be highest in patients who have cryptogenic stroke before age 55 years.

Primarily, diagnosis of valvular heart disease is made by

Patient history, physical exam, echocardiography, and Doppler The history and physical exam remains the cornerstone for diagnosis of acquired valvular heart disease. Radiographic studies such as echocardiography and color-flow Doppler studies confirm the diagnosis and provide vital information with regard to severity of disease.

A 60-year-old male with a 50 PPY smoking history is evaluated for recent unintentional weight loss. A biopsy of a small mass found on CXR is consistent with squamous cell carcinoma with no evidence of metastasis. Which of the following is the recommended management of choice?

Patient should undergo surgical resection

Which of the following pulmonary function tests is most easily carried out at home?

Peak expiratory flow rate

Which of the following is considered to be the first EKG evidence of acute myocardial infarction?

Peaking of T waves The initial EKG of a patient experiencing an acute MI may not reveal any significant changes at all. When an infarction pattern does occur, the first change that is seen is the peaking of the T waves.

Which of the following patients is at highest risk for the development of the skin disorder known as erythrasma?

People living in tropical climate People living in warm, tropical climate, people wearing occlusive clothing or shoes, obese patients, and those with hyperhidrosis are at increased risk for erythrasma. The diagnosis of this condition is made by demonstration of a coral red fluorescence

A 50 year-old male with history of alcohol abuse presents with acute, severe epigastric pain radiating to the back. The patient admits to an episode of coffee ground emesis. On examination he is ill-appearing with a rigid, quiet abdomen and rebound tenderness. Which of the following is the most likely diagnosis?

Perforated duodenal ulcer Perforation of a duodenal ulcer causes sudden, severe pain, with rebound tenderness and rigid abdomen on physical examination. It is often associated with coffee ground emesis.

A 60 year-old female with a 30 pack year smoking history complains of new onset shortness of breath. On physical examination, dullness is noted on percussion with diminished breath sounds over her left base. Chest x-ray shows a new left pleural effusion. Which of the following is the next step in the management of this patient?

Perform diagnostic thoracentesis Diagnostic thoracentesis should be performed whenever there is a new pleural effusion and no clinically apparent cause

A 59-year-old diabetic woman had suffered an acute AWMI. Five days later she gets into an argument with her husband and complains of chest pain. Her initial EKG shows no ischemic changes, but serum cardiac troponin levels are drawn and return mildly elevated at this time. Which of the following is the best next therapy?

Perform serial ECGS and obtain CK-MB

A 25 y/o female presents with a three day history of CP aggravated by coughing and relieved by sitting and leaning forward. She is febrile and a CBC with differential reveals leukocytosis. Which of the following PE signs is characteristic of her problem?

Pericardial friction rub Pericardial friction rub is characteristic of an inflammatory pericarditis.

Which of the following is the most likely chest radiograph presentation of adenocarcinoma of the lung?

Peripheral lesion Adenocarcinoma most commonly presents as a solitary, peripheral lesion on chest radiograph. It is sometimes associated with an ipsilateral pleural effusion.

A 56 year-old male is noted to have a recent diagnosis of polycythemia vera. His current hemoglobin is 21 gms/dl. What treatment should be instituted for this patient at this time?

Phlebotomy This patient has a diagnosis of polycythemia vera treatment begins with phlebotomy instituted on a weekly basis until the hematocrit is less than 45%. Maintenance of the hematocrit at 45% is achieved with repeated phlebotomy as necessary

An adult patient is found to have an elevated serum TSH in association with an elevated total serum free T4 index. The most likely explanation for these findings is:

Pituitary adenoma Rare condition Pituitary adenoma producing exogenous TSH T 4 will be elevated

An 18 year-old young woman comes to the office with a mildly itchy rash on her chest and back for the past two weeks. She noticed an initial area on her back two weeks ago, and now it has spread across her back and chest. On physical examination, she has several salmon-colored plaques with fine scale that follow the cleavage lines of the trunk. Which of the following is the most likely diagnosis?

Pityriasis rosea Pityriasis rosea is an acute eruption of fine scaling fawn-colored papules and plaques that are distributed along the cleavage lines of the trunk, often referred to as a Christmas tree pattern. A single plaque, called a herald patch, precedes the secondary eruption by 1-2 weeks.

A 47-year-old man presents with a history of worsening burning left heel pain for the last few weeks. He states that it is worse in the morning after getting out of bed and beginning to walk, then improves with exercise. He denies any foot numbness, tingling, or known injury. He states that he recently began a jogging regimen. On examination, Thompson test is negative.What is the most likely diagnosis?

Plantar Fasciitis Common cause of foot/heel pain; Plantar pain upon getting out of bed Inflammation of the plantar fascia as it inserts into anterior calcaneus. Often associated with a bony exostoses on calcaneus i.e. a "spur" Tx: Ice, NSAIDS, Stretching, arch supports Responds very well to local steroids

A 20-year-old college student presents with a 3-mo history of left sided pleuritic chest pain, shortness of breath with exertion, and night sweats. He admits to a 10-lb weight loss over the last several months. He is a nonsmoker and does not use illegal drugs. He is heterosexual. He recalls a negative PPD when he started college 2 years ago. On physical examination, his temperature is 100.9 F and his respiratory rate is 24/min. Lung examination reveals decreased fremitus, dullness to percussion, and diminished breath sounds over the left posterior lung. A pleural friction rub is audible at the left lung base. Which of the following is the most likely diagnosis?

Pleural effusion

Which of the following prophylactic interventions is recommended in all individuals with HIV regardless of disease stage?

Pneumococcal vaccine All HIV-positive individuals should receive prophylaxis against pneumococcal pneumonia

During the evaluation of a post motor vehicle accident patient who incurred blunt abdominal trauma, you notice a Grey-Turner's sign. He undergoes corrective surgery. Which of the following is an appropriate preventive strategy for this patient?

Pneumococcal vaccine For a patient who has functional or anatomic asplenia, pneumococcal vaccine should be administered as the spleen is specifically responsible for combating infection by streptococcus pneumoniae.

While examining a patient's chest you discover an area with absent breath sounds, increased tactile fremitus and egophony. Which of the following conditions do these findings most likely represent?

Pneumonia These physical exam findings most often correlate with an increased density of the pulmonary tissues. This would most commonly be caused by a consolidation secondary to pneumonia.

On a frontal chest radiograph view, you notice a visceral pleural line with a radiolucent area devoid of vascular and pulmonary markings on the right side only. Which of the following is the most likely diagnosis?

Pneumothorax Unilateral findings of a visceral pleural line with no vascular pulmonary markings in between this line in the chest wall are almost always indicative of a pneumothorax. Certain conditions like emphysema can result in a pneumothorax but these findings are not diagnostic of emphysema

Which of the following is the most sensitive laboratory test to diagnose herpes simplex virus?

Polymerase chain reaction (PCR) Diagnosis is usually made clinically however, PCR is more sensitive than culture.

Which of the following is the most common cause of ascites?

Portal hypertension Over 80% of patients with ascites have portal hypertension secondary to chronic liver disease. Infection, malignancy, and nephrotic syndrome are common causes of non-portal hypertensive ascites.

A patient with Pseudogout will most likely present with which of the following?

Positive birefringent calcium crystals

A 72-year-old black male presents after several days of voiding difficulty complaining of complete loss of urine production, although he has the desire to void. He denies any pain, fever, chills, or recent illness. He admits to a 22-year history of diabetes mellitus and a 2-pack-per day smoking habit. The most likely cause of his oliguria/anuria is

Postrenal anuria, due to bladder outlet obstruction

A 74 year-old female presents to the ED with nausea, vomiting, malaise, and confusion. The patient lives alone, but was brought in by her daughter who stopped by to visit and noticed the changes in her mother since she had last seen her 48 hours before. Her urinalysis is negative for infection and her BUN: Creatinine ratio is greater than 20:1. The patient is most likely suffering from which category of acute kidney failure?

Prerenal AKI

A66 year-old woman complains of skin itching. The symptom is most prevalent on the trunk and lower limbs and she has realized that the itching is worse after a hot bath or on retiring to bed at night. There are no domestic pets, she has not used new brands of washing powder or cosmetics and she has not noted an associated rash or fever. She mentions that over the past year a small pimple has developed on both eyelids Physical Examination: There are scratch marks on her back and legs but no discreet rash. Small xanthelasmas are evident on both upper eyelids. There is some pigmentation of the skin around the elbows and along the spine. On abdominal examination a firm liver edge is palpable 3 cm below the right coastal margin, and a top of spleen is also palpable. There is no ascites Lab Tests Liver tests: these indicate cholestasis(raised alkaline phosphatase and gamma-glutamyl transpeptidase) without elevation of bilirubin. The serum cholesterol and IgM levels are abnormally elevated.The most likely diagnosis is:

Primary biliary chirrhosis

A 55 year-old female presents to the clinic with lethargy, fatigue, constipation, and menorrhagia. Physical examination reveals an enlarged thyroid, dry skin, and a heart rate of 50 bpm. Laboratory results show a decrease in free T4, and an elevation in TSH. Which of the following is the most likely diagnosis?

Primary hypothyroidism This is a classic presentation of symptoms of primary hypothyroidism. Symptoms include weight gain, fatigue, lethargy, depression, weakness, constipation, menorrhagia; and patients often present with a palpable, enlarged thyroid

A 30-year-old woman presents with the chief complaint of shortness of breath with minimal activity. In retrospect, she feels she has been dyspneic for at least 1 year but has now progressed to the point where she has difficulty climbing stairs and walking short distances. She denies fever, cough, or chest pain. On physical examination, the patient has jugular venous distension(JVD) and a palpable right ventricular lift. On auscultation there is a loud S2 and a systolic murmur that increases with inspiration. Lungs are clear. There is no clubbing. Which of the following is the most likely diagnosis?

Primary pulmonary hypertension Primary pulmonary hypertension(PPH) is of unknown etiology and primarily affects women in their thirties and forties. The underlying problem it the disorder is a fixed increased resistance to pulmonary blood flow. Pulmonary function in PPH is usually normal, but the elevation in pulmonary artery pressure causes a decrease in cardiac output and eventually right ventricular failure. Patients become dyspneic and hypoxemia due to mismatch of pulmonary ventilation and perfusion and the reduced cardiac output. Physical examination reveals signs of right ventricular hypertrophy, right- and left-sided heart failure, and tricuspid and pulmonic regurgitation. The mean survival for this disease is 2-3 years from the time of diagnosis.

Which of the following is the most common presenting clinical manifestation of a patient with interstitial lung disease?

Progressive dyspnea with exertion

A 22 year-old female patient presents with palpitations and episodes of CP at rest. These episodes are very bothersome to the patient. PE reveals a mid-systolic click and a late systolic murmur. Holter monitoring reveals frequent PVCs during the episodes of palpation. Treatment of choice is:

Propanolol This patient has evidence of MVP and is symptomatic with palpitations and chest pain. The most efficacious drug in this setting has shown to be propanolol. The risk of type 1A antiarrhytmic drugs such as quinidine and procainamide far outweigh the potential benefits in this clinical setting. Verapimil and digoxin do not show specific efficacy in this setting

What is the most effective prophylaxis against respiratory syncytial virus (RSV) infection in the general pediatric population?

Proper hand-washing techniques Proper hand washing and reduction in exposure is most effective in general population to prevent RSV.

A 58 year-old female presents with acute onset of 105 degrees F fever, chills, delirium and tachycardia. Laboratory analysis reveals a TSH of 0.08 mcU/L (0.4-6.0 mcU/L), total T3 of 400 ng/dL (95-190 ng/dL)and a total T4 of 180 mcg/dL (5-11 mcg/dL). What is the initial treatment of choice to normalize this condition?

Propylthiouracil Propylthiouracil is the preferred initial drug in patients with thyroid storm

A 34 year-old man comes to the urgent care with fever of 102 degrees F, chills, and perineal pain for the past 2 days. In addition, he has difficulty voiding, dysuria, frequency, urgency, and nocturia. Which of the following physical examination findings are consistent with this history?

Prostate tender, warm, swollen

Which of the following is characterized by epidermal hyperplasia and an increase in the epidermal turnover?

Psoriasis Psoriasis is characterized by an increased epidermal cell turnover, increased numbers of epidermal stem cells, and an abnormal differentiation of keratin. This leads to the classic scale associated with psoriasis.

Silvery-scaled, plaque-like lesions commonly found on the extensor surfaces of the extremities are most likely:

Psoriasis Silvery-scaling plaque-like lesions found on the extensor surfaces of the extremities is strongly suggestive of psoriasis. T-Disordered of the immune system cells become over-stimulated increases risk of developing cardiovascular disease, IBD, and cancer Etiology: inherited drug-induced antihypertensives - BB, CCB, and Captopril NSAIDs Glyburide statins TNF inhibitors Lithium Anti-malarials antifungals (Terbinafine)

A 58 year-old with a 20 pack-year history of tobacco use presents to the emergency department with sudden onset of severe, progressive right-sided chest pain and shortness of breath. She denies fever, nausea and productive cough but reports returning from a mission trip to South Africa yesterday. Her exam reveals tachycardia, tachypnea, clear lung fields and 3+ edema and erythema of the left leg. Her chest radiograph is normal. Which of the following is the most likely diagnosis?

Pulmonary embolism The patient's risk factors, acutely progressive presentation and normal chest radiograph make a pulmonary embolism the most likely and most worrisome diagnosis.

A 55- year-old man with emphysema will have which kind of respiratory pattern of breathing?

Rapid and shallow breathing

A 27 y/o man presents because of a painful, swollen knee and ankle of 2 weeks' duration. He has never had joint disease prior to this time. The patient also complains of low back pain and a recent history of clear penile discharge. On examination he has vesicles (some of which have crusted over) on the palms, soles, and glans penis; injected conjunctivae; a swollen right index finger; and arthritis of the right knee and left ankle. The MOST likely diagnosis is

Reactive Arthritis(Reiters Syndrome) Acute nonpurulent arthritis complicating an infection elsewhere in body Up to 85 % of pts possess the HLA-B27 alloantigen Reiter's Syndrome first described which describes triad of arthritis, conjunctivitis, & nongonococcal urethritis Only a minority of patients have the other classic findings associated with reactive arthritis, including urethritis, conjunctivitis, oral ulcers and rash Epidemiology Follows ~ 1 % of cases of nongonococcal urethritis & 2 % of enteric infections(Yersinia enterocolitica, Shigella flexneri, Campylobacter jejuni, Salmonella spp.) or by GU infection with Chlamydia trachomatis & possibly by other agents It is thought that in individuals with appropriate genetic background reactive arthritis may be triggered by an enteric infection with any of several Shigella, Salmonella, Yersinia, & Campylobacter species

Which of the following is the most appropriate study for diagnosing Hirschsprung disease?

Rectal biopsy A rectal biopsy showing the absence of ganglion cells in both the submucosal and muscular layers of the involved bowel is the most appropriate diagnostic study for Hirschsprung disease.

A 68 year-old male has increasing serum creatinine and decreased urinary output. Which of the following is the safest and best imaging study for this patient?

Renal Ultrasound

A patient is complaining of constant nasal congestion for a month. It started with URI symptoms a month ago, but she only has nasal congestion now. When she blows her nose, it is clear and she has no fever. She has been using over the counter oxymetazoline since symptoms began. What is the most likely diagnosis?

Renitis medicamentosa

A 40-year-old man asks your advice about lipid lowering therapy. He states that his father died of a heart attack at age 48 and that he smokes 1 pack of cigarettes per day. He currently takes no medications, and has no known medical problems. You order a fasting lipid profile, with the following results: Total cholesterol = 252, Triglycerides 230, LDL = 174, HDL = 32. You recommend the following, except:

Repeat fasting lipid profile after 6 months of a low-cholesterol diet, and start Pravastatin at that time if his cholesterol remains elevated. As this patient has 2 risk factors for premature CAD, NCEP guidelines call for treatment with diet and drug therapy for patients with a LDL level > 160. For LDL levels between 130-159 choice D above could be followed.

Which of the following is the most common cause of pneumonia and bronchiolitis in children < 1 year of age

Respiratory syncytial virus Dx: Acute Bronchiolitis

A 34 year-old female 3 days status-post appendectomy complains of mild diffuse abdominal pain with associated nausea and vomiting. On examination you note diminished bowel sounds. There is generalized abdominal distension with mild tenderness on palpation. There are no signs of peritoneal inflammation. Abdominal radiographs show distended gas-filled loops of the small and large intestine. Abdominal CT scan shows no mechanical obstruction. Which of the following is the most appropriate management for this patient?

Restrict oral intake Most adynamic ileuses respond to restriction of oral intake and gradual advancement of diet as bowel function returns.

An elderly appearing adult male is transported to the emergency room with unconsciousness for an underdetermined amount of time. There is no family and the only history is provided by the paramedics. The patient arouses to verbal and painful stimuli. VS: T-97.0 degrees F rectally, P-52 bpm, R-10, BP-95/60 mmHg. Physical examination is unremarkable except for ecchymosis across his extremities. A Foley catheter is inserted draining a small amount of dark brown urine. Urine dipstick reveals 4+ positive hemoglobin and protein. Microscopic urinalysis reveals no RBCs but many renal tubular epithelial cells and renal tubular casts. Drug screen is negative, blood alcohol is 2.5 mg/dL, and creatinine is 4.9 mg/dL. What is the most likely diagnosis?

Rhabdomyolysis causing acute renal failure

A patient comes to you with sores in the corner of their mouth. You diagnose angular cheilosis. What vitamin deficiency is associated with angular cheilosis?

Riboflavin (B2)

A 5 year-old child has just been hospitalized with meningococcemia. Family members and close contacts should be given which of the following as prophylaxis?

Rifampin orally Exposed household, school, or day-care contacts of children with meningococcemia should receive chemoprophylaxis to eliminate the organism from the nasopharynx.

A 23-year -old woman complains of dyspnea and substernal chest pain on exertion. Evaluation for this complaint 6 months ago included arterial blood-gas testing, which revealed pH 7.48, PO2 79mm Hg, and PCO2 31 mm Hg. Electrocardiography then showed a right axis deviation. Chest x-ray now shows enlarged pulmonary arteries but no parenchymal infiltrates, and a lung perfusion scan reveals subsegmental defects that are thought to have a "low probability for pulmonary thromboembolism." Venogram performed was normal. The most appropriate diagnostic test now would be

Right-heart catherization to measure pulmonary artery pressures

A patient presents with a rash six days after returning from a camping and hiking trip in the woods. The rash is macular and first appeared on the ankles and then the rest of the lower extremities. On exam the physician assistant notes lesions on the soles of the feet and the trunk. Which of the following is the most likely diagnosis?

Rocky mountain spotted fever Rocky mountain spotted fever presents with a macular rash on the wrists, ankles, extremities, and trunk. After 5 days the rash appears on the palms and soles

Edema and hyperplasia of nasal tissues (rhinophyma) are associated with

Rosacea Rhinophyma-is a common complication Face usually appears red or flushed. Telangiectasia are often present . Pt may have scattered small inflammatory papulopustules and sometimes nodules occurring cheeks, chin, forehead, glabella and nose. Treatment: Avoid triggers-heat, sun, spicy food, alcohol -Topical metronidazole, sodium sulfacetamide, azelaic acid Oral antibiotics TCN, minocycline, or doxycycline if topical treatment fails

A 60 year-old right hand dominant male presents to your office complaining of right shoulder pain and progressively worsening arm weakness. His symptoms are aggravated when working above the shoulder level. On physical examination, there is no evidence of muscle atrophy. He has strong (5/5) adduction of his right shoulder but considerable weakness (1/5) with abduction when raising his arm above his head. Passive range of motion is intact. Which of the following is the most likely diagnosis?

Rotator cuff tear Rotator cuff tears are associated with full passive range of motion but have pain and weakness during active abduction. The "drop arm test" is useful to diagnose:

The "drop arm test" is useful to diagnose:

Rotator cuff tears

A patient presents with nausea, vomiting, and fever for one day. She describes periumbilical abdominal pain which has settled in her right lower quadrant. On examination she has right lower quadrant pain with deep left lower quadrant palpation. This finding is known as which of the following?

Rovsing's sign Right lower quadrant pain with deep left lower quadrant palpation is Rovsing's sign. Rebound tenderness is with quick withdrawal of RLQ palpation.

A 45 year-old female presents with bowel movements associated with bright red blood after being diagnosed with a stage II hemorrhoid 1 month ago. She has used increased fiber and fluid intake as her primary treatment up to this point. Which of the following is the most appropriate next step in managing this patient's condition?

Rubber band ligation Patients who have a stage I, II or III hemorrhoid with recurrent bleeding not responsive to conservative therapy should have injection sclerotherapy, rubber band ligation or application of electrocoagulation. Rubber band ligation is easy to use and has a high efficacy rate.

An unimmunized child has had a mild fever for several days. Today, the child is brought to the clinic because of the development of a rash. The rash is a pink discrete macular eruption mostly on the face and trunk. Postauricular and suboccipital lymph nodes are palpable. The child has a slight fever, but does not appear sick. Which of the following is the most likely diagnosis?

Rubella The characteristic lymphadenopathy and lack of systemic symptoms are most consistent with rubella

Which of the following types of hemoglobin is responsible for the development of sickle cell anemia?

SS SS is the dominant hemoglobin found in sickle cell anemia.

A first year medical student fails to use proper disinfection techniques with his microbiology experiment. Two weeks later, he has spiking fevers, cramping abdominal pain with diarrhea, a palpable spleen and leukopenia. What organism was he using in his experiment?

Salmonella typhi Symptoms and Signs Typhoid Fever Average incubation period of 10 d( 3-60 d) Insidious onset of HA, malaise, anorexia, altered sensorium, and fever(which lasts 4-8 weeks and may abate) "Rose spots" (erythematous macules of 2-4 mm on the upper abdomen) present in up to 50% of patients Relative bradycardia Hepatosplenomegaly Complications GI hemorrhage, perforation Localized infections(meningitis, hepatitis, cholecystitis, nephritis, myocarditis and pneumonia) Chronic carrier state in 3-5%; relapse in 20% of treated patients Diagnostic Studies Isolation of organisms from blood(90 % + rate week 1 ; decline thereafter) or stool(75% + by week 3) Serologic testing less reliable Management Ciprofloxacin Chloramphenicol Ampicillin Cefitraxone Trimethoprim-sulfamethoxazole Prevention Improved hygiene Immunization

A 63-year-old woman presents to you with a 5-year history of chronic kidney disease, stage 3. She states that she has not been very good about following her provider's orders and wants to know what things she can do to help her condition. Which of the following is appropriate dietary management for this patient?

Salt, water, phosphorus, potassium, and magnesium restriction with a protein maintenance

A 17-year-old patient presents with severe pruritus that is worse at night. Upon examination of the skin, areas of excoriated papules are observed in the interdigital area. Family members report similar symptoms. Which of the following is the most likely diagnosis?

Scabies Etiology Contagious caused by the mite Sarcoptes scabiei spread by skin to skin contact Clinical Features generalized itching, nocturnal pruritus, burrows in webs of fingers & toes, excoriated/non-excoriated papules, vesicles, pustules, erythema, scaling Explanation: Occurs most commonly in web spaces of hands and feet, genitalia, axillary folds Presents with pruritic burrows, vesicles, and/or nodules with excoriations and crusting Laboratory: Mineral oil drop on burrow, scrape with 15 blade, place on slide, one drop oil on slide, cover slip and look for mite, egg, or feces

A 33-year-old man presents with acute left eye pain. He was working in his garage on a woodworking project and as he hammered in a nail, he felt that something hit him in the left eye. On examination, you note that the left pupil has a teardrop appearance. What diagnostic test/procedure will most likely confirm your diagnosis?

Seidel Test

Which of the following classes of medications is considered first-line for the treatment of panic disorder?

Selective serotonin re-uptake inhibitors SSRIs are considered 1st line for the treatment of panic disorder because of efficacy and side-effect profile.

A 33 year-old female 2 years status-post gastric bypass surgery presents to the office with complaints of generalized fatigue and paresthesias in her hands. Examination reveals pallor, mild glossitis and decreased vibratory sense. Which of the following laboratory tests is most diagnostic for her condition?

Serum vitamin B12 Vitamin B12 deficiency leads to neurologic symptoms of paresthesias, glossitis, and pallor. Gastric bypass surgery aids in malabsorption.

A 54 y/o male stonecutter presents with a chronic cough & dyspnea with exertion. He denies any other symptom such as hemoptysis, chest pain or fever. PMH is unremarkable He smokes 1 pack of cigarettes daily. CXR is seen here. What diagnosis would you suspect in this patient?

Silicosis

A 60 y/o man is admitted to a hospital because of respiratory failure & tachycardia. T 101 F, RR 32 & BP 100/60. His rhythm strip is seen What does the above rhythm strip reveal?

Sinus tachycardia Diagnosis Seen in presence of pulmonary disease Irregular rate & rhythm with multiple but discrete P wave morphologies preceding QRS complexes If atrial rate is < 100, call it multifocal atrial rhythm or wandering atrial pacemaker Treatment Directed toward underlying pulmonary disease Verapamil to control VR

A 60-year-old female complains of dry mouth and a gritty sensation in her eyes. She states it is sometimes difficult to speak for more than a few minutes. There is no history of diabetes mellitus or neurologic disease. The patient is on no medications. On exam, the buccal mucosa appears dry and the salivary glands are enlarged bilaterally. This patient probably has

Sjogren' s Syndrome Immunlogic disorder characterized by progressive lymphocytic destruction of exocrine glands most frequently resulting in symptomatic eye & mouth dryness Can be associated with extraglandular manifestations Predominantly affects middle-age females May be primary or secondary when it occurs in association with other autoimmune diseases Clinical Manifestations Constitutional-Fatigue Sicca symptoms-keratoconjunctivits sicca (KCS) & xerostomia Dryness of other surfaces-nose, vagina, trachea, skin Extraglandular features-arthralgia/arthritis, Raynaud's, lymphadenopathy, interstitial pneumonitis, vasculitis (usually cutaneous), nephritis, lymphoma

A 73 year-old female patient was diagnosed 3 years ago with Alzheimer's disease and heart failure. Her 80 year-old husband can no longer help feed and bathe her or manage her medications. Which of the following support services is most appropriate for this patient?

Skilled nursing facility A skilled nursing facility is appropriate for patients requiring assistance with activities of daily living (i.e., feeding and bathing) and a higher level of safety assurance.

A patient presents with a non-displaced clavicular fracture lateral to the coraclavicular ligament. Which of the following is the treatment of choice?

Sling Mechanism: Fall on shoulder or direct blow to shoulder (football injuries common) vaginal birth, middle shaft is common lesion Signs and Symptoms: Tenderness, swelling, and deformity

Which is the most effective for visualizing corneal foreign bodies?

Slit lamp exam

A 29 year-old patient with idiopathic thrombocytopenia purpura (ITP) is treated with prednisone therapy. Despite therapy, platelet counts remain consistently below 20,000/microliter over the course of 6 weeks. Which of the following is the most appropriate intervention for this patient?

Splenectomy Persistently low platelet counts (< 20,000) require effective long-term treatment, and splenectomy is the treatment of choice

A 52 year-old female is diagnosed with chronic myeloid leukemia (CML). Which of the following would you expect to find on physical examination of this patient?

Splenomegaly CML patients present with an enlarged spleen.

A 28 year-old non-smoking black female presents with an 8 week history of dry cough, fatigue and numerous macular-papular lesions on the face. She denies fever, chills or hemoptysis though she has noticed enlarged, nontender lymph nodes in the neck. Lab tests reveal a normal CBC but her serum ACE level is three times the upper limit of normal. Her chest radiograph, seen here reveals bilateral, hilar adenopathy but is otherwise clear. Which of the following medications is most indicated in this patient?

Steroids Oral steroids have been shown to provide the best treatment for sarcoidosis.

The cover/ uncover exam is used to detect which condition?

Strabismus

Which of the following types of kidney stones occur secondary to infections due to urease-producing bacteria?

Struvite

A 17-year-old male is brought to the ED by his mother and has a "headache like I've never had before." The patient has been completely well, healthy, and active before this episode (which began last night). Nausea and vomiting began shortly after ha's onset. On examination, there is significant neck stiffness. The patient cannot move his neck without extreme pain. At this time, with the information you have now, what is the most likely diagnosis?

Subarachnoid hemorrhage This patient has the classic description of a subarachnoid hemorrhage ("headache like I've never had before" and acute onset) and you should consider it subarachnoid hemorrhage until proven otherwise Most common: leakage from congenital (berry) aneurysm

What is the initial treatment of choice for inflammatory bowel disease?

Sulfasalazine (Azulfidine) Sulfasalazine and other 5-aminosalicylic acid drugs are the cornerstone of therapy in mild to moderate inflammatory bowel disease as they have both anti-inflammatory and antibacterial properties.

A 30-year-old presents with a number of salmon-red oval, slightly raised plaques on the trunk and proximal upper extremities. The patient states the rash started with a single lesion and then spread. What would hasten the resolution of this rash?

Sun exposure Pityriasis Rosea What: papules/ plaques, scaly eruption When: after URI, fall/ spring Cause: unknown, ?Herpes 7 virus Sx:" A herald patch precedes the exanthematous phase: a larger, oval, dull pink to fawn -colored patch with a central collarette of scale. Dx: (-)KOH, (-)RPR Treatment: Reassure patient most likely cause by a virus and expect the rash to spontaneously resolve in 3-8 weeks. Emollients or even low to mid potency cortisone for the itching Antihistamines if significantly pruritic UVB or natural light can be helpful, if started during the 1st week of the eruption

A 2 y/o presents with left arm pain onset after the mother picked the child up by the arm after she fell. The child is holding her arm in the pronated position and refuses to use her arm. Which of the following is the most appropriate management of this child?

Supinate distal arm with elbow flexion Subluxation of head of radius from the annular ligament Mechanism: Caused by traction along the long axis of the arm Child was pulled by the hand and now will not move the elbow Child won't use affected arm and has tenderness over radial head Tests: X-rays are normal Treatment: Reduction by flexion and supination of the forearm Child will immediately use arm

Which of the following measures would be the most appropriate RX of the patient above (Sinus Tachy)?

Supplemental oxygenation or mechanical ventilation

A woman presents with a non-tender lesion on her tongue she noticed a few days ago. It is gray in color, raised, and oval-shaped. 3 other patches are noted in her oral cavity. She had a positive RPR "several years ago." What is the most likely diagnosis?

Syphilitic mucous patch

What is the hallmark finding that is essential to the diagnosis of primary dilated cardiomyopathy?

Systolic dysfunction Systolic dysfunction and left ventricular dilation are essential for the diagnosis of primary dilated cardiomyopathy.

Which of the following situations in the periinfarction period would suggest the presence of ventricular septal perforation

Systolic murmur, large v waves in pulmonary capillary wedge tracing; Po2 in right atrium is less than that in right ventricle

A 72 year-old farmer comes to the office for evaluation of a pearly ulcerated papule on his right nostril. The papule has been bleeding off and on for the past couple weeks. Which of the following findings would be most concerning on the physical examination of this patient?

Telangiectasia Telangiectatic vessels are often visible skin findings with basal cell carcinoma

A man is stabbed and arrives to the emergency room within 30 minutes. You notice that the trachea is deviated away from the side of the chest with the puncture. CXR is seen here Which of the following is the most likely diagnosis?

Tension pneumothorax

A 16 year-old female who is a competitive swimmer presents with a complaint that her toe nails are thickened and discolored. Examination reveals that the nails have a dry yellow brittle appearance along with a friable surface of the nail plate. Which of the following is the treatment of choice?

Terbinafine (Sporonox) Oral Terbinafine 250 mg is considered the best treatment for onychomycosis

A 4-week-old infant presents with cyanosis. Workup of the patient reveals a VSD , overriding aorta, PS & RYH. Which of the following is the most likely diagnosis?

Tetralogy of Fallot Tetrology of Fallot is a cyanotic congenital heart disease with four structural defects (listed in the question). Coarctation of the aorta and patent ductus arteriosus are non-cyanotic congenital heart diseases.

A 23 yr old G2P1 woman at 39 wk estimated gestational age presents to L&D triage stating "my water has broken". She reports a large gush of clear fluid followed by a constant leaking of fluid from her vagina. Uterine contractions are occurring approx. every 4 minutes. Her prenatal course has been uncomplicated with good prenatal care since 8 wks gestation. Review of her previous prenatal records show her first pregnancy resulted in an uncomplicated vaginal delivery of a 6 lb 14 oz healthy baby. She is admitted and placed on a fetal monitor. BP 110/70; P 90 reg; T 98.7F. PE - gravid abdomen, fundal height of 38 cm. Fetus has a cephalic presentation by Leopold maneuvers and an estimated fetal weight of 8 lbs. Cervix is 2 cm dilated. What signs and tests could confirm the presence of rupture of membranes? Baseline fetal heart rate? How do you treat positive group B stretococcus (GBS) vaginal culture at 36 wks gestation? What stage of labor is this? What is causing the decelerations in the fetal heart tracing?

Tests: 1. Visualization of amniotic fluid leaking from the cervix 2. Presence of pooling of amniotic fluid in the posterior vaginal fornix 3. pH of fluid collected > 6.5 using Nitrazine paper 4. Visualization of "ferning" on a sample of fluid on an air-dried microscope slide Fetal heart tones: 135 BPM w/ moderate variability and no accelerations or decelerations (from rhythm strip) + group B streptococcus (GBS) Tx: 1. Penicillin 5 million units IV loading dose followed by 2.5 million units IV every 4 hrs Other agents: IV ampicillin, cefazolin, clindamycin, & vancomycin Fetal infection with GBS can result in neonatal morbidity and mortality Labor: First stage, latent phase Stage 1 - from the onset of labor until the cervix is completely dilated Latent phase - contractions are becoming stronger, longer lasting & more coordinated Active phase - starts at 3-4 cm of cervical dilation Fetal heart deceleration: Compression of the fetal head Variable decelerations - caused by cord compression Late decelerations - signs of uteroplacental insufficiency

A 23 year-old female complains of fever and genital pain. Examination reveals the presence of lymphadenopathy in the groin and the presence of vulvar vesicles surrounded by erythematous skin. The diagnosis may be confirmed by

The presence of giant multinucleated cells on Tzanck smear. A Tzanck smear is the standard laboratory test to confirm the herpes virus as an etiologic agent of a vesicular lesion on an erythematous base

Which of the following is the drug of choice for the prevention of Wernicke's encephalopathy?

Thiamine Wernicke's encephalopathy, also called alcoholic encephalopathy, is an acute neurological disorder of thiamine deficiency. Patients are given 100mg po BID to TID for 1 to 2 weeks or parenterally 100mg in each liter of glucose.

Which of the following is the single most important prognostic factor in regard to melanoma?

Thickness of the lesion The single most significant prognostic factor in melanoma is tumor thickness. Ten year survival rate in years according to thickness : < 1mm, 95%; 1-2mm, 80%; 2-4 mm, 55%; >4mm, 30%.

A patient presents with a burn of the left upper extremity. The skin of the affected area appears pearly white without blistering and the patient is unable to feel any sensations. What is the correct classification of this patient's burn?

Third degree Explanation: First-degree-red, moist, swollen, and painful and blanches when lightly touched but does not develop blisters Second-degree-red, swollen, and painful, and develops blisters Third-degree-usually are not painful because the nerves have been destroyed. The skin becomes leathery and may be pearly white, black, or bright red. The burned area does not blanch, and hairs can easily be pulled from their roots without pain. No blisters develop

During a mental status examination of a patient with schizophrenia, you note increased time between the patient's comprehension and expression of thoughts. This finding is an increase in which of the following?

Thought latency Increased thought latency is otherwise known as blocking. It is common in patients with schizophrenia, and defined by a period of time between a thought and its verbal expression in increased.

A patient is diagnosed with hypothyroidism and started on levothyroxine (Synthroid). When is it ideal to recheck the TSH level?

Three to four weeks Patients taking levothyroxine (Synthroid) for thyroid replacement will achieve peak levels of T4 within three to four weeks.

An 8 year-old child is brought in by his mother with a two day history of spreading, non-pruritic red rash. The rash was preceded by moderate fever, sore throat and rhinorrhea. Examination reveals a moderately ill appearing child with a fine, macular-papular rash on an erythematous base spread diffusely over the trunk with some accentuation in the skin folds. The face is flush with perioral pallor. There is palpable anterior cervical lymphadenopathy. Which of the following is the most appropriate diagnostic study to establish the diagnosis?

Throat culture Scarlatina rash is due to infection with group A strep. A throat culture would be the most appropriate diagnostic study to establish the diagnosis in this patient

An afebrile 22 year-old female presents with 2 days of a painful rectal mass which worsens with defecation. She denies any blood with bowel movements. She admits to ongoing constipation and generally poor diet of fast food. She is otherwise healthy. On examination there is a tender, swollen, bluish, ovoid mass visible at the anal margin. Which of the following is the most likely diagnosis?

Thrombosed hemorrhoid A thrombosed hemorrhoid causes acute pain increased with defecation and sitting.

A 26-year-old man from Cape Cod sees his physician because of a 3-week history of an expanding, slightly burning ring of redness that first surrounded a red papule on the posterior neck. He complains of headaches, generalized muscle aches, anorexia, and malaise. On examination, he is noted to be febrile [38.3C (101F)]; his rash is slightly raised and slightly tender and displays central clearing but no scaling, even after vigorous scraping. Which of the following vectors has been strongly associated with the type of rash described above

Tick An expanding erythematous rash not associated with scaling is characteristic of erythema chronicum migrans. The disease first appears weeks to months after a a bite by tick infected with Borrelia burgdorferi . The lesion begins as a red macule at the site of the bite; the borders of the lesion then expand to form a red ring, with central clearing, as wide as 20 to 30 cm or more in diameter. Occasionally, secondary rings may occur within the original one. The lesion may itch or burn and may be accompanied by fever, headache, vomiting, fatigue, and regional adenopathy.

A 6-year-old child presents complaining of patchy hair loss on the back of the scalp. Examination reveals well-demarcated areas of erythema and scaling, and although there are still some hairs in the area, they are extremely short and broken in appearance. Which of the following is the most likely diagnosis?

Tinea capitis Explanation : broken hair shafts are seen as black dots Kerion (indurated, boggy inflammatory plaque studded with pustules) can be present in any location of tinea, but most commonly seen on the scalp

A 4-year-old boy has been seen in your office in the past year for 6 episodes of tonsillitis in the last year. Tonsils have been moderately swollen and inflamed, with exudate on 3 occasions. Four of the six episodes have had positive throat cultures for Group A Streptococcus; cultures were negative for the other 2 episodes. She does describe intermittent nocturnal snoring occurring nightly, but not throughout the night. Every so often, she has heard him make a loud sound with a pause afterward when it sounds as if he is not breathing. She has gone to check him and found him to be okay when she arrives in his bedroom. The mother is concerned about his frequent infections. On PE his tonsils are grade 3 with no pus, no difficulty with swallowing or talking. In advising her regarding the next step, the most appropriate advice is that:

Tonsillectomy, possible adenoidectomy

An adolescent with mild acne is currently on a treatment regimen utilizing topical keratolytics, but is only having some mild improvement. Which of the following is most appropriate to add to the treatment regimen next?

Topical antibiotics Topical antibiotics are the most appropriate agents to be added to topical keratolytics in treatment of mild acne

In acute angle closure glaucoma, aqueous humor is prevented from reaching which structure?

Trabecular meshwork

A 38-year-old gay male who is infected with the HIV virus presents with a one-week history of fever and tachypnea. Chest x-ray reveals bilateral alveolar infiltrates. Arterial blood gas determination reveals a PaO2 of 55 mm Hg on room air. Bronchoalveolar lavage is positive for methenamine silver staining material. Which of the following statements is correct concerning the current clinical situation?

Trimethoprim-sulfamethoxazole (Bactrim) alone should be administered

A 76 year-old male presents after returning from a Safari in Africa. Seven days ago he experienced CP lasting one hour that did not respond to three sublingual nitroglycerin tablets. There was no ability to have lab work or an EKG. The pain has not returned. If the patient had a non-STEMI myocardial infarction, which of the following studies will still be positive?

Troponin I Troponin I levels will stay positive for at least one week following myocardial infarction and is the preferred enzyme to measure in this setting

Which of the following is the only disease that forms an ulcer at the site of inoculation?

Tularemia Tularemia is an acute infection that is transmitted by handling the flesh of infected animals, by the bites of insect vectors and by inoculation of conjunctiva.

A 19-year-old girl attending school in Massachusetts presents with the chief complaint of bloody diarrhea for 2 months. She has abdominal discomfort and feels she has lost some weight. She also complains of tenesmus. Abdominal examination is normal. The rectal exam reveals stool containing blood and pus. Which of the following is the most likely diagnosis?

Ulcerative colitis Most commonly involves the rectosigmoid colon Involves the mucosa and submucosa only. Diffuse involvement (no skip lesions). Intermittent bloody diarrhea (lesions are friable). Tenesmus, Urgency. Lower abdominal cramping. Almost always involves the Rectum. (+) p-ANCA (anti- neutrophil cytoplasmic antibody). Biopsy - crypt abscesses, crypt branching, shortening and disarray, and crypt atrophy

A 20 year-old male presents with a painful testicle. A testicular mass is palpated on the physical exam. Which of the following is the initial diagnostic evaluation to pursue?

Ultrasound of the testicles

Which of the following is the study of choice to diagnose upper gastrointestinal malignancy?

Upper endoscopy Upper endoscopy is the study of choice to diagnose gastroduodenal ulcers, erosive esophagitis and upper gastrointestinal malignancy.

A patient who was recently diagnosed with viral pericarditis now complains of severe dyspnea and non-productive cough with pain over the precordial region. His VS reveal a HR 130 and respiratory rate 26. BP is 130/105 but fluctuates with inspiration resulting in a 20 mmHg decline in the systolic pressure. Which of the following is the most appropriate therapy for this patient?

Urgent pericardiocentesis This patient has signs and symptoms consistent with pericardial tamponade. Urgent pericardiocentesis is required

Which of the following interventions is most effective during the early stages of Alzheimer's disease?

Utilization of memory aids, such as post-it notes Memory aids are extremely helpful in assisting Alzheimer patients during the early stages of the disease.

A 70 year-old male is admitted to the ICU with fever, leukocytosis and purulent sputum. Sputum culture shows methicillin-resistant gram-positive cocci in clusters. Which of the following medications should be ordered for this patient?

Vancomycin (Vancocin) Vancomycin is the drug of choice for methicillin-resistant S. aureus.

Adult patients with a compromised immune system should not receive which of the following immunizations?

Varicella Varicella vaccine is a live vaccine and is contraindicated in those individuals with a compromised immune system. Hepatitis A, pneumococcal and influenza are recommended for patients with a compromised immune

A 50-year-old chronic alcoholic has bruising, inflamed and bleeding gums, and painful joints. The platelet count and prothrombin time are normal. The most likely cause for this clinical scenario is deficiency of which of the following vitamins?

Vitamin C

A 61-year-old woman noticed severe sharp pain in her back after lifting a suitcase. A compression fracture of the T11 vertebral body is identified on x-ray examination. Routine laboratory evaluation discloses a serum calcium concentration of 8.0 mg/dL, a serum phosphorus concentration of 2.4 mg/dL, and increased serum alkaline phosphatase activity. The serum parathyroid hormone level was subsequently found to be elevated as well. The most likely diagnosis is

Vitamin D deficiency

Which of the following is an acquired loss of skin pigmentation characterized by depigmented patches surrounded by a normal or hyperpigmented border?

Vitiligo Common Distribution Perioral, trunk, extensor surfaces of extremities, flexor wrists, axillae Occurs at sites of trauma (Koebner's phenomenon). Wood's lamp examination helps identify hypopigmented areas in patients with light complexions Usual Morphology Appears as irregular areas of hypopigmentation of the skin, as shown here on the hand. Many cases are idiopathic, though sometimes a systemic disease may be present. Chalk-white macules

A patient with conductive hearing loss in the left ear would have which of the following physical findings?

Weber lateralizes to the left ear Also correct: BC> AC left ear

Low molecular weight heparin (LMWH) dosage is based on which of the following?

Weight LMWH is based on a patient's weight in kilograms

A 50 year old man with a history of severe PUD presents for evaluation. He has been treated with high dose proton pump inhibitors (Nexium) for 4 weeks with no relief of symptoms. He complains that he also has been having 5-6 loose, greasy bowel movements per day for several weeks. He admits to a family history of similar problems. What is the most likely diagnosis?

Zollinger-Ellison syndrome This is characteristic of the Zollinger-Ellison syndrome which is a syndrome characterized by marked hypergastrinemia, gastric hypersecretion, and peptic ulceration, as well as an associated gastrin-producing tumor in the pancreas. A condition affecting about 1% of patients with peptic ulcer; 90% have ulcer; 35% diarrhea. Very high basal acid production. Tumor location is typically pancreatic. About half are malignant when first found

Which of the following is the most common cause of intrinsic acute renal failure?

acute tubular necrosis

Rates of alcohol use in the adolescent are reportedly higher in:

adoptive children whose biological parent is an alcoholic There is a three-to fourfold increase in risk for alcohol dependence in adopted children whose biological parents are alcohol dependents.

A 15-y/o girl is brought to the clinic by her mother who is concerned about her daughter's recent significant weight loss. The patient appears thin and cachectic although she actively participates in various sports and has started to jog 5 to 6 miles per day. He states that she just doesn't feel hungry and is satisfied with her current weight. What other findings might you expect?

amenorrhea Dx: Anorexia Nervosa

Which of the following is the most helpful serological test in primary biliary cirrhosis?

anti-mitochondrial antibodies Anti-mitochondrial antibodies are seen in over 90% of cases of primary biliary cirrhosis and are rare in other forms of liver disease.

A 37 year-old female with history of Turner's syndrome and coarctation of the aorta repaired at the age of 3 presents for routine examination. The patient is without complaints of chest pain, dyspnea, palpitations, or syncope. On PE, VS reveal a BP of 130/76, HR 70, regular, RR 16. On cardiac examination you note a grade II/VI SEM at the LSB and a grade III/VI blowing diastolic murmur. Which of the following does this patient require?

antibiotic prophylaxis This patient has a history of congenital heart disease and presently has a murmur consistent with aortic regurgitation. This patient requires antibiotic prophylaxis against infective endocarditis.

The hepatorenal syndrome is associated with

ascites, liver failure, renal failure, low urine sodium

The most important initial component of evaluating a patient with depressive illness is

assessment of suicidal risk. Two-thirds of all depressed patients contemplate suicide, and 10 to 15% commit suicide.

A 13 year-old male with sickle cell trait is interested in playing football for his school. Which of the following considerations should be reviewed with the patient and his parents?

avoid dehydration No restriction of sports activity is required of patients with sickle cell trait. Hydration is required in all sports activities

A 5 year-old is being evaluated for autism. Of the following, the finding most consistent with this diagnosis is

avoids eye contact Children with autism do not tend to make eye contact, and even avoid it

Which 2 forms of shock frequently occur together during a acute MI?

cardiac shock and hypovolemic shock Shock in the presence of an acute MI can be of two pathologic varieties. Either there is hypovolemia and associated hypotension (hypovolemic shock), or there is persistent hypotension and a poor cardiac index in the presence of adequate left ventricular filling pressures. Hypovolemic shock is best treated with volume replacement using either Ringer's lactate or normal saline solution. Care must be taken to avoid "volume overloading" which may produce pulmonary edema and congestive cardiac failure. Cardiogenic shock is generally associated with severe left ventricular dysfunction and occurs with large infarcts that damage more than 40% of the left ventricle. Treatment is urgent, and mortality is high.

Which of the following blood gas combinations occurs most commonly in a patient with a pulmonary embolism?

decreased PO2 and decreased pCO2

A patient with a history of anemia is receiving a unit of packed red blood cells. After transfusion of only a few cc's of blood, the patient develops shortness of breath, wheezing, and hypotension. After stopping the transfusion, the physician assistant should administer

epinephrine This patient is most likely having an anaphylactic reaction to the blood and should be given epinephrine

A 35 year-old female presents with fatigue. CBC results reveal the following: WBC: 6,300/microliter Hgb: 9.5 g/dl Hct: 28% MCV: 75 fL MCHC: 32 g/dl MCH: 24 pg Platelets: 550,000/mL Which of the following is the best treatment option for this patient?

ferrous sulfate Iron deficiency anemia is a microcytic, hypochromic anemia and is treated with ferrous sulfate.

What type of pharmacological agent would be used as a first-line medication to treat obsessive compulsive disorder

fluoxetine Treatment: Psychotherapy and group or behavioral therapy. Clomipramine, clonazepam and SSRIs may help if obsessive-compulsive sxs occur

A 65-year-old woman with long-standing diabetes presents with severe pain in the left ankle and foot. She gives a history of twisting the ankle about 2 months earlier. Physical examination reveals decreased sensation to pinprick in both legs from the knees downward and swelling of the left foot and ankle with an outward bulge in the sole. There is no evidence of vascular compromise. Radiographs show osteophytes as well as tapering of the distal metatarsal bones. Aspiration of the ankle joint reveals a small amount of sterile xanthrochromic fluid without crystals. The most appropriate treatment for this problem is

foot brace Neuropathic Joint Disease or Charcot's Foot Ankle joints are most commonly affected; patients often give a history of some initiating trauma. The diagnosis is made clinically and after oteomyelitis, osteoarthritis, and calcium pyrophosphate deposition disease are excluded. Treatment includes no weight-bearing; stabilization of the foot, usually with a brace (followed carefully due to the neuropathy); and sometimes surgical joint fusion.

Which of the following is the triad of clinical findings characteristic of normal pressure hydrocephalus?

gait disturbance, impaired mental function, incontinence Clinical triad: dementia, gait instability, and urinary incontinence with deterioration over 6 to 12 months

An 18-year-old patient presents with fever, pharyngitis, and cervical lymphadenopathy. The CBC shows a leukocytosis with 25% atypical lymphocytes. Which of the following is the diagnostic test of choice for this patient

heterophile test A heterophile test is the test of choice for the diagnosis of infectious mononucleosis

Diagnostic Studies/Hematology The physician assistant is evaluating a patient suspected of having an iron deficiency anemia. When examining the patient's peripheral blood smear, the diagnosis is reinforced by noting

hypochromic, microcytic red blood cells. Hypochromic, microcytic red blood cells are common in patients with iron deficiency anemia

An otherwise-healthy 75-year-old man who underwent a left total hip replacement 6 months ago develops slowly progressive left hip pain. He feels relatively well. Physical examination is unremarkable except for pain on internal rotation of the left hip. Routine laboratory studies are unremarkable except for an elevated ESR. Radiographs reveal loosening of the hip prosthesis. The next most appropriate action is

joint aspiration If an infection in a prosthetic joint is documented after joint aspiration, cure usually requires a combination of organism-specific antibiotic plus joint replacement. There is, however, an emerging literature documenting that a nonsurgical cure of staphylococcal infections may be possible with a prolonged course of rifampin plus ciprofloxacin, each given orally. Prevention of prosthetic joint infections via perioperative antibiotic prophylaxis, laminar intraoperative airflow, and aggressive treatment of extraarticular infections in the postoperative period is the best approach. Infectious complications of joint replacement surgery occur in 1 to 4% of patients and include acute infections with Staphylococcus aureus, streptococci, or enteric gram-negative rod bacteria as well as more indolent infections presenting much later (and in a more subtle fashion) caused by coagulase-negative streptococci or diptheroids.

A 52 year-old male, who underwent transphenoidal surgery and radiation for acromegaly 15 years ago, presents with the following exam findings: weight gain of 20 lbs, puffy eyes, coarse dry skin and normal visual fields. Laboratory findings reveal the following: TSH 0.3 µu/ml (normal 0.5-5.0), free T4 0.3 ng/ml (normal 0.7-2.7). Which of the following drugs would be indicated for this patient?

levothyroxine (Synthroid) The patient is suffering from hypothyroidism as indicated by the low free T4. TSH is not elevated in this case secondary to the previous history of transphenoidal surgery and radiation for his acromegaly

A 26 year-old male presents with a four week history of fatigue, night sweats, and a painless mass in his neck. Physical examination confirms the presence of an enlarged right posterior cervical lymph node. What is the next best step in the evaluation of this patient?

lymph node biopsy This patient presents with possible lymphoma. Diagnosis is made by lymph node biopsy. Bone marrow biopsy and CT scan of the chest are used for staging of the disease

A 5 year-old boy presents to the office for follow-up of complaints of pruritus in the perianal area. A cellophane tape test is positive. Which of the following is the treatment of choice?

mebendazole (Vermox) Treatment of choice for enterobiasis (pinworms) is mebendazole.

Which of the following laboratory assays demonstrates excessive amounts of catecholamine degradation products in the urine?

metanephrine and normetanephrine Metanephrine and normetanephrine are the degradation products that result from the metabolism of epinephrine and norepinephrine.

A patient is experiencing bitemporal hemianopsia. Where would you expect the lesion to be?

optic chiasm

A 14 year-old male presents to the ED experiencing a severe asthma attack. His respiratory effort is shallow and he is using accessory muscles to breathe. Auscultation of his chest reveals no audible wheezing. Vital signs include BP 90/60 mmHg, P 160 bpm, RR 52. An arterial blood gas (ABG) is ordered. Normal ABG values at your institution are pH 7.35-7.45, CO2 35-45, pO2 80-95. Which of the following ABG findings suggests the poorest prognosis? A. pH = 7.27 pCO2 = 46 pO2 = 56 B. pH = 7.60, pCO2 = 18 pO2 = 80 C. pH = 7.44, pCO2 = 38 pO2 = 90 D. pH = 7.52, pCO2 = 28, pO2 = 80

pH = 7.27 pCO2 = 46 pO2 = 56 is associated with the poorest prognosis in this patient.

A 58 year-old male presents with CP . Vital signs include BP of 210/175, pulse 80, RR 20. Which of the following would you expect to find on physical examination?

papilledema Malignant hypertension is characterized by marked blood pressure elevation with papilledema, often with encephalopathy or nephropathy.

The most important regulator of serum 1,25(OH)2 vitamin D concentration is

parathyroid hormone

Absorption of vitamin B12 requires intrinsic factor that is secreted by which of the following?

parietal cells Parietal cells are responsible for acid secretion and intrinsic factor.

A two-week-old female is being evaluated in the clinic, and on examination she is noted to have bounding pulses with a widened pulse pressure. There is a rough, machinery sounding murmur present at the second left intercostal space. Cyanosis is not present. Which of the following is the most likely diagnosis?

patent ductus arteriosus(PDA) Patent ductus arteriosus is characterized by a classic harsh, machinery-like murmur that is continuous through systole and diastole. This is heard best at the left second interspace and is commonly associated with a thrill. A large PDA may present with signs of heart failure, including failure to thrive, poor feeding, and respiratory distress. The older child may present with shortness of breath or easy fatigability. In a preterm (before 37 weeks) it can be closed medically using indomethacin (prostaglandin inhibitor). In a term baby if still open at 3 months of age then coil closure or devise by cardiac catherization is the method of choice.

A 56 year-old man is admitted to a hospital unit for evaluation of rectal bleeding and weight loss. He has a strong family history of cancer. Soon after admission, a barium enema is scheduled. The patient refuses the "prep" because he fears x-ray radiation. He states he has had previous x-rays, but becomes frightened at the thought of an x-ray and "can't face it." The most likely diagnosis is:

phobic neurosis. Phobic neurosis is a phobic ideation of displacement where the patient transfers feelings of anxiety from the object to one that can be avoided.

Which of the following is an absolute contraindication to thrombolytic therapy in a patient with an acute STEMI ?

previous hemorrhagic stroke Absolute contraindications to thrombolytic therapy include a previous hemorrhagic stroke, a stroke within one year, a known intracranial neoplasm, active internal bleeding, and a suspected aortic dissection. Severe, but controlled HTN , use of anticoagulation, and active PUD are relative contraindications in which the risk/benefit ratio must be weighed in each patient

A 22-year-old female, married for 18 months, has been trying to get pregnant without success. Approximately 9 months ago, she developed breast secretions, amenorrhea, and decreased libido. No other symptoms are present. On PE, there is definite galactorrhea present. Her blood pressure is 140/80 mm Hg. No abnormalities are found on physical examination. From the information provided, what is the most likely diagnosis?

prolactinoma This patient has hyperprolactinemia, most likely the result if a pituitary adenoma Combination of galactorrhea, amenorrhea, infertility, and decreased libido is caused by a prolactinoma

A 58 year-old female with a history of controlled hypertension is brought to the office after becoming incoherent during lunch 30 minutes ago. Examination reveals a right facial droop and right extremity weakness. A noncontrast head CT is negative for acute hemorrhage and an electrocardiogram reveals normal sinus rhythm. Which of the following is the most appropriate next step in the management of this patient?

rtPA bolus and IV infusion This patient meets the qualifications for rtPA according to the NIH stroke scale. While Heparin is a good choice in the acute setting, the most effective treatment choice would be thrombolytics.

A child has been under treatment for attention-deficit hyperactivity disorder (ADHD). No response has occurred with behavioral adaptations. Which of the following categories of medication should this patient be given?

stimulants Stimulants, such as Ritalin, Dexedrine, and Cylert, are effective in 50 to 80% of children with ADHD.

There are many sources of potential emboli that may cause a CVA. The most common source of cerebral emboli is:

the heart Arterial embolism/thrombosis: Cause of acute arterial insufficiency secondary to Afib, MS, hypercoagulable states, and trauma. Learn the 5 P's: Pain, Pallor, Parasthesia, pulselessness, and paralysis. Echo, arteriogram, Heparin, surgery.

Which of the following is the most common cause for acute myocardial infarction?

thrombus development at a site of vascular injury AMI occurs when a coronary artery thrombus develops rapidly at a site of vascular injury. This injury is produced or facilitated by factors such as cigarette smoking, hypertension, and lipid accumulation. In most cases, infarction occurs when an atherosclerotic plaque fissures, ruptures, or ulcerates and when conditions (local or systemic) favor thrombogenesis, so that a mural thrombus forms at the site of rupture and leads to coronary artery occlusion.

A 55 year-old morbidly obese male is seen in the office for routine examination. He has a history of PHTN and cor pulmonale. Examination reveals a visible JVP and a systolic flow murmur on the right side of the sternum. Which of the following is the most likely diagnosis?

tricuspid insufficiency Tricuspid insufficiency will result in blood being put back into the right side of the body with increased jugular pulsation in the neck, along with a palpable venous pulse in the liver.

The causative organism for molluscum contagiosum is which of the following?

virus Molluscum contagiosum is caused by a poxvirus

Which of the following statements about malignant melanoma's is true?

"Change in color of the lesion is a suspicious sign for potential malignancy" The characteristics that distinguish superficial spreading malignant melanoma from a normal mole include irregularity of its border and variegation of color. Instead of the homogeneous color and regular borders of a "normal" mole, the lesion shows disorderliness and irregularity. The first changes noted by persons who develop melanoma in a preexisting mole are a "darkening" in color or a change in the borders of the lesion. Irregularity of the borders in an expanding, darkening mole is melanoma until proved otherwise; excisional (not incisional) biopsy should be done promptly because early diagnosis and excision reduce the mortality. It is best not to cut into a lesion for which melanoma is in the differential diagnosis.

A 40 year-old patient with a history of recurrent kidney stones presents with acute onset of right flank pain and hematuria. The patient is afebrile and pain is poorly controlled on oral medications. On CT scan a 1 cm stone is noted in the renal pelvis. Which of the following is the most appropriate intervention for this patient? A. Antibiotics B. Shock wave lithotripsy C. Ureterolithotomy D. Fluid hydration

**Ureterolithotomy Shock wave lithotripsy?

What is the recommended target LDL to reduce the risk of coronary artery disease in a diabetic patient?

100 mg/dL The National Cholesterol Education Program clinical practice guidelines have designated diabetes as a coronary risk equivalent and have recommended that patients with diabetes should have an LDL cholesterol goal of 100 mg/dL.

As part of the long-term management of a patient with type 1 diabetes mellitus, the glycosylated hemoglobin (HgbA1C) level should be ideally maintained at

6 to 7%. The HgbA1C in patients with type 1 diabetes mellitus should be maintained between 6 and 7%. Lower levels lead to an increased number of episodes of hypoglycemia, and higher levels lead to an increased risk of retinopathy, nephropathy, and neuropathy

In HIV-infected females prophylaxis for mycobacterium avium complex should be initiated when the CD4 count is less than what value?

75 cells/microL Treatment for MAC should begin when the CD4 count falls below 75 cells/micro

Select the incorrect statement regarding Pneumocystis pneumonia (PCP): A. This is a cause of community acquired pneumonia in normal immune competent hosts B. One of the common presentations of HIV disease C. First line treatment is Bactrim D. Bactrim allergic patients may be treated with Dapsone, Atovaquone, and Pentamine E. Prednisone is indicated if PaO2 on room air is less than 70

A. This is a cause of community acquired pneumonia in normal immune competent hosts

Which of the following is an example of a hypersensitivity reaction in which complement is activated by antibodies reacting with antigens leading to cell damage or death?

ABO transfusion reaction An ABO transfusion reaction is a type II hypersensitivity reaction in which complement is activated by the interaction of antibodies and antigen on the red blood cell surface

A 44-year-old man with type 2 DM is noted to have BP readings of 145/90 and 150/96 mm Hg on two separate occasions. Which of the following is the best initial therapy for this patient?

ACE inhibitor For diabetics in general, the antihypertensive agent of choice is the ACE inhibitor. If the BP is uncontrolled, then a thiazide diuretic may be added. An ACE inhibitor provides a survival advantage to diabetics with hypertension

A 60-year-old male has diabetic retinopathy, proteinuria, hypertension, a serum creatinine of 1.5 mg/dL, and CHF. What is the preferred antihypertensive?

ACE inhibitors

A 25-year old previously healthy smoker comes to you complaining of a sore throat and painful, productive cough during 7 days. Exam reveals a temperature of 101.2 F with some rhonchi and expiratory wheezes on auscultation of the both lungs. The most likely diagnosis is

Acute bronchitis Inflammation of the large airways of the tracheobronchial tree.

A mother brings her 4-year-old daughter into the walk-in clinic. The child has had a cough with hoarseness and low grade fever. On physical exam the patient is tachypneic at 34, she is not posturing or retracting. HEENT is unremarkable, inspiratory stridor is audible on auscultation of the lungs. During the exam, you note a harsh barking cough. The following x-ray is taken The most likely diagnosis is:

Acute Laryngotracheobronchitis (Croup)

A 22-year-old man is brought to the emergency room after being found unconscious in a swimming pool and was intubated and then resuscitated by paramedics using advanced cardiac life support measures. The patient is mildly cyanotic. The blood pressure is 80/50 mm Hg, heart rate is 120/min, and respiratory rate is 26/min. his core body temperature is 89 F. Pupils are 4 mm bilaterally and reactive. The patient is moving all extremities and responds appropriately to questions. Crackles are heard bilaterally on lung auscultation.. ABG reveals a pO2 of 50 mm Hg on 100% O2. CXR reveals bilateral "white-out" of the lungs consistent with interstitial and alveolar infiltrates. There is no cardiomegaly. Which of the following is the most likely diagnosis?

ARDS

A 37-y/o female with a new onset of AF is sent to the ED by her primary care provider. She complains of a recent onset of fatigue and DOE which has now progressed to dyspnea at rest. The patient also admits to associated orthopnea and peripheral edema. On cardiac exam, prominent RV & PA pulsations are visible and palpable. S2 is widely split and does not vary with breathing & a loud SEM is heard in 2nd & 3rd interspaces parasternally. The patient states she was told that she had a murmur when she was a child, but she does not recall what it was or if she had any other testing. Which of the following is the most likely diagnosis?

ASD The patient in the above scenario is in CHF , mostly likely due to an ASD. An ASD is an opening in the atrial septum that allows for the passage of blood (shunting) between the atria.

A 25-year old previously healthy smoker comes to you complaining of a sore throat and painful, productive cough during 7 days. Exam reveals a temperature of 101.2 F with some rhonchi and expiratory wheezes on auscultation of the both lungs. The most likely diagnosis is

Acute bronchitis

A 29-year-old female has a long history of SVT , for which she has been treated with long-term flecanide, as well as prior therapy with verapamil. She continues to have repeated episodes, sometimes two to three times a week, along with SOB and at times hypotension that has been recorded. What is the next best therapy for this patient?

Ablation therapy After exhaustion of non-invasive therapies, ablation therapy can be used to try to negate the aberrant pathway for SVT. Pacemakers will not allow for an override of the pathway, and cardioversion is only a temporary solution to an acute event. Implantable telemetry monitoring is only diagnostic and not therapeutic to treat.

A 10 year-old male has abrupt, brief episodes of impaired consciousness and rhythmic blinking. Which of the following is the most likely type of seizure?

Absence Absence seizures are generalized seizures characterized by abrupt, brief episodes of impaired consciousness, staring, and rhythmic blinking.

The condition manifesting as velvety, hyperpigmented, papillomatous lesions of the neck, axillae and groin, associated with insulin resistance, is known as:

Acanthosis nigricans Explanation: Hyperpigmentation with thick, velvety accentuation of the dermal lines Most commonly on the axillae, neck but can also be groin, anogenitalia, antecubital fossae, knuckles, submammary, and umbilicus Laboratory: R/O Diabetes Mellitus, Carcinoma of clinically suspected Treatment: No proven treatment other that for underlying disorder if present

A 45 year-old male nonsmoker presents with a 3-month history of difficulty swallowing. He describes the gradual onset of dysphagia for solids and liquids; he now complains of substantial discomfort after eating. What is his most likely diagnosis?

Achalasia Achalasia is by far the most common cause of dysmotility. This is particularly true in a non-smoker (less likely to be esophageal carcinoma or a lung cancer causing a paraneoplastic syndrome) and someone who has not been to an area endemic for Chagas disease.

Which of the following results from hypersecretion of growth hormone in a 27 year-old patient?

Acromegaly This disease results from excessive growth hormone production generated by the anterior pituitary or an ectopic source

A patient in a Haitian orphanage is seen for a routine exam. His recent PPD test was negative. On further inspection he has extensive dental carries. The gram stain shows gram- positive branching filamentous bacteria. Sulfur granules are also present. What is the diagnosis?

Actinomycosis

Five days after going on a nature walk, a 13-year-old boy develops well-demarcated, erythematous plaques and vesicles over his arms and face. The plaques are arranged in a linear fashion and are crusting. The boy has some facial edema. He has no history of fever or chills but complains of pruritus. Which of the following is the most likely diagnosis?

Acute contact dermatitis

A 72 year-old man presents with acute left lower quadrant abdominal pain. He has nausea, vomiting, and constipation. He has a fever of 101° F and guarding and rebound tenderness in his left lower quadrant. His white blood cell count is elevated. He has no prior history of gastrointestinal disease. Which of the following is the most likely diagnosis?

Acute diverticulitis Acute abdominal pain, fever, left lower abdominal tenderness, and leukocytosis are hallmark signs of acute diverticulitis.

A 70-year-old man with a history of COPD presents complaining of worsening shortness of breath for the last seve`ral days. He is coughing large amounts of yellow-colored sputum and he is receiving no relief from his B-2-agonist and ipratropium aerosolized pumps. On physical examination, the patient's respiratory rate is 40/min and his heart rate is 110/min. His blood pressure is 150/85 mm Hg. The patient is afebrile. He is using his accessory muscles of respiration to assists in breathing. Lung examination reveals inspiratory and expiratory diffuse wheezing. CXR is seen here. Which of the following is the most likely diagnosis?

Acute exacerbation of COPD

A 48 year-old male complains of weakness and general malaise for about 2 months. Patient denies any recent illness and does not take any medications. Physical examination reveals a pale looking male in no acute distress. His heart rate is 110 beats/minute without a murmur and his abdominal examination reveals hepatosplenomegaly. A CBC reveals the WBC to be 62,000/microliter, Hgb is 8.3 gms/dl, Hct is 24.6%. A differential reveals a predominance of monoblasts and promyelocytes with Auer rods present. What is the most likely diagnosis?

Acute myelogenous leukemia Acute myelogenous leukemia (AML) is a leukemia that presents in adults with a cell lineage of the immature granulocytic cells seen in the peripheral blood e.g. myeloblasts, promyelocytes. Auer rods are commonly seen in this condition

A 65 year-old woman is admitted to the hospital for severe abdominal pain and vomiting over the last 6 hours. Her laboratory findings include: WBC Count - 19,000 Serum triglyceride level - 1100 mg/dL Glucose - 280 mg/dL Which of the following is the most likely diagnosis?

Acute pancreatitis This patient has acute pancreatitis. Typical symptoms include abdominal pain and nausea and vomiting. Significant hypertriglyceridemia is present. Transient hyperglycemia is common, in addition to elevation of lipase and amylase, serum amino-transferase and alkaline phosphatase are elevated in 50% of patients with acute pancreatitis.

A 24 y/o white male comes to the ER due to a sensation of tachycardia, slight dizziness, and vague chest fullness. BP 110/70. The following EKG was obtained (SVT). The initial pharmacological therapy of choice in the above stable patient is

Adenosine 6 mg rapid IV bolus Adenosine(6-12 mg IV) first line medical therapy terminates most SVTs With non-adenosine sensitive atrial tachycardia, adenosine will increase level of AV block, but atrial tachycardia will persist In presence of hemodynamic instability, electrical cardioversion treatment of choice Chronic Treatment: Radiofrequency Ablation(RFA) Success rate > 95% in eliminating or modifying AV nodal slow pathway function 1% risk of inducing CHB For patients with an accessory pathway, 85-95% successful elimination of accessory pathway function For atrial tachycardia,~70-80% success depending on site of tachycardia Type I or Type II antiarrythmic agents may also be used but currently rarely needed because of success of RF ablation

A 68 year-old woman presents with intermittent crampy abdominal pain and vomiting of 6 hours' duration. She feels bloated but denies rectal bleeding. Examination of the abdomen reveals no palpable mass and you note an upper midline scar from previous abdominal surgery. An abdominal radiograph demonstrates dilated loops of small bowel with air-fluid levels. Which of the following is the most likely cause of this patient's symptoms?

Adhesions The patient's surgical history, presenting symptoms and abdominal imaging are consistent with small bowel obstruction. Peritoneal adhesions are the most common cause of small bowel obstruction.

A 68 year-old male with a history of atrial fibrillation treated with warfarin (Coumadin) presents to the emergency department after vomiting large amounts of bright red blood. INR is 3. Which of the following is most appropriate to rapidly lower the patient's INR?

Administer fresh frozen plasma Fresh frozen plasma is the most rapid way to lower the patient's INR

A 25 year-old male presents to the emergency department for evaluation of a wound on his hand. He cut his hand while tearing down a chicken coop. On examination of his right hand you note a dirty 3cm jagged laceration. The patient is unaware of his tetanus immunization status. Besides cleaning and debriding the wound, what is the recommended clinical intervention in this patient?

Administer tetanus-diptheria toxoid (Td) and tetanus immune globulin (TIG) Td or Tdap with TIG is recommended as tetanus prophylaxis in a patient with a contaminated wound and unknown tetanus vaccination status.

A 50-year-old male is brought to your office by his wife. His wife states that for the past several months he has experienced extreme weakness and fatigue. In addition, he has gained 20 pounds in the past 3 weeks. During the last few weeks the patient has eaten nothing. When you question the patient, he states that his wife is overreacting. The patient's wife is extremely concerned about his alcohol intake. When you question the patient, he states that he is a social drinker. When you pursue this line of questioning further and ask him what he means by being a social drinker he states that he drinks a few drinks before and after most meals plus a few drinks before he goes to bed. When you ask him what he drinks, he says vodka. When you ask him if he drinks two 26-ounce bottles a day he says, "Heck, no! I would never put away more than a bottle a day." On physical examination, the patient has a significant enlarged abdominal girth. There is a level of shifting dullness present. The patient's liver edge is felt 6cm below the right costal margin. It has a nodular edge. In addition, spider nevi are present over the upper part of the patient's body. Palmar erythema is noted, and a flapping tremor is elicited. What is the most likely diagnosis?

Alcoholic cirrhosis

A 72 year-old man with long-standing diabetes mellitus, renal insufficiency and hypertension presents to the clinic complaining of burning and tingling pain in his feet. What agent would you prescribe to help control his pain?

Amitriptyline (Elavil) Amitriptyline is one of several preferred agents for management of peripheral neuropathy

Which of the following is the most appropriate initial and long-term therapy for this patient?

Amphotericin B IV for 14 days, followed by fluconazole 200 mg daily chronically Treatment of CNS Amphotericin B(0.5-1.0 mg/kg/day followed by fluconazole 400 mg PO QD for 10 weeks then 200 mg PO QD Treatment of Non Meningitis Fluconazole 400 mg QD x 6-8 mo For more severe disease may use Amphotericin 0.5-0.8 mg/kg/day until response

A 45 year-old female presents to the emergency department with generalized, hot, erythema of the skin. Physical exam reveals an oral temperature of 102 degrees Fahrenheit, purulent conjunctivitis, and mucosal erosions. Her skin is painful and separates from the dermis with touch. Which of the following is the most likely cause for this condition?

Ampicillin Medications are most frequently implicated in toxic epidermal necrolysis. These usually include, analgesics (NSAIDs), antibiotics (Ampicillin) and anticonvulsants (Carbamazepine).

A 35 year-old male presents with fever, perineal pain, and dysuria. On physical examination, the patient is toxic- appearing, febrile, and his prostate is very tender to palpation. Laboratory testing reveals leukocytosis, pyuria, and bacteriuria. He has an allergy to sulfa, which of the following is the initial treatment of choice for this patient?

Ampicillin and gentamicin

A 39-year-old man presents with progressive weakness of his arms and legs. He noticed difficulty in performing tasks such as buttoning up his shirt several months ago, and his symptoms have continued to worsen. On physical examination, cranial nerve and sensory findings are normal. Severe atrophy and fasciculation's are seen in the legs, arms, and tongue. The patient has a spastic muscle tone, hyperactive reflexes, and bilateral extensor plantar reflexes. Which of the following is the most likely diagnosis?

Amyotrophic lateral sclerosis(ALS) A progressive degenerative disease affecting upper and lower motor neurons (UMN and LMN). Currently incurable, eventually leading to death.

A 22 year-old male with a history of chronic low back pain presents with a four month history of worsening and persistent low back pain, fatigue, and morning stiffness which are relieved with exercise. He also reports intermittent decreased vision and photophobia in his right eye. A lumbar spine series reveals squaring of the vertebral bodies with normal disc space. Which of the following is the most likely diagnosis?

Ankylosing Spondylitits Ankylosing spondylitis (AS) is characterized by low back pain that persists for more than 3 months. It is often associated with morning stiffness relieved by exercise. Anterior uveitis affects up to 40% of patients with AS. Patients have diminished forward flexion and decreased chest expansion on inspiration. Radiographic evidence includes erosions in the sacroiliac (SI) joint line, pseudowidening, subchondral sclerosis, bony replacement of the SI joints, squaring of the vertebral bodies, and syndesmophytes (bridging of the vertebrae).

A 63-year-old woman presents to the urgent care facility with complaints of persistent heartburn. The patient has not had any success in alleviating symptoms with antacids or over-the-counter ranitidine. She feels that her heartburn spreads throughout her entire chest and upper abdomen. Vital signs are normal. Blood sample for tests has been obtained and sent for analysis, and reports are awaited. At the intake, ECG has been obtained. What is the most probable diagnosis?

Anterior myocardial infarction Prevent Extension ASA Heparin/LMWH Nitrates (dilate coronary arteries) Beta Blockers(< O2 demand) Remove obstruction PCI/PTCA Angioplasty Stent Thrombolytics(TPA) CABG; Anti-platelet agents

Which of the following will minimize the effect of barotrauma while diving?

Antihistamines Auto insuflation

A 23-year-old student presents to your office for health clearance to play collegiate sports. He is asymptomatic and exercises daily. On PE, his BP is 160/50 mm hg and his HR 60/min. there is pulsus bisferiens. Heart examination reveals a blowing diastolic murmur at the LSB. Nail beds reveal a Quincke pulse. Which of the following is the most likely diagnosis?

Aortic insufficiency Aortic Insufficiency: Usually remain asymptomatic for decades, often until fourth or fifth decade of life DOE, Orthopnea, PND Usually after significant cardiomegaly and LV dysfunction Angina Develops later, often during nighttime Palpitations/Head pounding Especially in supine position, pounding of heart against chest wall.

A 68 y/o WF presents to her PA with c/o "fainting spells off and on for a month or so". She is well known to your staff for her history of non-compliance with her antihypertensive medications. On PE you find bilateral carotid bruits, no JVD and clear lung fields. Cardiac exam is significant for a regular sustained apical impulse lateral to the midclavicular line and a 4/6 systolic cresendo-decresendo murmur at the base. The most diagnosis and cause of her symptoms is

Aortic valve stenosis Chronic left ventricular pressure overloading Eventually, left ventricular hypertrophy occurs and may cause: Diastolic dysfunction with the onset of heart failure symptoms Myocardial oxygen needs in excess of supply with the onset of angina. Exertional syncope due to the inability to increase cardiac output and maintain blood pressure in response to vasodilation. PE Findings Harsh late-peaking SEM (systolic ejection murmur) radiating to neck +/-Palpable systolic thrill Weak /slow rising carotid pulse (pulsus parvus et tardus) Sustained LV impulse A2 decreased S4 gallop if LVH present Narrow pulse pressure may be present (ie BP: 90/70)

A 33- year-old engineer with a recent history of prolonged analgesic use (Daypro, a NSAID) passes out at the gym. His wife states he had been feeling OK but had recently experienced some gingival bleeding while brushing his teeth. No past medical history is this previously healthy young man. On exam, he is slightly pale; otherwise the exam is completely normal. His blood counts are as follows: hemoglobin 5.2, hematocrit 15.6, MCV 90, platelets 20,000

Aplastic Anemia Aplastic Anemia Pancytopenia (lack of RBC's WBC'c, and platelets) resulting from bone marrow failure Causes Chemo, XRT, or drugs like phenytoin, chloramphenicol, sulfonamides Familial--Fanconi's

Which of the following is the standard treatment for this patient?

Arm sling or posterior splint Most common elbow fracture Fall on the outstretched arm typical presentation, inability to extend the elbow Look for a radial head fracture, or "fat pad sign" Even though a fracture may not be visible on the x-ray, soft tissue findings correlate well with a radial head fracture. Treat with a posterior splint, or simple sling.

A 50 year-old woman presents to the ER with severe RUQ abdominal pain radiating to her scapula that began shortly after eating at a local fast-food restaurant. Physical exam reveals a temperature of 101 °F, right upper quadrant tenderness with guarding. Your initial evaluation reveals probable acute cholecystitis. Lab studies ordered reveal moderate elevation of the AST, ALT, alkaline phosphatase and bilirubin. After you control her pain with analgesics and hydrate her next most appropriate step would be:

Arrange for ERCP to assess for Choledocholithiasis and possible papillotomy and extraction of common duct stones

A 69 year-old male presents with complaint of increasing dyspnea over the past 6-8 months. The patient currently denies cough, hemoptysis or chest pain. He has been a lifelong smoker and worked most of his life as a shipbuilder Physical examination reveals inspiratory crackles at the bases and clubbing of the nails. Chest x-ray reveals interstitial fibrosis of the lower lungs, thickened pleura and calcified pleural plaques of the lateral chest wall. Pulmonary function testing shows a restrictive pattern with a decreased diffusing capacity. What information is most likely noted in this patient's history?

Asbestos exposure Asbestos exposure often presents years later with increasing dyspnea and interstitial fibrosis of the lower lungs, thickened pleura and calcified pleura plaques. They will have a restrictive pattern on PFT.

An 84-year-old man presents with right upper quadrant pain, fever, and jaundice. He is alert and well-oriented. Ultrasound evaluation reveals multiple stones in the gallbladder and a slightly dilated common duct. What is the MOST LIKELY diagnosis?

Ascending Cholangitis Acute Cholangitis usually presents as fever, RUQ pain, and jaundice (Charcot's triad)

A 76 year-old active female with history of HTN and hypothyroidism presents with complaints of palpitations and dyspnea on exertion. She has normal cardiac function otherwise with a normal EF and no valvular disease. On PE VS's are BP 120/80 mmHg, HR 76 bpm, irregular, RR 16. Heart examination reveals an irregularly, irregular rhythm without murmur Lungs are clear to auscultation and extremities are without edema. EKG is on next page What is the heart rhythm on the below EKG?

Atrial Fibrillation

A patient presents to the office following a syncopal episode. The patient claims that the syncope occurs when he changes position such as rolling over in bed or when he bends over to tie his shoes. Which of the following is the most likely explanation for this presentation?

Atrial myxoma Atrial myxoma most commonly presents with sudden onset of symptoms that are typically positional in nature due to the effect that gravity has on the tumor. Myxomas are the most common type of primary cardiac tumor in all age groups and are most commonly found in the atria

Which of the following electrocardiographic findings represents a manifestation of digitalis toxicity?

Atrial tachycardia with variable block Digitalis glycosides are effective are effective in increasing myocardial contractility and in the treatment of certain atrial tachyarrhythmia's. However digitalis actually increases myocardial automaticity (increase in premature beats) and facilitates reentry (atrial tachycardia). Digoxin also slows conduction through AV nodal tissue and has central effects that can mimic vagal influence on the heart and thus may produce sinus arrest. Paroxysmal atrial tachycardia with variable block represents the classic rhythm of digitalis intoxication. Digoxin is profibrillatory but its administration should not lead to atrial flutter.

A positive direct Coombs' test may be seen in which of the following conditions?

Autoimmune hemolytic anemia A positive direct Coombs' test indicates that antibody has attached to an antigen on the RBC which causes agglutination. Autoimmune hemolytic anemia results when the patient has antibodies against their own RBCs. G6PD deficiency does result from an antibody antigen reaction. Sickle cell anemia is caused by a defective hemoglobin which detected by hemoglobin electrophoresis. Hereditary spherocytosis is detected utilizing the osmotic fragility test.

An 18 year-old college freshman presents to the student health clinic complaining of an outbreak of "athlete's foot" in his dormitory. He has no current signs or symptoms of infection, but is asking for advice on how to prevent contracting this condition. Which of the following is the best piece of preventative advice?

Avoid excessive moisture and dry the skin carefully after bathing Personal hygiene is essential for prevention. Avoiding extra moisture and drying carefully between the toes after showering is effective prevention.Wear open toe shoes and change absorbent, nonsynthetic socks regularly.

A 9 year-old boy comes to the office with his mother to discuss treatment for chronic dry skin, and pruritic inflammatory lesions of the flexor surfaces of the neck, hands and wrists. Past medical history is significant for allergic rhinitis in the spring and fall. On physical examination the skin lesions are excoriated, and lichenified with crusted patches. Which of the following prevention strategies can help minimize the symptoms in this patient?

Avoid rubbing or scratching Rubbing or scratching plaques can exacerbate the pruritus and lichenification in atopic dermatitis.

Which of the following nerves may be damaged in patients with acute anterior shoulder dislocation?

Axillary

What antibiotic would be used to treat GABS in a penicillin allergic patient?

Azithromycin

A 4 month-old infant is brought to the clinic by his mother with complaints of a cough for the past 3 weeks. Initially, symptoms included running nose, sneezing and an irritating cough. Over the past week the cough has changed to persistent staccato, paroxysmal forceful coughs ending with a loud inspiration. WBC is 20,0000/mcl with 72% lymphocytes. Which of the following is the drug of choice for managing this patient?

Azithromycin (Zithromax) Azithromycin terminates respiratory tract carriage of Bordetella pertussis

Which of the following is the treatment of choice for pulmonary Legionellosis?

Azithromycin (Zithromax) Legionella is an intracellular pathogen and therefore requires a drug that can achieve adequate intracellular concentrations. Azithromycin, along with quinolones and tetracyclines, is considered first line therapy. Rifampin may be used in combination with first-line drugs but never as monotherapy.

The most common fracture of childhood involves the

clavicle

Which of the following is not a complication of the Obesity-Hypoventilation syndrome? A. Development of hypercapnia and hypoxemia B. Pulmonary Hypertension C. Orthostatic hypotension D. Secondary polycythemia E. Eventual development of PHTN and eventual R HF

B. Pulmonary Hypertension

The test you can order for intussusception which is both diagnostic and curative is?

Barium enema

An 80 year-old woman with a 40 pack-year history of smoking presents with progressive dysphagia for solids over the past 6 months. She states that she has often had to regurgitate her food because it "gets stuck." She has had an unintentional weight loss of 20 pounds. Which of the following is the initial diagnostic test of choice?

Barium esophagography This patient's history of smoking, unintentional weight loss, and progressive dysphagia for solids makes esophageal neoplasm most likely. The initial less invasive diagnostic test is barium swallow which will detect lumen narrowing (mechanical) lesions such as cancer.

Stress-induced transverse depressions in the nails are called

Beau lines Horizontal depressions across the nail plate are caused by a transient arrest in nail growth. Acute stress such as high fever, MI, PE can cause an arrest in nail growth. The nail plate takes 3-4 months to grow from its base to its distal edge; therefore, it is possible to estimate the date of illness. Mee's lines are white transverse lines that mark a stressful event and are a variation of Beau's lines.

An African American male is placed on hydroxychlorquine (Plaquenil) for travel to Africa. Within six weeks he presents complaining of fatigue. CBC shows anemia. Which of the following is the diagnostic study of choice in this patient?

G6PD assay G6PD assay is the diagnostic study of choice for G6PD deficiency.

A patient presents with elevated mood for over two weeks, during which time she went to Las Vegas and ran up a $145,000 gambling debt, charged $4500 for clothes on her credit cards, and contracted gonorrhea from one of the seven men that she had sex with during this time. She has slept for less than two hours a night for the past several weeks. She was brought in by ambulance after attempting to gain entrance to the White House by declaring that she was the wife of the President of the United States. Medical and drug screening are within normal limits. Which of the following is the most likely diagnosis?

Bipolar disorder The manic phase of bipolar disorder is characterized by an expansive and euphoric mood, with inflated self-esteem or grandiosity, decreased need for sleep, talkative, flight of ideas, distractibility, increase in goal directed activity, excessive involvement in high-risk fun activities (spending money, sexual indiscretions). The mood disturbance is so severe that it interferes with normal social functioning, and is not related to substance abuse.

The bite from which of the following is associated with fever, lacrimation, rhinorrhea, bradycardia, hypertension, and tachyarrhythmias?

Black widow spider Black widow spider bites can inject venom that contains a neurotoxin which can produce reactions at the site of the bite along with varying degrees of systemic symptoms. Brown recluse spider bites may cause reactions ranging from mild urticaria to full thickness necrosis but typically, there are no systemic symptoms.

An 18 year-old woman presents to the clinic complaining of fatigue. She reports a past history of lifelong frequent nosebleeds and bleeding gums. She also has menorrhagia. Her mother and maternal grandfather have a similar bleeding history. Initial lab results are as follows: WBC 9,500/mm3, Hgb 10.9 g/dL, HCT 33%, MCV 69 fL, MCHC 26 pg and platelets 284,000/mm3. Which of the following tests should be ordered to evaluate this patient's diagnosis?

Bleeding time and platelet aggregometry The patient's presentation is consistent with a congential qualitative platelet disorder, most likely von Willebrand's Disease, necessitating a bleeding time and evaluation of platelet function

A 17 year-old college student is brought to the emergency department with a 10 hour history of fever, headache and increasing listlessness. Examination is significant for a temperature of 103 F, nuchal rigidity and a petechial rash on the trunk and extremities. Which of the following is the most appropriate initial treatment?

Blood culture and administration of IV fluids and empiric antibiotics Clinical presentation is meningococcal meningitis. Blood cultures must be obtained and IV antimicrobial therapy initiated immediately. This may be done prior to LP.

Which of the following symptoms most commonly indicates diarrhea of an inflammatory etiology?

Bloody stools Bloody diarrhea occurs due to mucosal inflammation and/or erosions/ulcerations and may be caused by infection, inflammation, or ischemic enterocolitis.

A patient is brought to the emergency room for evaluation of mental status changes. She has a history of fever as high as 105 F. She has had a headache and symptoms consistent with an upper respiratory infection 2 weeks before the worsening of these symptoms tonight. You are updating this patient's clinical progress. You diagnose her with a coma. You note that her respiratory pattern has become irregular, with Cheyne-Stokes breathing. Her pupils, which were previously equal, are now unequally reactive to light What is the most likely cause of her coma?

Brain Abscess Symptoms: Headache Triad of fever, headache, focal deficit, Seizures Nausea/Vomiting, Nuchal rigidity & papilledema

A 69 year-old male with a history of chronic lymphocytic leukemia presents to the clinic complaining of cough, dyspnea and production of copious amounts of foul smelling sputum. Physical examination reveals crackles at the lung bases. Chest x-ray shows dilated and thickened bronchi that appear as ring-like markings. Which of the following is the most likely diagnosis?

Bronchiectasis This patient has signs and symptoms consistent with bronchiectasis including CXR findings of dilated and thickened bronchi that may appear as tram-tracks or as ring-like markings.

A 19 year-old college student has a severe sore throat and a temperature of 102° F. On examination, there are vesicles on the soft palate with some erythema. The tonsils are not enlarged. There is mild cervical lymph node enlargement. Which of the following is the most likely diagnosis?

Coxsackie virus group A Coxsackie virus group A is characterized early by petechiae or papules/vesicles on the soft palate that become shallow ulcers in about 3 days.

A 56-year-old man presents with a history of persistent and progressive unrelenting hoarseness for the last few months. He is a 50-pack-year smoker but quit 1 year ago. Physical examination demonstrated a 2 cm firm, non-tender right anterior cervical lymph node. Which of the following is the most likely diagnosis?

Laryngeal cancer

A 54-year-old male presents to your office complaining of sleepiness. He states that he invariably naps after dinner and normally is in bed for 8 to 10 h nightly. He does report, however, that he feels that his sleep is fragmented. When the patient awakens in the morning, he rarely feels refreshed. His wife has complained for years of his snoring to the extent that she now sleeps in a different room. The patient notes that he frequently has difficulty staying awake during the late afternoon at work. He is seeking evaluation now because his job performance has been impaired and he has been put on probation at work. Past medical history is significant for hypertension since age 38 that has recently worsened despite medication. Body mass index is 36 kg/m2. He is noted to have central obesity and a thick neck. You suspect obstructive sleep apnea (OSA). All the following are long-term consequences of untreated OSA except: A. Personality disturbance B. Worsening of congestive heart failure C. Increased risk of proteinuria and chronic kidney disease D. Increased risk of systemic hypertension E. Increased risk of motor vehicle accidents

C. Increased risk of proteinuria and chronic kidney disease

A 30-year-old male patient presents with a recurrent history of mouth and jaw pain. He states that he gets swelling on the right side of his jaw/ face that begins while eating and subsides within 2 hours after he is done eating. On physical exam, there is no swelling or pus noted, but a mass is palpable on the right side of the jaw, near the TMJ. No other significant findings are noted. What is the suspected cause?

Calculus formation in the Stenson's duct

A 25 year-old female with a history of diarrhea presents to the emergency department with Guillian-Barre syndrome. What is the most likely causative pathogen in this patient?

Campylobacter jejuni Campylobacter jejuni is a Gram-negative bacteria that has been linked to the subsequent development of Guillain-Barré syndrome (GBS). GBS usually develops two to three weeks after the initial illness

A 75-year-old male presents with a two-day history of SOB at rest. He states he has not been able to complete his normal daily activities. On PE, his BP is noted to be 90/40 mm Hg, pulse 95/minute. Diffuse pulmonary rales bilaterally, and 3+ pitting edema to the mid-tibia bilaterally are noted EKG, serum electrolytes and cardiac enzymes are normal. Which of the following is the best initial therapy for this patient?

Captopril and hydrochlorothiazide Congestive heart failure presents with shortness of breath and decrease in ability to perform normal daily activities. On physical examination, rales are noted on pulmonary exam and edema is noted in dependent areas. Diuretics are the most effective at providing relief of symptoms and inhibition of the renin-angiotensin-aldosterone system with an ACE inhibitor should be considered part of the initial treatment plan.

A patient presents with decreased hearing and fullness in the right ear. Otoscopic exam reveals hard, yellow/ brown debris. The tympanic membrane can not be visualized. What is the treatment of this ear condition?

Carbamide peroxide

A patient has developed greater than normal(10mm Hg) inspiratory decreases in systolic BP. The arterial pulse (taken with the patient's wrist overlying his lap) almost seems to disappear when palpated during inspiration These findings are most consistent with:

Cardiac tamponade Prolonged and severe inflammation leads to fluid accumulation around the heart. Can lead to Cardiac Tamponade. Friction rub may be noted if effusion secondary to pericarditis. Need Echo. Pericardiocentisis may be necessary.

A 57-year-old man is noted to have a BP of 68/50 mm Hg, HR of 140 bpm, elevated jugular venous pressure, inspiratory crackles on exam, and cold clammy extremities. Which of the following is the MOST likely etiology?

Cardiogenic shock The patient is hypotensive with signs of left and right heart failure, that is probably cardiogenic shock. Septic shock and adrenal crisis are forms of distributive shock that would produce warm extremities. Hypovolemic shock should have flat neck veins and no pulmonary edema.

A 40 year old female is brought into the emergency department status post fall. She is C-spine immobilized complaining of low back pain with shooting pains down the right leg. She apparently has lost function of her bowels and bladder. Your physical exam reveals significantly decreased motor function in the lower extremities and complete loss of sensation in both feet. During rectal exam, you note loss of sphincter tone and

Cauda equina syndrome Mechanism: Rare condition involving large midline disc herniation that compresses several nerve roots, usually at L4-L5 level Causes: trauma, lumbar disc disease, abscess, spinal anesthesia, tumor, metastatic disease, late stage ankylosing spondylitis, and idiopathic Signs and Symptoms: low back pain, unilateral or bilateral sciatica, saddle anesthesia with poor rectal tone, bowel/bladder incontinence, lower extremity motor and sensory loss Must ask if bowel or bladder incontinence Must perform sensation exam Need ortho and neuro referrals immediately

A 31 year-old female complains of diarrhea. She admits to associated weight loss and steatorrhea, but denies melena or hematochezia. Which of the following is the most likely diagnosis?

Celiac disease Greater than 10 grams of fecal fat in 24 hours is most consistent with a malabsorption syndrome such as celiac sprue.

A 26-year-old African American woman presents to her physician with complaints stemming from the symptoms of a urinary tract infection. Patient states that the infection began approximately 2 weeks after giving birth. As you begin your investigation, as part of your disease and disorder considerations, which of the following would be a likely cause of her type of condition?

E. coli

A 78 YO W male presents with unilateral painless loss of vision in the right eye for 3 hours. Visual acuity is light perception only in the right eye and 20/30 in the left eye. He has an afferent pupillary defect on the right side. Retinal examination OD reveals a deep-red spot. Retinal examination of the OS is unremarkable. What is the most likely diagnosis in this patient?

Central retinal artery occlusion

A 2 year-old presents with sudden onset of cough and stridor. On examination the child is afebrile and appears nontoxic with a respiratory rate of 42 breaths per minute. What is the next step in the evaluation of this patient?

Chest x-ray Chest x-ray should be done first when foreign body aspiration is suspected.

A recent Haitian immigrant presents to the clinic for an employment physical examination before starting work at a local hospital. The patient has a history of receiving bacilli Calmette-Guerin (BCG) vaccination. Screening for tuberculosis for this employee should include which of the following tests?

Chest x-ray Chest x-ray is the test of choice in patients where the PPD test is not indicated or in high-risk individuals.

Routine prophylaxis for patients going to an area of malaria infestation requires the administration of which chemoprophylaxis agent?

Chloroquine The use of Chloroquine is still recommended as the drug of choice for prophylaxis in areas of non-resistant falciparum malaria.

A 69 year-old female presents to the clinic complaining of fatigue. Physical examination reveals lymphadenopathy, splenomegaly and pale conjunctiva. The remainder of the examination is unremarkable. CBC reveals a normochromic normocytic anemia. White blood cell count is 45,000/mm3, with a differential of 77% mature lymphocytes, 3% eosinophils, 18% segmented neutrophils and 1% basophils and monocytes. Platelets appear adequate in number. What is the most likely diagnosis?

Chronic lymphocytic leukemia Chronic lymphocytic leukemia typically occurs after age 50 presenting with lymphocytosis with lymphocytes that appear small and mature.

A 56 year-old female with a 35 pack year smoking history presents to the clinic with shortness of breath and cough. On examination, she is thin with no recent weight loss. She appears uncomfortable, breath sounds are diminished without adventitious sounds. Pulmonary function tests show a marked increase in total lung capacity (TLC) and a decreased FEV1. What is the most likely diagnosis for this patient?

Chronic obstructive pulmonary disease Lung volume measurements in COPD reveal a marked increase in residual volume indicative of air trapping.

Which of the following descriptions is most typical of a patient with acromegaly?

Coarsened facial features with prognathism and prominent brow Coarsened facial features with prognathism and prominent brow are typical findings for acromegaly

A 20 year-old male presents to the ED with complaints of palpitations and agitation, which developed suddenly while attending a party. On examination, the patient is moderately agitated and tremulous. Vital signs include a pulse of 110/minute and regular; respiratory rate 22/minute and blood pressure 160/92 mmHg. Skin is diaphoretic and pupils are dilated. Which of the following is the most likely diagnosis?

Cocaine use Cocaine, as well as amphetamines, leads to a clinical picture of increased sympathetic stimulation and dilated pupils.

An asymptomatic 30 year-old male with no significant past medical history presents to the office. He is concerned about his father being diagnosed with colon cancer at the age of 60. Which of the following is the most appropriate health counseling for this patient?

Colonoscopy at age 40 It is recommended that patients with a single first-degree relative with colon cancer diagnosed at or after age 60 should begin screening at age 40 preferably with a colonoscopy.

A 13 year-old male presents with a two day history of fatigue and fever. He also complains of recent epistaxis and bleeding gums. Examination demonstrates pale skin with palpable purpura. Laboratory evaluation demonstrates pancytopenia with 42% blasts and a positive D-dimer. Which of the following treatment regimens is most likely to bring remission?

Combination chemotherapy Combination chemotherapy is key in the treatment of acute lymphocytic leukemia and produces remission in 90% of patients

A newborn presents with a possible cyanotic, CHD. Which of the following is in the differential diagnosis?

Complete transposition of the great arteries

A 67 y/o patient with a 15 year history (COPD) presents with worsening respiratory symptoms and abdominal pain. On PE , you observe peripheral edema, JVD , epigastric pulsations, and tender hepatomegaly. You auscultate a SEM . Echocardiogram reveals a dilated RV with decreased EF and a normal appearing LV. Which of the following is the most appropriate diagnosis for this patient?

Cor pulmonale Cor pulmonale denotes RV hypertrophy and eventual failure resulting from pulmonary disease. It is most commonly caused by COPD. The signs and symptoms presented are classic for the diagnosis.

A 36 year-old patient presents with poor wound healing following sutures to a hand laceration two weeks prior. She had regular periods up to 4 months ago, then they ceased unexpectedly. Examination reveals central obesity, a prominent dorso-cervical fat pad and hirsutism. Which of the following is the most likely diagnosis in this patient?

Cushing syndrome This patient is exhibiting signs and symptoms of Cushing syndrome or hypercortisolism, which include poor wound healing, central obesity, moon facies, buffalo hump, hirsuitism and amenorrhea.

A 65 year-old male smoker presents with a new onset of hematuria. Which of the following studies would be the most definitive diagnostic study in this patient?

Cystoscopy

In a patient with a low probability of pulmonary embolism (PE) based on the Wells criteria, which of the following lab studies is most helpful in ruling out a PE?

D dimer assay Even though useful in ruling in a pulmonary embolism, the D dimer is negative in over 95% of patients who do not have a pulmonary embolism.

When a patient presents with newly diagnosed heart failure, the clinician should search for possible reversible precipitating factors. Which of the following conditions may be surgically correctable. A. Constrictive pericarditis B. Calcific AV stenosis C. Acute MVR 2nd to bacterial endocarditis D. All of the above E. None of the above

D. All of the above

Coronary artery bypass graft (CABG) surgery may be indicated as the treatment of choice for angina pectoris with which of the following angina patients? A. A patient with triple-vessel disease B. A patient with one-vessel disease C. A patient with two-vessel disease D. CABG may be first-line therapy in any of the above

D. CABG may be first-line therapy in any of the above

A 49-year-old male presents to the clinic with symptoms of nausea, occasional vomiting, vague epigastric pain, nocturnal pain, fatigue, and weight loss of 35 lbs. over the past few months. On exam you find a palpable abdominal mass. Which of the below is the most likely diagnosis?

Gastric adenocarcinoma Weight loss and palpable abdominal mass is highly suggestive of gastric adenocarcinoma

Which of the following studies is most appropriate in a patient diagnosed with sleep apnea? A. EEG to assess stage sleep patterns B. Ventilation pattern to detect apnea C. Arterial O2 saturation D. Polysomnography to include all of the above

D. Polysomnography to include all of the above

A 48-y/o woman c/o lightheadedness & some discomfort in her chest. VS BP 75/48; HR 150. Rhythm strips seen below (A-Flutter EKG) Which of the following is the most appropriate next step?

DC cardioversion at 50 joules (biphasic) Treatment of Aflutter VR Control Same as AF (calcium channel blockers, beta blockers, digoxin) Acute electrical cardioversion If hemodynamic instability exists Type III agents (sotalol, amiodarone) used to facilitate cardioversion or maintenance of NSR Radiofrequency ablation May be used to eliminate circuit responsible for atrial flutter with a 90% success rate

A 21 year-old male with a diagnosis of type 1 von Willebrand disease undergoes dental extraction of his wisdom teeth. The patient comes to the clinic with continued oozing of the dental sockets despite packing. Treatment should begin with which of the following?

DDAVP DDAVP causes the release of vWF and factor VIII from storage sites significantly which is needed to complete hemostasis. Factor VIII is indicated for patients with Hemophilia A. Fresh frozen plasma is indicated in Coumadin overdosage and vWF concentrate is indicated in type 2 and 3 patients with von Willebrand disease.

A 31 yr old woman presents c/o a lump to her left breast x 3 days. She noticed the lump while showering. She denies any nipple discharge or breast pain but reports mild tenderness with palpation. She has never had any other breast lumps in the past. She takes OCPs daily and a MVI. -Denies personal or FHx of breast disease -Does not smoke or drink alcohol -Has never been pregnant -No previous mammogram PE: 104/68 P - 68 R - 16 T - 99.4F General: slender white female, A&O x3, appears somewhat anxious Lymph: no LAD Breasts: Left - 3 cm firm, well circumscribed, mobile mass noted at 1:00. Mildly tender to palpation. No dimpling, retraction or discharge noted. Right - no masses, retraction, dimpling or discharge noted Remainder of PE is wnl. What are DDx? What is the Dx? What evidence supports your choice? First Step evaluation? Next step? Risk Factors for breast cancer?

DDx: Fibroadenoma, Breast cyst, cancer, or abscess, fibrocystic breast, mastitis Dx: Cyst or fibroadenoma Evidence: Age (Cancer is usually >50 y/o) and her benign lesion characteristics First step: Breast Ultrasound Next step: Reassurance, FNA (clear/yellow: f/u 1-2 months or no fluid, bloody, or does not resolve then biopsy) Risk factors: FHx, early menarche, nulliparity, HRT, Obesity, physical inactivity, alcohol use

62 year-old female presents to the emergency room with significant back pain without radiation after lifting a box weighing approximately 15 pounds. She denies any previous trauma or injuries. Past history includes hysterectomy at age 42 and a 49 pack year smoking history. Her current weight is 107 pounds. Lumbo-sacral spine film indicates a spinal compression fracture at level L4. Which of the following tests would you perform to further assess the patient's findings?

DEXA scan Low patient weight, smoking, and early estrogen deficiency are all risk factors for osteoporosis. Spontaneous fractures occurring from lifting with above risk factors should be evaluated for bone density.

70-year-old ICU patient c/o fever and shaking chills. The patient develops hypotension and blood C/S are + for gram-negative bacilli. The patient begins bleeding from venipuncture sites and around the foley catheter: Laboratory studies are as follows: Hct 38%WBC 15,000/mLPlatelet count: 40,000(normal 130,000-400,000)PBS: fragmented RBC'sPT & PTT elevatedPlasma fibrinogen 70 mg/dL (normal 200-400) The patients coagulopathy is most likely caused by?

DIC

A 70 year-old patient presents with complaints of chronic, worsening shortness of breath for the past few years. He admits to a 60 pack-year history of tobacco use but no industrial exposures. Examination reveals a barrel chest, hyperresonant thorax and faint breath and heart sounds. Which of the following would be the most likely pulmonary function abnormality in this patient?

Decreased FEV1, decreased FEV1/FVC ratio

Which of the following laboratory values would be noted in a patient with primary hyperthyroidism?

Decreased TSH, increased free T4 Serum TSH is depressed and serum T3, T4 and uptake are usually all increased in patients with primary hyperthyroidism.

A 50 year-old female presents describing poor concentration, generalized fatigue, oligomenorrhea and severe acne outbreaks over the past year. On examination she exhibits central obesity, a round face, numerous striae and elevated blood pressure. Which of the following is the most appropriate initial diagnostic study?

Dexamethasone suppression test Dexamethasone suppression test as well as midnight serum cortisol level, late night salivary cortisol level, and urinary free cortisol used singly or in combination to diagnose Cushing's syndrome (Hypercortisolism)

A supervised vasopressin challenge test is a significant part of the diagnostic workup for which of the following endocrine disorders?

Diabetes insipidus Vasopressin is given in an initial dose of 5-10 mcg intranasally (or 1 mg subcutaneously or intravenously) with measurement of urine volume for 12 hours before and 12 hours after administration. Patients with central diabetes insipidus notice a distinct reduction in thirst and polyuria. Serum sodium should remain normal.

A 52-year-old man comes to the outpatient clinic complaining of bilateral, gradual defective vision. He visited his ophthalmologist 6 months ago, and he performed an out-patient treatment session where he was asked to follow up after 6 months. The patient has a 15-year history of NIDDM and is on metformin. The blood sugar level is usually well-maintained. He has some skin trophic changes in the lower limbs but no impairment of sensation. The vision is 20/100 bilaterally. The lens is clear, and the vitreous shows no abnormalities. The image included is the fundus of the patient's right eye. What is the most likely diagnosis?

Diabetic retinopathy

A 45-year-old man presents with a history of crushing nonradiational chest pain. EKG & exercise stress testing reveal no evidence of cardiac ischemia A more detailed history is taken, and the patient states that he has had a sensation of sticking after swallowing. He notes this sensation equally whether he is eating solids or liquids. UGI series revealed a "corkscrew/ rosary bead" esophagus. The diagnosis that would most likely account for these symptoms is

Diffuse esophageal spasm Sticking" during the passage of food through the mouth, pharynx, or esophagus is almost always associated with a significant pathologic problem. Since this patient has dysphagia with both solids and liquids and has severe chest pain, diffuse esophageal spasm is the likely diagnosis.

Which of the following is the most characteristic physical examination finding with Grave's disease?

Diffuse thyroid gland enlargement Diffuse thyroid gland enlargement is the typical presentation for Grave's disease.

A 50 y/o male with history of alcohol abuse presents with complaint of worsening dyspnea. PE reveals bibasilar rales, elevated JVP , an S3 and lower extremity edema. CXR reveals pulmonary congestion and cardiomegaly. EKG shows frequent ventricular ectopy. Echocardiogram shows left ventricular dilatation and an EF of 30%. Which of the following is the most likely diagnosis in this patient?

Dilated cardiomyopathy

A 28 year-old female, 3 months post-partum, complains of gradually increasing DOE . She also complains of near syncope last week. She denies chronic medical conditions and hospitalizations other than for the delivery of her child. She denies tobacco, alcohol or illicit drug abuse. Exam is noteworthy for a laterally displaced apical impulse, an S3 and a SEM best heard in the left axilla. Which of the following is the most likely diagnosis?

Dilated cardiomyopathy. Dilated cardiomyopathy complicates 1 in 3000 to 1 in 15,000 pregnancies and can present from the last month of pregnancy to five months post-partum. Signs and symptoms of congestive heart failure are typical on presentation. The mitral insufficiency is the result of the dilation of the annulus.

A patient receiving heparin therapy for 6 days for deep vein thrombosis develops thrombosis at the IV site. The INR is 1.1. The aPTT is 66 seconds and the platelet count is 47,000 down from 148,000 on admission. Which of the following is the most appropriate treatment?

Discontinue the heparin Once the diagnosis of heparin induced thrombocytopenia is suspected treatment should include immediate discontinuation of all forms of heparin and treatment with a direct thrombin inhibitor should begin. Platelet administration is generally not indicated. Protamine sulfate is used for heparin overdosage and plasmapheresis would not be indicated as the platelet count will resume when the heparin is discontinued

Which of the following is an antibiotic of choice in the treatment of Rocky Mountain spotted fever(RMSF) ?

Doxycycline Drugs of choice for the treatment of RMSF are doxycycline or chloramphenicol (50 mg/Kg/day). Administration of doxycycline for 7-10 days is generally sufficient (to cover the febrile period and 2-3 days following abatement of fever. To date, no resistant strains to these drugs have been reported

A 32 year-old male presents to your office 1 week following a camping trip to New England. He complains of low grade fever, chills and myalgias. On physical examination a 3 cm, erythematous, papular lesion with central clearing is identified on the left thigh. Which of the following is most appropriate for this patient?

Doxycycline In cases where patients present with a pathopneumonic lesion consistent with Lyme disease, treatment is recommended and the Lyme titer is not indicated.

An 80 year-old male from an assisted living facility is brought in for evaluation of repeated near-syncopal episodes when getting up from a seated position. His past medical history is significant for DM and HTN for which he takes enalapril (Vasotec). Which of the following would you expect on PE ?

Drop in systolic BP of at least 20 mmHg and a rise in pulse rate of more than 15 beats per minute immediately upon arising from a supine position. A greater than normal decline (20 mmHg) in BP immediately upon arising from the supine to the standing position is observed. With/without tachycardia depending on the status of autonomic (baroreceptor)

Which of the following is the diagnostic study of choice in the evaluation of Zollinger-Ellison syndrome?

Gastrin level Zollinger-Ellison Syndrome (ZES) is characterized by elevated levels of serum gastrin resulting from gastrinsecreting tumors usually located in the duodenum or pancreas.

Which of the following is true about massive obesity and its effect on the respiratory system ? A. Massive obesity imposes a mechanical load on respiratory system B. Small percentage of morbidly obese pts develop hypercapnia, hypoxemia, & ultimately polycythemia, PHTN, & R HF C. Most pts have mild to moderate airflow obstruction D. Treatment includes weight loss, smoking cessation, & pharmacological respiratory stimulants such as progesterone E. All of the above statements are true

E. All of the above statements are true

Which of the following statements regarding chronic bronchitis is true? A. The disease develops in 10% to 15% of cigarette smokers B. Cigarette smokers in whom this disease develops usually report the onset of cough with expectoration 10 to 12 years after smoking began C. Dyspnea is noted initially only on extreme exertion; as the condition progresses, it becomes more severe and occurs with mild activity D. Pneumonia, pulmonary hypertension, cor pulmonale, and chronic respiratory failure characterize the late stages of the disease E. All of the following are true

E. All of the following are true

Which of the following is a(are)specific recommendation(s) regarding DVT prophylaxis? A. Patients at moderate risk for any reason should receive low-dose heparin or intermittent pneumatic compression B. Patients undergoing neurosurgical procedures should be treated with pneumatic compression C. Patients undergoing urological procedures should be treated with pneumatic compression D. Patients undergoing surgery for hip fracture should receive warfarin to a PT of 1.2 to 1.5 times that of the control E. All of the above

E. All the above

Which of the following is/are true of myasthenia gravis? A. increase in weakness as the day progresses B. may be complicated by a cholinergic crisis C. deterioration of strength often caused by an infection D. may be associated with a thymoma E. all of the above

E. all of the above

Conditions that increase a patient's risk for development of avascular necrosis include all of the following EXCEPT: A.Chronic alcoholism B.Trauma C.Sickle cell disease D.Long term corticosteroid use E.Ankylosing spondylitis

E.Ankylosing spondylitis

What is the treatment of choice for the previous patient (S. pneumoniae pneumonia)?

EES & Ceftriaxone

What diagnostic test will identify the most likely etiology of the patient's symptoms (ASD)?

Echocardiogram

A 65-year-old patient presents with dyspnea at rest, two-pillow orthopnea and a 2+ bilateral ankle edema. Which of the following diagnostic tests would be most helpful in the evaluation of this patient?

Echocardiogram Dyspnea, orthopnea and edema are consistent with a diagnosis of congestive heart failure. Echocardiogram should be obtailed to assess cardiac function and ejection fraction.

A 35 y/o male status-post PDA repair at 5 years of age, presents with low-grade fever, fatigue and dyspnea worsening over the past 10 days. Prior to the onset of these symptoms, he was healthy and free of any complaints. Examination is significant for petechiae on the palate, a high pitched holo-systolic murmur heard best at the apex, and splinter hemorrhages on both hands under his fingernails. Which of the following is the most appropriate next step in the evaluation of this patient?

Echocardiogram The history is most representative of a patient with infectious endocarditis. The clinical criteria, referred to as the modified Duke criteria include (1) two positive blood cultures for a microorganism that typically causes infective endocarditis or persistent bacteremia (2) evidence of endocardial involvement documented by echocardiography or (3) development of a new regurgitant murmur. Diagnosis is made if one major and one minor criterion or three minor criteria are met.

You are seeing in your office a patient with the chief complaint of relatively sudden onset of SOB and weakness but no CP. ECG shows nonspecific ST-T changes. You would be particularly attuned to the possibility of painless, or silent, MI in the

Elderly diabetic Be weary of the five causes of silent / atypical MI presentations: D Diabetes E Elderly A Alcohol T Trauma to the thoracic spinal cord H Hypertension Women more commonly than not, have atypical presentations Presentation in these patients is often syncope or dyspnea or undue fatigue

A 55 year-old female presents with a lesion on her face that is painful, bright red, with distinct raised borders. She also is complaining of fever and chills. There is no evidence of any marks which would show a portal of entry. Which of the following is the most likely diagnosis?

Erysipelas Erysipelas is a distinct type of superficial cellulitis with redness, a distinct and raised border, and sharp demarkation from uninvolved skin. It is typically associated with systemic symptoms such as fever and chills. It is caused by group A strep most commonly

A 37-year-old woman with Raynaud's phenomenon complains of progressive weakness with inability to arise out of a sitting position without assistance. On examination, the patient has swollen "sausage-like" fingers, alopecia, erythematous patches on the knuckles, facial telangiectasias, and proximal muscle weakness. Laboratory evaluation includes a normal CBC and serum chemistries, except for creatine phosphokinase, 4.5 kat/L (270 U/L), and aldolase, 500 nkat/L (30 U/L). The following serologic profile is found: rheumatoid factor is positive at 1: 1600; ANA is also positive at 1:1600 with a speckled pattern and very high titers of antibodies against the ribonuclease-sensitive ribonucleoprotein component of extractable nuclear antigen. This patient probably has

Emixed connective-tissue disease (MCTD) MCTD is a syndrome characterized by high titers of circulating antibodies to the ribonucleoprotein component of extractable nuclear antigen in association with clinical features similar to those of SLE, systemic sclerosis, polymyositis, and rheumatoid arthritis. The average patient with MCTD is a middle-aged woman with Raynaud's phenomenon who also has polyarthritis, sclerodactyly (including swollen hands), esophageal dysfunction, pulmonary fibrosis, and inflammatory myopathy. Cutaneous manifestations include telangiectasias on the face and hands, alopecia, a lupus like heliotropic rash, and erythematous patches over the knuckles. Myopathy may involve severe weakness of proximal muscles associated with high levels of creatine phosphokinase and aldolase. Both pulmonary involvement and esophageal dysmotility are common but are frequently asymptomatic until quite advanced. Almost all patients have high titers of rheumatoid factor and antinuclear antibodies. Such antibodies are directed toward the ribonuclease-sensitive ribonucleoprotein component of extractable nuclear antigen.

A 60-year-old man presents to your office with an 80-pack-year history of cigarette smoking. He complains of some dyspnea on exertion. He has an asthenic body habitus and pursed-lip breathing. He has an increased anteriorposterior thickness in the thorax. Lung examination reveals decreased fremitus, hyperresonance on percussion, and diminished breath sounds. CXR is seen on the right. Which of the following is the most likely diagnosis?

Emphysema

Which of the following agents has been shown to reduce mortality in patients with congestive heart failure?

Enalapril The agents most typically used in the treatment of congestive heart failure-diuretics and cardiac glycosides-have never been formally shown to prolong survival. However, there have been at least four trials that have demonstrated benefit in the use of afterload reduction in the treatment of heart failure.

A 66 year-old female presents to your office complaining of progressive difficulty swallowing over the last 6 months. Initially she had difficulty only with meats, but now she has dysphagia with other foods as well. Which of the following is the most appropriate initial diagnostic study?

Endoscopy Endoscopy is the diagnostic study of choice in a patient with progressive dysphagia because of its ability to obtain tissue for diagnosis.

A 75 year-old man with a long history of COPD presents with acute onset of worsening dyspnea, increased productive cough, and marked agitation. While in the emergency department he becomes lethargic and obtunded. His ABG reveals a PaO2 40 mmHg, PaCO2 65 mmHg, and arterial pH 7.25. Which of the following is the most appropriate management at this point?

Endotracheal intubation and mechanical ventilation This patient is in severe respiratory arrest with markedly impaired mental status; conventional mechanical ventilation is required.

A 30-year-old woman recently spent 3 weeks touring rural areas in Mexico. The patient has experienced persistent diarrhea since returning and has come to your clinic following the onset of bloody stools for 3 days. Microscopy of a fecal smear discloses cysts and amoeba with ingested erythrocytes. What is the most likely etiologic agent involved in this infection?

Entamoeba histolytica Entamoeba histolytica, a common parasite that causes amebiasis is endemic to warm climates lacking adequate sanitation.

A child is brought to your office with high fever, difficulty swallowing, and drooling over the past several hours. On exam you notice an ill-appearing child with inspiratory stridor, breath sounds that are equal bilaterally, and symmetrical chest movement. The most likely diagnosis is

Epiglottitis

A 68 year-old male with a history of alcohol abuse presents with coffee-ground emesis. He denies vomiting prior to this episode. What is the most likely cause of his bleeding?

Erosive gastritis Drugs and alcohol are the most common causes of upper gastrointestinal bleeding.

A. 61 year old female presents with complaints of breaking out on her face and neck(see below). She relates a history of cellulitis on her face, and back, but no other history of skin problems. She has no allergies to food or medication, has had no recent international travel and avoids recreating in the outdoors. With the rash, she noticed some fever yesterday, and had chills last night before she went to bed. The lesion is red, warm to touch, and tender. The borders are clearly demarcated and sharp. This is likely which of the following?

Erysipelas Symptoms and P.E. prodromal symptoms, pruritus, "burning" sensation, tenderness begins as a small erythematous patch that progresses to a fiery-red, indurated, tense, and shiny plaque classically exhibits raised, sharply demarcated, advancing margins Prodromal symptoms usually are present within 48 hours of cutaneous involvement Etiology superficial bacterial skin infection characteristically extends into the cutaneous lymphatics Streptococcus pyogenes occurs on the face and legs bacterial inoculation into an area of skin trauma is the initial event predisposing factors: Diabetes Alcoholism HIV infection Immunocompromising conditions rapidly invades and spreads through the lymphatic vessels

A 16-year-old student with a history of herpetic gingivostomatitis develops a generalized and symmetric rash. The lesions are 1-3 cm in diameter and look like round patches. They consist of two concentric rings surrounding a central disk. The rash is burning and pruitic. A few erosive lesions are visible in the oral mucosa. Which of the following is the most likely diagnosis?

Erythema multiforme "Target Lesion" eruptions in which the lesions are slightly raised circular and more erythematous peripherally. Vesicles or bullae my be present centrally Often localized to the hands and feet, but can be generalized Patients complain of fever, malaise, and weakness. The lungs and eyes may be affected Three most common causes are drug reaction (particularly penicillin's and sulfonamides) or concurrent herpetic or Mycoplasma infection Treatment removal of trigger low dose antivirals cyclosporin corticosteroids Cytoxan

All of the following statements about obstructive sleep apnea syndrome are true EXCEPT ? A. Men are affected more often than woman B. Systemic HTN is a common finding C. Alcohol can be a contributing factor D. Estrogens are frequently useful E. Personality changes may be the presenting complaint

False: Estrogens are frequently useful

A 45 year-old man presents for a routine appointment. He tells you his mother and father have both had ischemic strokes in their 70's. He does not smoke. His blood pressure is 128/80 mmHg, pulse 78/minutes and regular, respiratory rate of 12/minute. What diagnostic studies would you order to further evaluate this patient's risk of stroke?

Fasting lipid profile Hyperlipidemia is a known risk factors for stroke that can be modified with treatment.

A 41 year-old female presents with RUQ pain. She is febrile and slightly tachycardic. Examination demonstrates abdominal tenderness and abrupt cessation of inspiration with palpation under the right costal margin during deep inspiration. Ingestion of which of the following is most likely to have proceeded the onset of this condition?

Fatty meal. Cholecystitis presents with complaint of RUQ pain and on exam there may be a positive Murphy's sign. It is associated with meals high in fat. Alcohol, dairy products and high fiber meals are not associated with cholecystitis and would not present as fever, tenderness and positive Murphy's sign

A 42-year old man develops fever and chills within a few hours after a blood transfusion. His temperature is 101.6 F and blood pressure is 120/80 mm Hg. He is slightly tachycardic but his respiratory rate is normal. His CBC is normal except for the anemia for which he was receiving the transfusion. Laboratory data including electrolytes, liver function tests and urinalysis are normal. Which of the following si the most likely diagnosis?

Febrile nonhemolytic reaction to blood transfusion

Which of the following types of hip fracture has the highest risk for avascular necrosis and nonunion?

Femoral neck Fractures involving the femoral neck typically disrupt the blood supply to the femoral head and may cause avascular necrosis and nonunion

A patient with a history of multiple trauma has received a combination of 12 units of packed red blood cells and whole blood while in the emergency department. Later that night it is noted that the patient is beginning to ooze blood from sites of injury and needle punctures. Which of the following is the most appropriate intervention?

Fresh frozen plasma When a patient receives more than one total blood volume (10 units) without replacement of clotting factors, the patient will continue to bleed without administration of fresh frozen plasma

Two weeks after returning from a camping and fishing trip in the mountains of New York. A 32-year-old man develops abdominal pain and profuse, watery diarrhea that has persisted over several days. No blood in the stool. Exam of 3 successive stool samples shows cysts and a flagellated protozoan with a ventral sucking disc. What is the most likely etiologic agent involved in this infection?

Giardia lamblia Giardia lamblia often is acquired from drinking mountain lake water and presents with crampy abdominal pain, flatulence and non-bloody profuse watery diarrhea. Stools for O & P will be positive showing a flagellated protozoan with a ventral sucking disc. Treatment is with Flagyl.

A 28-year-old man has had nausea, vomiting, and diarrhea for 36 hours. His symptoms began 12 hours after eating turkey, which had been left unrefrigerated in the oven for 3 days. Because of dehydration, you admit him to the hospital overnight for intravenous fluids. You obtain the following liver function test results: Alkaline phosphatase = 75 Units/L (30-120) Alanine aminotransferase (ALT) = 27 Units/L (0-35) Aspartate aminotransferase (AST) = 30 Units/L (0-35) Total bilirubin = 3.0mg/dL Direct (conjugated) bilirubin < 15% of Total bilirubin Of the following, which is the most likely explanation for this pattern of liver chemistry results?

Gilbert's syndrome

What examination finding would be expected in a patient with von Willebrand's disease?

Gingival bleeding A patient with von Willebrand's disease most commonly presents with mucosal bleeding seen in epistaxis, gingival bleeding, and menorrhagia

A 54 year-old male smoker presents to the clinic complaining of frequent vague headaches with associated vomiting that awaken him from sleep occasionally and have been present upon awakening for about two weeks. The headache typically resolves about an hour into his morning routine. The patient is afebrile. What is the most likely cause of this patient's headaches?

Glioblastoma Morning headaches associated with vomiting are indicative of increased intracranial pressure and raise concern of a CNS tumor such as a glioblastoma.

A 22 year-old woman comes to the office because her urine is cola-colored and she has not urinated since yesterday morning. Her past medical history is significant for pharyngitis two weeks ago. Her mother and grandmother have type 2 diabetes. Her blood pressure is 146/92 mm HG. On physical examination, she has edema of her face and hands. Which of the following is the most likely diagnosis?

Glomerulonephritis

A 24 year-old female presents with complaints of abdominal pain, bloating, and chronic diarrhea. She says she felt worse last month while on vacation in Italy. Despite eating well she lost weight. What would be the most appropriate treatment for this patient?

Gluten-free diet Celiac disease is a diffuse disease of the small bowel that is caused by immunologic malfunction. This disease is active only in the presence of gluten, a constituent of wheat. Avoidance of gluten-containing foods is the treatment of choice.

A 21-year-old sexually active man presents with a 5-day history of fever, chills, and persistent left ankle pain and swelling. On PE, maculopapular and pustular skin lesions are noted on trunk and extremities. He denies any symptoms of GU tract infection Examination of the synovial fluid revealed 190,000 WBC's/cu mm and 95% PMN's. The MOST likely diagnosis is

Gonococcal arthritis Epidemiology & Clinical Manifestations Less common than DGI & always follows disseminated infection, which is unrecognized in 1/3 of patients A single joint, such as hip, knee, ankle, or wrist, usually involved Synovial fluid, which contains >50,000 leukocytes/ L, can be obtained; gonococcus is only occasionally evident in gram-stained smears, & C/S of synovial fluid + in <40% Because it is difficult to isolate gonococci from synovial fluid & blood, specimens for culture should be obtained from potentially infected mucosal sites Treatment Ceftriaxone(1g/d) until improvement; the 7-day course can be completed with an oral fluoroquinolone(ciprofloxacin 500 mg bid) If fluoroquinolone resistance is not prevalent a fluoroquinolone can be given for entire course

For the past 2 days, a 24-year-old woman has had fever and pain in the left wrist, right ankle, and left knee. Nine painful skin lesions are present on the distal extremities, predominantly about the joints The most likely diagnosis is

Gonococcemia DGI Skin lesions occur on distal extremities, usually around joints, and appear within a week of the onset of joint symptoms. The lesions, which may number as many as 20 (average 4-5) often painful Each crop of new lesions is associated with a temperature rise. Lesions begin as a red macule or purpuric spot and then develop into a papule, a vesicle, and finally, a pustule. Organisms rarely are cultured from the skin lesions; Can be demonstrated occasionally on Gram stain and more regularly with immunofluorescent techniques.

Which of the following histories best describes spinal stenosis?

Gradual onset of back and thigh pain exacerbated by walking and alleviated by sitting Spinal stenosis typically presents as back pain and thigh pain aggravated by ambulation and relieved by sitting.

Radioactive iodine is most successful in treating hyperthyroidism that results from

Grave's disease Radioactive iodine is an excellent method to destroy overactive thyroid tissue of Grave's disease

An 11 year-old female recent immigrant from Haiti, presents to your office with a 3 day history of mild sore throat, low grade fever and malaise. There had been a recent outbreak of diphtheria in your area. Which of the following physical examination findings would you expect in this patient?

Gray membrane coating the pharynx Pharyngeal diphtheria presents with gray membrane covering the tonsils and pharynx

A 33 year-old male presents to the emergency department reporting progressive distal-to-proximal upper and lower extremity weakness over the past 24 hours. Examination reveals no muscular atrophy, but absent patellar and biceps reflexes. Sensory examination over the same area is intact. Which of the following is the most likely diagnosis?

Guillain-Barre syndrome Guillain-Barre is an acute inflammatory demyelinating disorder characterized by rapidly progressive distal-to- proximal muscle weakness that is generally first noticed by gait abnormalities and can progress to affect respiration and swallowing.

A 45-year-old female has noted increasing bronze pigmentation of the skin even though she has not been out in the sun. She is found to have fasting serum glucose of 160 mg/dl. Physical examination reveals a palpable spleen tip and a firm liver edge. Arrhythmias are noted on electrocardiogram. The best explanation for these findings is:

Hemochromatosis

A patient has been followed for 3 years with a continual decline in glomerular filtration rate (GFR). Currently the GFR is 10 ml/min and examination of the patient reveals a pericardial friction rub. Which of the following is the most appropriate intervention at this time?

Hemodialysis

A patient presents to the office for routine examination. Initial labs reveal a Hgb of 8.5 mg/dL and Hct of 26%, MCV 66 fL, and RDW 12.1. Peripheral blood smear shows numerous target cells. Based upon these findings what would be the next laboratory test to order?

Hemoglobin electrophoresis The findings of target cells on the peripheral blood smear indicate that a hemoglobinopathy exists. To confirm what type of hemoglobinopathy exists an electrophoresis should be done.

This sign is indicative of _____ infection [with a photo of man with vesicles on his nose tip.

Herpes

A 32-year-old previously healthy man is brought to the emergency room after having a seizure. He has no family history of seizures and denies alcohol use, illicit drug use or trauma. A family member states that recently the patient has been c/o a headache and has been acting bizarre, which is a change in his personality. PE Temp 101 F. BP & HR are normal. During examination the patient has a partial complex seizure. CT scan of the head reveals hemorrhagic necrosis of the temporal lobes. Which of the following is the most likely diagnosis?

Herpes Encephalitis HSV 1-most common cause of acute sporadic encephalitis. Arboviruses-responsible for both sporadic and endemic cases. Enteroviruses, mumps, EBV, and VZV also common causes Patients present with subacute course; personality changes, fever, headaches, and seizures Temporal lobes primarily affected; Fatal without treatment Treatment IV acyclovir(10/mg/kg q 8hr) PCR-confirmed should receive 14-day course of therapy CSF PCR testing for HSV sufficiently sensitive- negative result excludes the dx. of HSV-encephalitis and allows discontinuation of acyclovir

A 68 year-old female presents to your office with a 3 day history of pain around her right rib cage. She denies chest pain, shortness of breath, palpitations, recent injury or trauma. Her past medical history is significant for rheumatoid arthritis for which she has used Etanercept (Enbrel) weekly for 4 years. Physical examination is unremarkable. She returns the following day with grouped, vesicular eruption along the T5 dermatome. Which of the following is the most likely diagnosis?

Herpes Zoster This is an immune compromised patient with classic prodromal shingles symptoms followed by vesicular, dermatomal rash.

Which of the following tests is most commonly used to confirm the presence of acute Epstein -Barr virus?

Heterophile test

An 18 year-old female presents to the office with fever, fatigue and sore throat. Physical examination reveals an erythematous pharynx, cervical lymphadenopathy and splenomegaly. CBC reveals an increased white blood cell count with atypical lymphocytes, normal hemoglobin and hematocrit and normal platelet count. What additional laboratory test will help you make the diagnosis in this patient?

Heterophile test The heterophile test is used for the diagnosis of infectious mononucleosis in children and adults

A 22-year-old gay man from New Orleans presents with a 2-week history of fever, anorexia, and progressive diffuse lymphadenopathy. Physical findings reveal an emaciated young man who has several tongue ulcers. Hepatomegaly is noted. Laboratory examination reveals pancytopenia, an elevated alkaline phosphatase, and hyperkalemia. A chest radiograph reveals a miliary pattern of diffuse infiltration. A tongue biopsy reveals the presence of hyphae that bear both large and small spores. The correct diagnosis is

Histoplasmosis Clinical Manifestations Usually asymptomatic or mild symptoms Cough, fever, malaise Erythema nodusum and erythema multiforme have been reported CXR -hilar adenopathy, with or without areas of pneumonitis Following resolution, multiple small calcified granulomas may be seen on CXR Treatment Acute pulmonary histoplasmosis Usually requires no Rx Oral itraconazole (200 mg/d) may shorten illness Disseminated or chronic fibronodular pulmonary histoplasmosis IV amphotericin B for the initial Rx of severely ill or immunosuppressed or who have CNS infectionRegimen can be changed to itraconazole (200 mg twice daily) once clinical improvement for 6 to 12 months. AIDS patients after acute Rx. should receive itraconazole (200 mg/d) for life

Which personality disorder is characterized by excessive attention seeking, exaggerated emotional response, seductive, uninhibited and lively behaviors, that are frequently superficial and fleeting?

Histrionic Personality Disorder

A 40-year-old patient presents with fever, night sweats, and a 3 cm, nontender lymph node in the neck. Chest x-ray shows mediastinal adenopathy. The most likely diagnosis is

Hodgkin's disease Hodgkin's disease typically presents with nontender lymphadenopathy in the neck and axilla and B-symptoms. Mediastinal adenopathy is also common.

Which of the following non-pulmonary physical exam finding would be most likely in a patient with a large, apical lung cancer?

Horner syndrome Horner syndrome, typified by a constricted pupil, partial ptosis and loss of sweating is associated with apical lung cancers

A 72 year-old patient with type 2 diabetes treated with a sulfonylurea is brought into the clinic by her daughter, who provides the patient's history. The patient has had a recent upper respiratory infection and excessive diuresis for the past 2 days. She has had decreased oral intake. At the clinic, she becomes increasingly stuporous and lethargic. Which of the following is the most likely diagnosis in this patient?

Hyperglycemic hyperosmolar state Hyperglycemic hyperosmolar state is most common in an elderly patient with mild diabetes. Lethargy and confusion develop as osmolality rises to 300 mosm/kg or higher.

A 30 year-old female with diabetes mellitus type 1 presents for her annual exam with complaints of intermittent nausea with occasional vomiting, diarrhea and abdominal pain worsening over the past six months. She states she has been irritable lately and feels anxious today. Her menses ceased two months ago. On physical examination she is mildly hypotensive. Urine pregnancy test is negative. Which of the following are you also likely to find on her physical examination?

Hyperpigmentation over the knuckles The patient exhibits signs and symptoms consistent with Addison disease (chronic adrenal insufficiency) which can be a complication of diabetes. Skin examination tends to show hyperpigmentation in areas of non-sun- exposed skin, as well as sun-exposed areas. This effect is particularly clear in areas of skin creasing such as knuckles, elbows, and posterior neck

A15-year-old male has anterior chest pressure radiating into his left arm while playing soccer. The pressure ends in 5 minutes with rest. BP is 145/95, pulse is 90 and regular, and respirations are 20. Exam shows a rapid carotid pulse upstroke, double apical impulse, and 2/6 SEM in the 4th ICS along the LSB that radiates to the apex. Which of the following is the most likely diagnosis?

Hypertrophic obstructive cardiomyopathy Etiologies Frequently congenital, or familial Autosomal dominant with a high degree of penetrance Pathophysiology Hypertrophy of IVS results in decreased LV outflow with or without obstruction Systolic function is normal Increased LV stiffness results in elevated diastolic filling pressures Prototype of "diastolic dysfunction" Symptoms secondary to Elevated diastolic pressure Dynamic outflow obstruction Arrhythmias Clinical Presentation DOE Angina Pectoris Presyncope and syncope especially during exercise Ventricular arrhythmias are common and sudden cardiac death may occur, especially in athletes after extreme exertion MI

A patient complains of "pins and needles" sensations in his hands and face, frequent painful muscle spasms and muscle twitching. A metabolic disturbance that typically causes such symptoms is:

Hypocalcemia

A 17 year-old girl presents with complaints of being fat and tired, and that her periods are irregular. She says she has been dieting by limiting caloric intake, and typically runs six miles a day, but lately even five miles makes her very tired. On exam, height is 5'5", weight 92 lbs, temperature 96 degrees F, pulse 48, respiratory rate 18, and BP 88/58 mm Hg. Skin is dry and scaly. Remainder of the exam is unremarkable. Which of the following diagnostic studies would be consistent with your diagnosis?

Hypokalemia Routine laboratory studies in anorexia nervosa show decreased red blood cells, white blood cells, platelets, potassium, magnesium, thiamine levels, and serum albumin. BUN, AST, ALT, LDH, and beta-carotene levels may be elevated.

A 17 year-old female presents with complaints of intermittent abdominal pain associated with frequent episodes of regurgitation of food for the past several months and worsening over the past 12 hours. She maintains a normal weight for her height; however, she seems obsessed with losing weight. On examination she has multiple dental caries, bilateral tenderness of the parotid glands and mild epigastric tenderness. Which of the following findings would you expect to find on laboratory tests to support your suspected diagnosis?

Hypokalemia This patient most likely has bulimia nervosa - purging type. Self-induced vomiting is the most common method of purging and this is supported by the physical examination findings noted in this patient. Laboratory findings to support this diagnosis include hypochloremia with subsequent hypokalemia due to renal compensatory mechanisms, hypomagnesemia and metabolic alkalosis.

A 55- year-old male presents with several months of dyspnea and a nonproductive cough. Physical examination reveals dry crackles at both lung bases. Chest radiography and high resolution CT of the chest reveal a bibasilar reticular nodular pattern in the lung field. Spirometry reveals reduction in total lung capacity, vital capacity, and residual volume. The carbon monoxide diffusion capacity is reduced to 35 percent of normal. Resting arterial hypoxemia is demonstrated on arterial blood gas testing. Transbronchial biopsy results reveal an increase in inflammatory cells on the alveolar surface, predominantly macrophages, as well as diffuse intraalveolar fibrosis. The most likely diagnosis is:

Idiopathic pulmonary fibrosis

A 22 year-old man collapses on the baseball field after he is hit in the chest by a fastball. He is unresponsive and does not have a pulse. CPR is initiated, EMS arrives quickly, & a rhythm strip shows. Which of the following is the best intervention now? EKG shows V-Fib

Immediate unsynchronized cardioversion 200J (biphasic)/defibrillation

A 43 year-old female calls the office stating that her husband was recently diagnosed with Hepatitis A. She denies a past history of or immunization for this disease and specifically denies recent jaundice, fever, nausea, or abdominal pain. Which of the following is the most appropriate next step in the care of this patient?

Immune globulin Immune globulin is indicated in close contacts of patients with Hepatitis A who have not previously had the disease or been immunized against it.

A patient states that he has been camping in the mountains of North Carolina for the past two weeks. He presents to the clinic complaining of "flu-like" symptoms for the past 10-14 days however he notes that he started to develop a slight rash on his wrist and ankles about seven days ago. Which of the following tests would provide a confirmatory diagnosis?

Immunofluorescent assay The immunofluorescent assay will confirm antibodies to Rickettsia

Which of the following is the pathophysiological mechanism by which Crohn's disease may cause development of a megaloblastic, macrocytic anemia?

Impaired incorporation of folate into nucleic acid

A 62-year-old male survives an episode of VF. There is no evidence of a AMI although the patient has had one previous MI, 3 years ago. EF on echocardiogram is 45% What is the recommended Rx for this patient?

Implantable cardioverter-defibrillator(ICD)

Which of the following has been associated with an effective approach towards the prevention of diabetic retinopathy?

Improved control of blood glucose concentrations Preventing diabetic retinopathy is the most effective approach toward the preservation of vision.

A 37 year-old obese, otherwise healthy female presents with complaints of an enlarging painful, red mass on her left medial thigh. She had tried over-the-counter hydrocortisone cream and triple antibiotic ointment without improvement. Examination reveals a 4 cm, warm, tender, fluctuant mass with surrounding erythema and no inguinal lymphadenopathy. Which of the following is the most appropriate clinical intervention for this patient?

Incision and drainage, wound culture and oral antibiotic therapy The patient has a carbuncle with early cellulitis. The mainstay of treatment is incision and drainage with wound culture followed by administration of oral antibiotics, primarily for the early cellulitis.

A 58 year-old female comes to the office because she noticed a white spot on her tongue. She has a 42 pack/year smoking history. On examination, she has an 8 mm white thickened, keratinized lesion on the ventral tongue. The lesion cannot be removed by rubbing the mucosal surface. Which of the following studies is most appropriate to confirm the suspected diagnosis?

Incisional biopsy Leukoplakia is characterized as a white plaque that cannot be removed by wiping the mucosal surface. Tobacco use is strongly associated with an increased incidence of leukoplakia and oral cancer. An incisional biopsy of the lesion is required for histologic evaluation of the tissue

A 66 year-old female with a history of DM and HTN presents for routine evaluation. Since beginning her ACE inhibitor and diuretic therapy her BP have averaged 138/85 mmHg. Which of the following is the next best step in the management of this patient?

Increase ACE inhibitor to achieve blood pressure of less than 130/80 mmHg. (c)JNC VII guidelines set target blood pressure for patients with diabetes or chronic kidney disease at less than 130/80.

A 48 year-old male with a recent episode of nephrolithiasis comes to the office to discuss prevention of further urinary stone disease. Which of the following dietary recommendations can reduce recurrence of nephrolithiasis? A. Increase fluid intake B. Decrease dietary fiber C. Increase salt intake D. Decrease calcium intake

Increase fluid intake

A patient with adrenal insufficiency is taking hydrocortisone 25 mg daily. What should the patient do with the hydrocortisone dose when they develop a minor illness such as a cold?

Increase the dose to 50 mg daily until the illness resolves To better mimic the normal physiologic response the baseline dose should be doubled for the duration of the illness. Doses should be increased 5-10 fold with major events such as surgery

A 55 year-old male with Class D NYHA stage IV HF with a severely depressed EF is brought to the ER via ambulance and is difficult to arouse, diaphoretic, and has had no U/O for the last 24 hours. He is hypotensive (BP 80/palp) and tachycardic (125/min). PE reveals elevated JVP to the angle of the mandible, S1, S2 +S3 heart sound, pulmonary rales throughout auscultation of the lungs, and 4+ b/l lower extremity edema. Which of the following would best describe the hemodynamic profile in this patient?

Increased pulmonary capillary wedge pressure

A 30 year-old male presents with sudden onset of chills, fever, chest pain and a cough productive of greenish-brown sputum. On examination his temperature is 102 degrees F. He appears acutely ill and his respirations are shallow. Chest x-ray demonstrates left lower lobe consolidation. Which of the following findings would most likely be present on examination of his left lower lung?

Increased tactile fremitus Increased tactile fremitus occurs in the presence of fluid or a lung consolidation such as lobar pneumonia.

When performing a pre-participation sports PE in adolescent population, a murmur with which of the following qualities indicates a risk for sudden death during exercise

Increases with the Valsalva maneuver Hypertrophic cardiomyopathy (HOCM) is a known cause of sudden death during or just after physical exertion and competitive sports.

A 24-year-old man with a recent history of a viral illness comes to the ED complaining of severe left-sided chest discomfort, which radiates through to the left trapezius region. On coming into the room, you note that he is sitting up and hunched forward. On PE , the patient's T 39°C, BP 135/78, with HR of 85, and a pericardial friction rub is noted. Laboratory findings demonstrate elevated serum creatine kinase levels and normal serial troponin levels. His EKG demonstrates diffuse ST segment elevation in the precordial leads. His CXR demonstrates no acute process. Which of the following is the most appropriate treatment for this patient?

Indomethacin

A 44 year-old male presents as a new patient. He has a history of hypertension, rheumatoid arthritis, polycystic kidney disease, and seasonal allergies. On review of his current medications, which of the following agents would you consider removing or replacing?

Indomethacin (Indocin)

A 27 year-old male presents with gradually worsening low back pain and stiffness for the past two years. His symptoms are worse upon awakening and gradually improve throughout the day. Lumbosacral flexion is less than 50%. Lumbosacral spine films show erosions in the joint line of both sacroiliac joints. HLA-B27 test is positive. Which of the following is the most appropriate first-line medication for this patient?

Indomethacin (Indocin) NSAIDs, such as indomethacin, are the mainstay of therapy in ankylosing spondylitis (AS).

A contact lens wearer presents with increasing pain, discharge, and decreasing vision in the left eye. What is the correct diagnosis?

Infected ulcer

Hypoglycemia in an elderly patient with diabetes is most likely to manifest as which of the following symptoms?

Mental confusion Manifestations of hypoglycemia in the elderly are mainly from impaired central nervous system function. Manifestations include mental confusion, bizarre behavior, and ultimately coma.

Appropriate action (of giant cell arteritis) includes

Initiating high-dose glucocorticoid therapy and referring the patient for a temporal artery biopsy Treatment Trial of corticosteroid therapy, if successful, may provide a presumptive diagnosis in lieu of biopsy.

Which of the following histories is most consistent with rheumatoid arthritis (RA)?

Insidious onset, morning stiffness, symmetrical joint swelling and tenderness Rheumatoid arthritis has an insidious onset, morning stiffness is typically longer than 30 minutes, and is typically symmetrical in presentation.

A 42-YO Asian woman immigrated to the U.S. and adopted an American diet. She now complains of bloating, abdominal cramping, and excess flatulence. She denies any weight loss. All stool cultures are negative. Fecal leukocytes are negative. Which of the following is the most likely diagnosis?

Lactose intolerance

A cirrhotic patient presents with progressive drowsiness and delirium. Physical examination reveals asterixis and tremor. Which of the following medications would you initiate?

Lactulose (Kristalose) This patient has hepatic encephalopathy. Increased ammonia levels contribute to the mental status changes associated with hepatic encephalopathy. Lactulose leads to a change in bowel flora so that fewer ammonia forming organisms are present and also helps in the acidification of colon contents which leads to a nonabsorbable ammonium ion creation.

A 35-year-old factory worker presents to your office for evaluation of pain in the left arm. He denies any acute trauma or injury. His job involves inspecting jars, and he has to test the opening and closing of jar lids. He denies fever or chills. On physical examination, there is no swelling over the elbow. You palpate the olecranon process, and he has tenderness on the left lateral epicondyle but not the medial epicondyle. Based on this information, what is your most likely diagnosis?

Lateral epicondylitis Inflammatory condition affecting the origin of wrist extensors. Overuse injuries and microtrauma Local pain at lateral, extensor origin, aggravated by continual use. Degeneration of extensor/flexor tendons X-rays negative TX NSAIDS, modification, rest, ice, stretching, strengthening, injections, strapping Positive "tennis elbow" test Pain with resisted wrist extension

A 19 year-old male presents to the ED complaining of a sudden onset of dyspnea and left sided chest pain. He denies fever, chills, cough or sore throat. General survey shows that he is 6 feet 2 inches tall and weighs 135 lbs. Vital signs are BP 86/60 mmHg, HR 130 bpm, RR 28, temp. 98.6 degrees F. Which of the following would you likely find on examination of his thorax?

Left-sided hyperresonance Hyperresonance is present with accumulation of air in the pleural space.

A 55 year-old female presents to the emergency department with complaints of dyspnea, chest pain and coughing with hemoptysis. Past medical history includes breast cancer 5 years ago, currently in remission. Vital signs are Temp. 98.6 degrees F, BP 150/90 mmHg, P 110 bpm, RR 20. Physical examination shows her right leg swollen with pain on palpation of deep veins. Which of the patient's history or examination findings is most suggestive of a pulmonary embolus (PE)?

Leg swelling and pain with palpation of deep veins Leg swelling and pain with palpation of the deep veins are consistent with a DVT and increase the likelihood of a PE.

A 35-year old-male patient has chronic renal failure. He had a renal transplant 6 months ago and is being treated with corticosteroids and cyclosproine. He came to the Emergency Department 4 days ago with fever, dyspnea, and 2 days of diarrhea. The chest x-ray shows and a diffuse interstitial infiltrate. He has failed to respond to ceftriaxone and continues with a fever and a cough. Today he is lethargic and confused. What is the most likely organism causing his pneumonia?

Legionella pneumonia

This 16 year-old male came to the emergency room complaining of severe abdominal pain in the right lower quadrant. He had a fever of 101 F. On physical examination, he had a rigid, board-like abdomen and rebound tenderness in the right lower quadrant. Bowel sounds were reduced. CBC showed Hgb 14.8 g/dL, WBC 21,800. and platelet count 240,000. Urinalysis was within normal limits This is most consistent with:

Leukemoid reaction Extreme elevation of leukocyte count( can be > 50,000/uL) composed of mature & /or immature neutrophils

A 58 year-old male presents with a 4 month history of instability upon ambulation. Examination reveals resistance to passive movements, mild tremor of the lips and a shuffling gait. Which of the following medications is most appropriate for this patient?

Levadopa/carbidopa (Sinemet) Levodopa (L-DOPA) = metabolic precursor of Dopamine. Works with most people especially for bradykinesia. Need a dopa decarboylaxe inhibitor (carbidopa) as L-DOPA will be metabolized otherwise. Sinemet is a combination of this inhibitor and Levodopa

A 35-year-old female presents to your office for follow up after treatment for pneumonia that was treated with antibiotic therapy 2 weeks ago. She now states that she has a frontal headache, postnasal drip, and bad breath. She has tried a decongestant, without relief. On PE, she has tenderness over her maxillary and frontal sinuses. The sinuses do not transilluminate. What antibiotic would you prescribe in this patient?

Levofloxacin

A 30 year-old patient is seen for a non-painful mass on the upper back which has grown slowly over the past year. He denies previous trauma, drainage from the area or history of dermatologic diseases. Examination reveals a four centimeter firm, but highly mobile subcutaneous mass with no overlying skin discoloration or punctum with drainage. Which of the following is the most likely diagnosis?

Lipoma A lipoma is a benign fatty tumor usually developing slowly in the subcutaneous layer and generally remaining small though can become very large.

A 36-year-old man twisted his knee while playing rugby. Physical exam discloses a positive posterior draw test and ballottement sign. Radiographs of the knee are normal. The most appropriate next step is:

Magnetic resonance imaging scan(MRI) PE findings in this patient suggest a torn posterior cruciate ligament and possible damage to his meniscus. Initial radiographic evaluation should include a MRI scan which can visualize cartilage and to a lesser degree ligament damage.

A 23 year-old Peace Corps volunteer returned from Central America 2 weeks ago. She presents with a week long history of intermittent headache and myalgias. Today she is experiencing shaking chills followed by a temperature of103.5 F and diaphoresis. She reports she had similar symptoms 2 days ago which she thought was the flu. Which of the following is the most likely diagnosis?

Malaria Malaria affects patients who live or travel to endemic areas. Classic presentation includes intermittent attacks of chills, fever and sweating. Patients can also experience flu-like symptoms, headache, myalgias and/or nausea and vomiting.

Which one of the following techniques is used to detect a torn meniscus?

McMurray Sign This test is performed with the patient in a supine position on the table, with the practitioner holding the proximal tibia in one hand, and the distal femur in the other. A figure eight motion is made, putting medial and lateral pressure against the two menisci. Clicking, locking, and pain indicate a tear.

Which of the following is the best strategy for screening for acute pancreatitis?

Measurement of both serum amylase and serum lipase Though widely used as a screening test to rule out acute pancreatitis in a patient with acute abdominal or back pain, only about 85% of patients with acute pancreatitis have an elevated serum amylase level. Confounding issues include delay between symptoms and obtaining blood samples, the presence of chronic pancreatitis, and hypertriglyceridemia, which can falsely lower levels of both amylase and lipase. Because the serum amylase level may be elevated in other conditions such as renal insufficiency, salivary gland lesions, tumors, burns, and diabetic ketoacidosis as well as in other abdominal diseases such as intestinal obstruction or peritonitis, amylase isoenzyme levels have been used to distinguish among these possibilities. Therefore, the pancreatic isoenzyme level can be used to diagnose acute pancreatitis more specifically in the setting of a confounding condition. The serum lipase assay is less subject to confounding variables. However, the sensitivity of the serum lipase level for acute pancreatitis may be as low as 70%. Therefore, the recommended screening test for acute pancreatitis is both serum amylase and serum lipase activities.

A 45 year-old female secretary presents with complaints of numbness, tingling, and pain in the thumb, index, and third fingers. On examination the patient experiences numbness and tingling while holding her wrists in a flexed position for 60 seconds. This suggests compression of which of the following nerves?

Median Carpal tunnel syndrome results from compression of the median nerve that innervates the first three digits. It is associated with pregnancy, amyloidosis, flexor tenosynovitis, overuse phenomenon, acute or chronic inflammatory conditions, traumatic disorders of the wrist, diabetes mellitus, hypothyroidism, and tumors within the carpal tunnel.

Patients with solid tumors who develop nephrotic syndrome usually have which form of glomerular disease?

Membranoproliferative glomerulonephritis

A patient is 2 days s/p bilateral ureterosigmoidostomy for bladder resection due to cancer. He complains of increasing shortness of breath. The patient denies cough, chest pain or fever. Physical examination is unremarkable except for an increased respiratory rate of 30/minute. Labs include Na 132 mEq/L, K 5.6 mEq/L, and Cl 127 mEq/L. Arterial blood gas reveals pH 7.28, PO2 98mmHg, PCO2 22 mmHg, and HCO3 13mEq/L. What is the most likely acid-base status?

Metabolic acidosis

What is the most likely acid-base disturbance according to the ABG given below? ABG = pH- 7.51, pCO2- 45, HCO3- 32

Metabolic alkalosis

A 43 year-old female presents with a two year history of frequent episodes of pain and morning stiffness in both hands and wrists. She experiences some symptomatic relief with ibuprofen, but feels that the episodes are becoming more frequent and severe. On examination, you observe joint swelling of several MCP joints on both hands. X-ray of the hands shows joint space narrowing of the MCP joints. In addition to NSAIDs, What is the most appropriate first- line long-term medication to treat this patient?

Methotrexate (Rheumatrex) Methotrexate is the standard first-line medication in the treatment of rheumatoid arthritis. This DMARD is generally effective, well-tolerated, and affordable

A 56 year-old diabetic male with a three-month history of epigastric pain after eating says he gets full really fast now. His test for Helicobacter pylori is negative. Which of the following is the recommended medication to treat this patient?

Metoclopramide (Reglan) Metoclopramide accelerates gastric emptying and is the treatment of choice for this patient.

A patient was treated for community acquired pneumonia with amoxicillin-clavulanate (Augmentin). On day 7 of therapy he develops fulminate diarrhea. The diarrhea is described as greenish and foul-smelling. He admits to associated abdominal cramps. Which of the following is the treatment of choice for this patient?

Metronidazole (Flagyl) Patients with C. difficile colitis should be treated with Flagyl for 10-14 days following cessation of the diarrhea-inducing antibiotics.

A 26-year-old female comes into the ER with complaints of dyspnea on exertion. On further questioning, she gives a history of increasing cough over the last week and limitation of her daily activities. She also gives a history of breathlessness at night after going to bed. Her PMH is significant for a bout of RF at age 15. Examination shows a female in distress. There is peripheral and facial cyanosis. There are prominent 'a' waves in the JVP . Palpation yields a diastolic thrill at the apex in the left lateral position. Auscultation shows the presence of a mid-diastolic murmur best heard in the mitral area. What condition does this patient most likely have ?

Mitral stenosis In normal adults, MV area is between 4 to 6 cm2. In severe cases, the valve area reduces to about 1 cm. In such cases, more pressure (about 25 mmHg) is required to maintain CO This elevated LA pressure increases the pulmonary venous pressure, leading to decreased pulmonary compliance. This ultimately leads to dyspnea on exertion and cough. Due to decreased CO , there is cyanosis of the periphery and face. Increased RA systole due to PHTN leads to prominent 'a' waves on the JVP Diastolic thrill on palpation is also due to the increased force of beating following the heart to maintain CO Mid-diastolic murmur is due to the turbulent flow of blood across the stenosed MV . Any valve area greater than 4-6 cm2 is more than the normal and hence does not suggest MS 2D and Doppler echocardiography are the investigations that can reveal morphology of MV precision. TEE, although not always necessary, can be used to detect pathologies of MV such as vegetation's.

A radiograph reveals a fracture of the proximal third of the ulnar diaphysis with an anterior dislocation of the radial head. Which of the following fractures does this describe?

Monteggia fracture A type I Monteggia fracture is a fracture of the proximal third of the ulnar diaphysis with anterior angulation or anterior dislocation of the radial head

A 61-year-old male gives a history of back pain for several months. A radiograph of the spine reveals several 1 to 2 cm lytic lesions of the vertebral bodies. He has no lymphadenopathy or splenomegaly. His serum urea nitrogen and creatinine are steadily rising. He recently developed a cough productive of yellowish sputum, and a sputum culture grew Streptococcus pneumoniae. During the course of his hospitalization, a bone marrow biopsy is performed, and the marrow smear is shown here The most likely diagnosis is:

Multiple myeloma

A 21 year-old college student presents with a 3 week history of slowly worsening dry cough, generalized fatigue and most recently low-grade fevers. He denies nasal congestion, sore throat and nausea and has no past history of pulmonary disease or tobacco use. He does note that many other dorm residents have had similar symptoms over the past two months. Examination reveals mild inspiratory crackles but is otherwise normal. Chest radiograph is clear and CBC is normal. Which of the following is the most likely causative organism?

Mycoplasma The subacute onset in this demographic of patient along with the lack of more advanced findings on exam and chest radiograph points most strongly to Mycoplasma. The other organisms listed have a generally more acute and severe onset and are associated with patients who have complications or other comorbid conditions.

A 72-year-old man who has become progressively more fatigued is found to be anemic. Hematological laboratory values are as follows: Hemoglobin: 10 g/dLHematocrit: 27.5 %Mean corpuscular volume(MCV): 101 fLMean corpuscular hemoglobin(MCH): 30 pgMean corpuscular hemoglobin concentration(MCHC): 34 g/dLReticulocyte count: 0.5 %White blood cell count (WBC): 7300/LPlatelet count: 210,000/L The most likely diagnosis is:

Myelodysplastic syndrome A slightly increased mean corpuscular volume and an inappropriately low reticulocyte count are characteristic of a macrocytic, hypoproliferative anemia. A macrocytic, hypoproliferative anemia in the older man described in the question would most likely be due to a myelodysplastic syndrome. A bone marrow examination with iron stain would be required to define the precise subtype of this heterogeneous disorder, which is characterized by a stem cell defect leading to disordered hematopoietic maturation. Given the normal platelet count and white blood cell count, either refractory anemia or refractory anemia with ringed sideroblasts is the most likely subtype

A-40-year-old man presents to the emergency room complaining of severe abdominal pain that radiates to his back accompanied by several episodes of vomiting. He drinks a quart of Vodka daily. On physical examination, the patient is found on the stretcher lying in the fetal position. He is febrile and appears acutely ill. The skin of his abdomen has an area of bluish periumbilical discoloration. There is no flank discoloration. Abdominal examination reveals decreased bowel sounds. The patient has severe midepigastric tenderness on palpation and complains of exquisite pain when your hands are abruptly withdrawn from his abdomen. Rectal examination is normal. Which of the following is the most likely diagnosis?

Necrotizing pancreatitis On PE a Grey-Turner sign is present (focal areas of discoloration about the umbilicus and in the region of the loins, in acute hemorrhagic pancreatitis and other causes of retroperitoneal hemorrhage). Most common causes: Alcohol & Gallstones/Biliary Pancreatitis Epigastric pain radiating to the back Elevated Amylase and Lipase (Lipase remains elevated longer) Treatment: IV FLUIDS, analgesics (Demerol does not constrict the sphincter of Oddi like morphine), and bowel rest

A 54 year-old man comes to the urgent care because he was awoken suddenly from his sleep this morning with severe left flank pain radiating to his left testicle with associated nausea and vomiting. He is afebrile and vital signs are normal. He is constantly moving to find a comfortable position. On physical examination, left flank tenderness is noted with no direct testicular tenderness. Urinalysis reveals a pH of 5.5 and microscopic hematuria, but is otherwise unremarkable. Which of the following is the most likely diagnosis? A. Acute epididymitis B. Appendicitis C. Diverticulitis D. Nephrolithiasis

Nephrolithiasis

An elderly man who recently emigrated from a war-torn area of Africa is brought to the clinic by his daughter. She explains that her father's diet was very limited in calories and protein and that he mostly ate corn and very little fresh foods. He has chronic diarrhea and examination reveals pigmented regions on hands, arms, and face and mild dementia. His tongue is smooth and shiny. This patient most likely has a deficiency of which of following vitamins?

Niacin This patient's syndrome known as pellagra is due to niacin (vitamin B3) deficiency. It is often seen in people where corn is the major source of energy and is still endemic in parts of Africa. The syndrome includes glossitis, pigmented dermatitis, dementia, and diarrhea.

A patient with known pemphigus vulgaris presents to the office for evaluation after skin trauma. You note the superficial detachment of the skin in the area of trauma. This physical exam finding is known as which of the following?

Nikolsy sign Nikolsky sign is the superficial detachment of the skin after pressure or trauma to the affected area as seen in pemphigus

A 26 year-old woman comes to the office for her first prenatal visit at 9 weeks gestation. During evaluation, her urinalysis reveals asymptomatic bacteriuria. Which of the following antibiotics is the preferred treatment in this patient?

Nitrofurantoin (Macrobid)

A 60-year-old male presents with substernal chest tightness after mowing the lawn. He also noted mild dyspnea and diaphoresis. All his symptoms resolved in ten minutes. Vitals and PE are normal. EKG reveals normal sinus rhythm with no acute changes. Which of the following is the most appropriate treatment for this patient?

Nitroglycerine (Nitro) Stable angina presents with substernal chest pain, heaviness, dyspnea, diaphoresis, and nausea that resolves within 15 to 30 minutes. EKG is typically normal. Initial treatment is nitroglycerine to increase coronary blood flow and provide pain relief.

An ambulance transports a patient to your facility from the nursing home with anuria and diminished LOC . Paramedics report a BP of 225/130. Which of the following medications is most appropriate for this patient?

Nitroprusside sodium (Nipride) Parenteral therapy is indicated in most hypertensive emergencies, especially if encephalopathy is present. Nitroprusside lowers blood pressure in seconds by direct arteriolar and venous dilation.

A 54-year-old male presents to your office complaining of daytime sleepiness. He states that he invariably naps after dinner and normally is in bed for 8 to 10 h nightly. He does report, however, that he feels that his sleep is fragmented. When the patient awakens in the morning, he rarely feels refreshed. His wife has complained for years of his snoring to the extent that she now sleeps in a different room. The patient notes that he frequently has difficulty staying awake during the late afternoon at work. He is seeking evaluation now because his job performance has been impaired and he has been put on probation at work. PMH is significant for hypertension since age 38 that has recently worsened despite medication and non-insulin-dependent diabetes. BMI is 36 kg/m2. He is noted to have central obesity and a thick neck . Examination of the oropharynx is normal. Based on the patient's presentation, what is the most likely diagnosis?

Obstructive sleep apnea

A 59-year-old man complains of severe chest pain that radiates to his back. His brachial pulses appear unequal between his right and left arms. He appears hemodynamically stable. On CXR, he has widened mediastinum. Which of the following is the best next step?

Obtain CT of chest with IV contrast This clinical presentation of severe chest pain radiating to the back, unequal brachial BPs, and a widened mediastinum on CXR is consistent with acute aortic dissection. A CT scan of the chest is a quick imaging test to confirm the acute dissection. Thrombolytic therapy or anticoagulation can worsen the process

A 7-year-old is brought to the ED after sustaining a fall onto his outstretched hand. He complains of pain involving the "entire arm" and refused to move his arm which is held in anatomical position with the elbow flexed at 90 degrees. On PE, there is tenderness to palpation over the elbow with associated swelling and pain on attempted rotation. There is no apparent tenderness to palpation involving the wrist or shoulder, but the child will not allow evaluation of range of motion. X-rays reveal the presence of a positive posterior fat pad sign(R). The most likely diagnosis is

Occult fracture of the radial head Most common elbow fracture Fall on the outstretched arm typical presentation, inability to extend the elbow Look for a radial head fracture, or "fat pad sign" Even though a fracture may not be visible on the x-ray, soft tissue findings correlate well with a radial head fracture. Treat with a posterior splint, or simple sling.

The characteristic early finding associated with carcinoma of the cecum is:

Occult gastrointestinal bleeding Cecal carcinomas can become very large before they become symptomatic. They rarely cause obstruction. The most characteristic early finding is occult GI bleeding which may cause a Fe deficiency anemia.

A 55-year-old male smoker presents with burning epigastric pain several hours after a meal, which is relieved by antacids. His PA determines that the stool guaiac is positive. UGI endoscopy discloses an ulcer with a well-demarcated border at the duodenal bulb. He undergoes upper endoscopy, and the lesion seen here grossly is identified. Histologic examination of a biopsy specimen of the ulcer crater reveals eosinophilic necrosis with surrounding fibrosis without evidence of malignancy. Furthermore, analysis of a histologic section involving the gastric mucosa reveals invasion with a gram-negative rod. Which of the following is the most appropriate therapy?

Omeprazole plus clarithromycin and metronidazole or amoxicillin. This patient has the classic clinical symptoms and endoscopic findings of a duodenal ulcer. The incidence of duodenal ulcers is about 10% of the population of industrialized countries. The pathophysiology of duodenal ulcers includes excess gastric acid secretion; however, H. pylori infection, as documented in this patient, may be playing a critical role. Whatever the mechanism, it is now a consensus recommendation that H. pylori infection should be eradicated in patients with documented peptic ulcer disease.

A 48 year-old male with a history of AIDS presents with complaints of odynophagia and dysphagia for 1 week. The patient denies nausea, vomiting, diarrhea, constipation or melena. Endoscopic evaluation reveals diffuse, linear plaques which are yellowish-white in color and adhere to the esophageal mucosa. Which of the following is the treatment of choice for this patient?

Oral fluconazole (Diflucan) Oral fluconazole is the treatment of choice for a patient with candidal esophagitis.

The mainstay of therapy in this patient (Idiopathic pulmonary fibrosis) is at this time is

Oral prednisone Prednisone 1(mg/Kg) d, for 8-12 weeks, If pt fails to respond to prednisone, immunosuppressive therapy with cyclophosphamide Response assessed by symptoms & PFT's Smoking cessation, O2 (pO2< 55 mmHg) Rx. RHF & bronchospasm may improve symptoms

A 3 year-old presents with profuse watery diarrhea for the past three days. The child vomited twice yesterday, but not today. On examination, the child is febrile, with pulse of 142, respiratory rate of 18, and blood pressure of 60/40 mmHg. The child is alert and responsive, with no focal findings. Which of the following is the most appropriate intervention?

Oral rehydration The goal of therapy for a child with severe gastroenteritis and dehydration is to restore fluid loss. Oral rehydration with an appropriate electrolyte solution is the best option if the child is not actively vomiting and is alert enough to take oral fluids

A 27 year-old presents with acute shortness of breath with chest pain, dizziness, sweating, and nausea that started about a half hour ago. She has had episodes like this in the past, the first of which occurred about six months after her mother died. They resolved with breathing into a paper bag. In bet ween attacks, she feels fine. Which of the following is the most likely diagnosis?

Panic attack Panic attacks are characterized by sudden onset of intense apprehension, fear, or a sense of impending doom, accompanied by at least four somatic symptoms, such as shortness of breath, dizziness, palpitations, sweating, trembling, chest pain, nausea, and others.

Which of the following is the most common thyroid malignancy?

Papillary Carcinoma Almost always manifests as a palpable thyroid nodule Twice as common in females than males, but males have a worse prognosis. Papillary carcinoma is the most common type and is the least aggressive. Anaplastic is the worst! Present with a single, hard nodule that is showing rapid, painless growth.

A 30 yr old woman presents c/o a 1 wk H/O vaginal discharge. She describes the discharge as green-yellow in color with a bad odor. She has never had this type of discharge in the past. The discharge increases after intercourse accompanied by vaginal soreness. She denies pruritus, abdominal pain, nausea, vomiting, fever, chills or sweats. She has been sexually active with 1 male partner for the past 3 months and uses an IUD for contraception. He has no symptoms. She reports a history of multiple sexual partners. She was 15 yrs old when she first had intercourse. LMP was 2 wks ago and was normal. She was treated with oral antibiotics for a chlamydial infection 2 wks ago with resolution of symptoms. VS; BP 108/64; 72; 14; 98.2F Pelvic exam: Normal external genitalia Small amount of frothy, homogenous green-gray discharge at the introitus Cervix - "strawberry" appearance with scant discharge noted in the cervical os IUD string is in place Bimanual exam - no cervical motion tenderness, unremarkable uterus & adnexa Chlamydia & gonorrhea specimens are obtained, vaginal discharge is collected for lab examination What organism is causing this? What is seen on microscopic examination of the discharge? What is the treatment?

Organism: Trichomonas vaginalis Microscopy: Motile, flagellated trichomonads, & many WBCs Treatment: Metronidazole 2 g by mouth in a single dose for the pt and her partner OR Metronidazole 500 mg BID for 1 wk

A 55-year-old executive assistant presents to your office for evaluation of pain in her wrists. She states that the pain has been present intermittently for several months, but over the last 2 weeks, it has been present daily. She has taken over-the-counter analgesics for the pain, which seems to help. She denies fever, chills, or rashes. On physical examination, she has pain and tenderness over the right wrist but not he left. She has a hard dorsolateral nodule over the DIP joint of her right middle finger. The MCP joints are normal. What is your most likely diagnosis?

Osteoarthritis

Focal ulcerated areas of cartilage with bone eburnation, development of bone cysts and the presence of osteophytes best demonstrates the pathophysiology of which of the following?

Osteoarthritis The pathophysiology of osteoarthritis is characterized by focal ulcerated areas of cartilage, bone eburnation, and the development of osteophytes

Which of the following disorders responds most readily to treatment with vitamin D alone?

Osteomalacia Vitamin D deficiency decreases the intestinal absorption of calcium and is the most common cause of osteomalacia. Treatment is vitamin D in various natural and pharmaceutical forms. The addition of supplemental calcium is usually not necessary when treating osteomalacia

A 12 year-old male presents complaining of progressive and worsening nontraumatic left knee and femur pain which began two weeks ago. The patient now refuses to bear weight on the affected limb. Examination reveals pain with passive motion and a temperature of 101 F. Gram stain of a bone aspiration reveals Staphylococcus aureus. Which of the following is the most likely diagnosis?

Osteomyelitis Acute osteomyelitis is characterized by destruction of the bone secondary to an infectious organism, most commonly Staphylococcus aureus in the metaphysis of long bones. Males are affected more frequently than females and present with increasing pain and disuse of the affected limb. Radiologic findings include soft tissue swelling, narrowing or widening of the joint space, bone destruction, and periosteal reaction. The gold standard diagnostic study is a bone aspiration of the lesion revealing the organism.

A 44 year-old female presents to clinic for evaluation of a syncopal episode that occurred while walking her dog two days ago. She denies amnesia or head trauma. She has had increasing DOE and pedal edema. PE reveals clubbing of her fingers and central cyanosis. Auscultation reveals TR, widely split 2nd heart sound with a palpable P2. Echocardiogram reveals a large ostium secundum ASD with bidirectional flow. Which of the following is a secondary complication in this patient?

PHTN Her symptoms and exam findings are consistent with pulmonary hypertension and in her case, Eisenmengers disease, which is a late finding. Eisenmenger's Syndrome ↑PVR resulting in bi-directional flow Redirection of blood flow through VSD, ASD, or PDA from L-to-R to R-to-L shunt Hallmarks: Cyanosis + Pulmonary HTN + Erthyrocytosis Possible complications Hyperviscosity Cerebral hemorrhage Poor blood flow to the brain CHF; Heart attack; Stroke Sudden death

A 55 year old male presents to clinic for "weakness." He looks "depressed." He complains of fatigue and difficulty lifting his arms and some leg weakness when he tries to go up to his upstairs bedroom at night. If he sits for long periods of time, he arises stiff. He states that he has felt feverish over the past few weeks, between 100-102 F orally when he has taken his temperature and admits to some "night sweats" when prompted. He has lost about 10 pounds over the past month and his pants are looking "big" for him. He's just not hungry. He continues to work as a lumber yard manager. Physical exam reveals no discernable weakness on objective neurological exam and an otherwise normal exam except his slow, paced movements indicating fatigue. Lab studies show an elevated erythrocyte sedimentation rate, mild anemia, and an increased alkaline phosphatase. What is the MOST likely diagnosis

PMR Polymyalgia Rheumatica Clinical syndrome characterized by aching & morning stiffness in the shoulder girdle, hip girdle, or neck for > 1 month Elevated ESR, & rapid response to low-dose prednisone(15mg qd) Rarely occurs before age 50 Can occur in association with giant cell arteritis, which requires treatment with higher doses of prednisone Evaluation Careful history to elicit Sx suggestive of giant cell arteritis ESR Labs to R/O other disorders( RF, ANA,CBC, CPK. SPEP) Renal, hepatic, & thyroid function tests Treatment Prednisone 10-20 mg qd but may require treatment over months to years

An 18 month-old male presents with his parents who report symptoms of a barking cough and intermittent stridor that has worsened over the past 12 hours. They note improvement in symptoms when he was taken outdoors to the cool night air. Which of the following is the most likely organism causing this patient's symptoms?

Para-influenza virus Croup is most often caused by parainfluenza virus.

Which of the following organs are affected in multiple endocrine neoplasia (MEN) type I?

Parathyroid, pituitary, pancreas

A 15-year-old boy residing with his parents on a military base presents with a fever of 38.6°C (101.5°F) and complains of lower back, knee, and wrist pain. The arthritis is not localized to any one joint. He gives a history of a severe sore throat several weeks earlier. Physical examination of the skin reveals pea-sized swellings over the elbows and wrists. He also has two serpiginous, erythematous pink areas on the anterior trunk, each about 5 cm in diameter. Laboratory investigation includes negative blood cultures, negative throat culture, normal CBC, and an erythrocyte sedimentation rate (ESR) of 100. An antistreptolysin-O (ASO) titer is elevated. At this point, appropriate therapy would consist of

Parenteral penicillin and aspirin

A 66 y/o female with a history of CAD presents with a new onset of dizziness and fatigue for two weeks. She recalls nearly passing out on one occasion. Examination is unremarkable except for bradycardia. 1616EKG rhythm strip is seen below. (shows 2nd Degree Type II Heart Block). Which of the following interventions is the therapy of choice:

Permanent pacemaker

Which of the following is the treatment of choice for this patient?

Permethrin (Elimite) Treatment: 5% Permethrin lotion or cream applied neck down, leave on 8 hours, wash off, repeat in 7 days Treat close contacts Antihistamines and topical steroids for itching Wash all clothing' bedding All bedding and clothing should be washed and Dried in the DRYER of exposed household members % Lindane can be used: toxic in children <2 y.o., pregnant/ lactating women, extensive open lesions

A patient presents with 3 weeks of worsening pruritic rash located on the upper extremities and interdigital spaces. He is a migrant farm worker with no history of skin disorders. Examination reveals excoriated, erythematous papules with numerous 3-4 mm long, narrow tracts spreading from the papules. What is the best treatment for this patient?

Permethrin (Nix) Permethrin is used to kill the scabies mite.

Which of the followingstatements concerning obstructive sleep apnea syndrome is true?

Personality changes may be the presenting complaint

A 30 year-old diabetic female complains of persistent numbness in her right thumb and forefinger that has been awakening her from sleep for the past week. She is right hand dominant and denies any history of activities involving repetitive motion of the hands. Which of the following is the next step in the evaluation of this patient?

Phalen maneuver The Phalen maneuver is used to reproduce the symptoms of carpal tunnel syndrome by flexion of the wrist.

A fair-skinned, 6-week-old infant who was normal at birth is brought to the pediatrician with a scaly skin rash, irritability, and a mousy odor. . Which of the following is the most likely diagnosis?

Phenylketonuria

Which of the following treatments is used to reduce the hepatic complications of hemochromatosis?

Phlebotomy Hemochromatosis is an autosomal recessive inherited disorder that causes cirrhosis, diabetes, and bronze pigmentation to the skin due to the abnormal accumulation of iron in tissues. Intensive phlebotomy is the treatment of choice until the iron overload is corrected.

A 26-year-old, sexually promiscuous intravenous drug abuser presents with fever and shortness of breath. He complains of dyspnea on exertion and some bilateral pleuritic chest pain. He admits to a recent 30-lb weight loss. On physical examination, heart rate is 124/min, respiratory rate is 28/min, blood pressure is 100/70 mm Hg, and temperature is 102.4 F. Pulse oximetry reveals a saturation of 85% on room air. Kung auscultation reveals scattered bilateral crackles posteriorly. Chest radiograph reveals bilateral interstitial infiltrates and no cardiomegaly. Which of the following is the most likely diagnosis?

Pneumocystis carinni pneumonia Based on the patient's risk factors for HIV. PCP is the most likely diagnosis in this patient, but PCP rarely presents with any physical examination findings that distinguish it from other pneumonias. The chest radiograph may reveal bilateral interstitial infiltrates, and patients are often hypoxemic. CHF may present with a similar chest radiograph, but patients will have JVD and a S3 gallop. CMV, varicella zoster, and Kaposi's sarcoma(due to herpes virus 8) are opportunistic infections seen in immunocompromised patients.

A 31-year-old man presents with fever and arthralgia's for 1 day. He complains of diffuse abdominal pain and inability to move his left foot due to weakness. He also states he has had hematuria for several hours.On PE the patient has a T of 101.2 F. BP 150/95 mm Hg.He has diffuse abdominal tenderness on palpation but has no rebound tenderness. Testicular exam revels marked tenderness of the testes but no urethral discharge. Neurologic exam reveals a left foot drop. Laboratory studies reveal anemia of chronic disease, high ESR, and polymorphonuclear leukocytosis. CXR is clear. . The most likely diagnosis is

Polyarteritis nodosa Life threatening vasculitis of the medium-sized muscular arteries "Classic" form affects medium to small arteries in skin & visceral organ Frequently associated with arteriographic aneurysms Can be associated with Hepatitis B( 50 % of patients) Clinical Manifestations Course waxes & wanes, leading to varied signs & symptoms that can be confusing to diagnose. Most commonly affects young females Unusual findings, such as infarcts in odd places such as liver or testes, should heighten suspicion Kidney, heart, liver, & GI, pancreas, skin, muscle, & brain may be involved Early systemic features include fever, weakness, anorexia, weight loss, myalgias, & arthralgias Pericarditis Laboratory findings + pANCA ; ESR elevated; Anemia of chronic disease, & polymorphonuclear leukocytosis 50 % microaneurysms, demonstrable by angiography Treatment:Most patients improve with corticosteroid &/or cytotoxic drug therapy

A 60-year-old woman has lower-back pain. Radiographic examination reveals diffuse demineralization and a compression fracture of the fourth lumbar vertebra. The serum calcium concentration is 2.8 mmol/L (11.5 mg/ dL). The blood count is normal. This clinical picture is most compatible with the presence of which of the following conditions?

Primary hyperparathyroidism The presenting findings in both primary hyperparathyroidism and multiple myeloma can include hypercalcemia and vertebral compression fractures. The absence of several key features-anemia, elevated erythrocyte sedimentation rate, abnormal serum protein electrophoresis, and Bence Jones proteinuria-is helpful in eliminating the possibility of multiple myeloma. If doubt remains about the diagnosis of myeloma, a marrow aspiration should be performed. The presence of hypercalcemia makes unlikely the diagnoses of osteomalacia, which is associated with hypocalcemia, and osteoporosis and Paget's disease, which are associated with normal blood calcium values.

A 30 year-old pregnant female presents with anxiety, palpitations and weight loss. On examination she is diaphoretic, tachycardic and hyperreflexic. The TSH is lower than normal. Which of the following is the treatment of choice in this patient?

Propylthiouracil (PTU) PTU is favored in the management of hyperthyroidism in a pregnant female due to fewer problems in the newborn.

A 6 year-old male, a recent immigrant from Latin America, is brought to the emergency department with difficulty swallowing and breathing. On physical examination you note a gray-tan pseudomembrane in the back of the pharynx. There is also tender cervical lymphadenopathy. What physical examination finding is most specific for diagnosis in this patient?

Pseudo-membrane in pharynx Diphtheria most commonly presents with sore throat, adherent tonsillar, pharyngeal, or nasal pseudomembrane and low grade fever. The pseudo-membrane is most specific for diphtheria.

A 13 year-old male with known cystic fibrosis presents to the emergency department, accompanied by his parents, with increased coughing, wheezing and low grade fever. Rales are audible on auscultation of the lungs. Treatment should target which of the following organisms?

Pseudomonas aeruginosa Pseudomonas aeruginosa is the most predominant pathogen in patients with cystic fibrosis

A 55-year-old woman complains of stiff, aching hands, especially in the morning. Radiographs of the hands reveal expansion at the base of the terminal phalanges, tapering of the proximal phalanges, and cuplike erosions and bony proliferation of the distal terminal phalanges. This patient most likely has

Psoriatic arthritis There are three main varieties of psoriatic arthritis (an inflammatory arthritis affecting 5 to 42% of patients with psoriasis): asymmetric inflammatory arthritis, symmetric arthritis [more common in women, usually in the hands, similar to rheumatoid arthritis (RA), but without nodules], and psoriatic spondylitis. The pathology is similar to that seen in RA (early neutrophil and later monocytic synovial infiltration). There are no characteristic laboratory abnormalities in patients with psoriatic arthritis, but radiographs, if they reveal features unique to this disease relative to RA, may be diagnostically helpful. Such special features include the pencil-in-cup appearance of the distal terminal phalanx due to cuplike erosions and bony proliferation with tapering of the proximal phalanx; proliferation of the bone near osseous erosions, terminal phalyngeal osteolyis, bone proliferation and periostitis, and telescoping of one bone into another (opera-glass deformity). Inflammation of the tendons and ligaments (ethesopathy) is also characteristic. Nonsteroidal anti-inflammatory agents are the therapeutic mainstays; sulfasalazine or methotrexate is often required in difficult cases

At the urging of his wife a 47 year-old construction worker presents to your office with a skin lesion. He reports he has had a slowly growing nodule on the side of his nose for the past year or two. It recently developed a small central area of erosion. He denies pain, fever, chills, pruritus or other similar lesions. On physical exam the lesion is approximately 1 cm as described above, with a few telangiectatasias. Which of the following is the best diagnostic study for this condition?

Punch biopsy This patient has a basal cell carcinoma (BCC), which is the most common form of skin cancer and occurs in sun exposed areas of the skin. Any lesion suspected of being a BCC should be biopsied with a punch biospy.

A 74 year-old male with a history of coronary artery disease and atrial fibrillation presents to the clinic for follow-up of his shortness of breath. Patient's medications include amiodarone (Cordarone) and metoprolol (Lopressor). His chest x-ray reveals patchy ground-glass infiltrates. Which of the following is the most likely diagnosis?

Pulmonary fibrosis Pulmonary fibrosis presents with ground-glass infiltrates on CXR and is often associated with certain medication use.

A 25-year-old male presents to the clinic c/o mild DOE. Examination reveals a prominent jugular pulsation and a palpable parasternal lift. There is a harsh systolic murmur best heard at 2nd and 3rd LSB; it radiates to the left shoulder. An early systolic sound precedes the murmur during expiration. ECG demonstrates RAD. What is the most likely diagnosis?

Pulmonic Stenosis This is a classic description of pulmonic stenosis. Mild forms of this congenital disorder may not be apparent until adulthood when left sided failure develops

Which of the following findings would be most indicative of a bacterial sinus infection?

Purulent discharge

A 33 year-old presents with increasing feelings of worthlessness and hopelessness over the past several months. Other symptoms include frequent crying episodes, loss of appetite, weight loss, insomnia with daytime sleepiness, and thoughts of suicide without any specific plans in place at this time. Which of the following would be the most appropriate first-line medication for this patient?

SSRI: Fluoxetine (Prozac) Selective serotonin reuptake inhibitors are the drugs of first choice for treating major depression.

A patient admitted with substernal CP undergoes cardiac catheterization. Angiography reveals 98% occlusion of the RCA . All other vessels are 100% patent. Which of the following is the most expected. EKG finding in this patient?

ST elevation in leads II, III, and avF The right coronary artery is responsible for inferior wall of the heart. Inferior wall of the heart is best illustrated on EKG in leads II, III, aVF.

A child sustains a fracture through the epiphysis into the articular surface. How would you classify this fracture?

Salter-Harris III S - I - Slipped - separation physis A - II - Above - metaphysis and physis L - III - Lower - epiphysis and physis T - IV - Through - metaphysis, physis, epiphysis R - V - Ruined/Rammed - crushed physis

A 36 year-old male who is hospitalized because of severe injuries from a motor vehicle accident develops rapid onset of profound dyspnea. Initial chest x-ray shows a normal heart size with diffuse bilateral infiltrates. Follow-up chest xray shows confluent bilateral infiltrates that spare the costophrenic angles. Which of the following is the best clinical intervention for this patient?

Tracheal intubation Treatment of hypoxemia in acute respiratory distress syndrome (ARDS) usually requires tracheal intubation.

A 36 year-old African American female comes to the clinic for an insurance physical which requires a chest x-ray. She denies any respiratory symptoms. Examination of her chest is negative. X-ray results show marked lymphadenopathy in the right paratracheal region. Angiotensin-converting enzyme (ACE) levels are elevated. Which of the following is the most likely diagnosis?

Sarcoidosis Sarcoidosis is characterized by paratracheal lymphadenopathy and elevated ACE levels. It is more common in African American patients and may be asymptomatic.

A 68 year-old male who is obese has a 50 pack-year history of smoking. He experienced a sudden and total loss of vision in his right eye, which resolved after 20 minutes. Which of the following is the most likely diagnosis?

Transient ischemic attack This history of sudden onset of a unilateral neurological deficit in a patient with risk factors for vascular disease suggests vascular ischemia. By definition, a transient ischemic attack is a neurological deficit that resolves quickly.

A two month-old infant appeared well until three weeks ago when he became dyspneic and had difficulty feeding. A 4/6 holosystolic murmur is heard at the LLSB in the 3rd ICS. An EKG shows both LVH, & RVH. Which of the following is the most likely diagnosis?

Ventricular septal defect This is a classic presentation for a ventricular septal defect.

Four days after an anterior MI, a patient abruptly develops severe hypotension. A pansystolic murmur is heard at the LSB. Most likely diagnosis is:

Ventricular septal rupture The presentation is typical of a ventricular septal rupture. Diagnosis can be confirmed with Echocardiogram or Swan-Ganz catherization, which should show a oxygen step-up. (O2 saturation should be higher in the PA than RA)

A patient who has long QT syndrome is most likely to have syncope caused by which of the following

Ventricular tachycardia Clinical Presentation When Sympathetic NS Is Activated Syncope Sudden Cardiac Death / Crib Death Due to Torsades de Pointes (vent. tachy) Structurally normal heart Normal cardiac exam when patient is asymptomatic Remember These 2 Causes of Syncope and Sudden Cardiac Death in Children, Adolescents and Young Adults Long QT interval Hypertrophic Cardiomyopathy

Meniere's Disease is associated with:

Vertigo, hearing loss, tinnitus

Stress, poor hygiene, and malnutrition is most associated with which condition?

Vincent's angina

What is the most common pathogen in sinusitits?

Viral

A 23 year-old male comes to the office with an intensely itchy rash, especially at night for the past week. His roommates have similar symptoms but have not been evaluated. On physical examination, the wrists, web spaces of the hands, and axillae have thin linear markings, tiny vesicles, and excoriations. Which of the following is the most likely diagnosis?

Scabies Scabies is an infestation of the mite Sarcoptes scabiei. It is usually spread by skin-to-skin contact. Scabies is characterized by intense pruritic burrows, vesicles and excoriations of the finger webs, wrist creases and axillae.

What is the MOST APPROPRIATE next step (Melanoma)?

Schedule a wide excisional biopsy ASAP A full-thickness wide excisional biopsy is recommended since it permits the pathologist to measure the thickness of the lesion. Since biopsy or curettage will not afford accurate histological assessment and is contraindicated

A 62 year-old male smoker presents to the clinic with the complaint of a chronic cough, hemoptysis, and weight loss. Chest CT shows a mass obstructing the bronchus with hilar and mediastinal lymph node abnormalities. Bronchoscopy with biopsy is performed. On reviewing pathology results you explain to the patient that his type of lung cancer is prone to early hematogenous spread, is rarely amenable to surgical resection and has a very aggressive course. What type of lung cancer is most likely in this patient?

Small cell Small cell lung cancer is very aggressive with a median survival (untreated) of 6-18 weeks.

Which of the following hormones primarily inhibits growth hormone secretion from the pituitary gland?

Somatostatin Somatostatin inhibits the release of growth hormone from the pituitary gland as well as hyperglycemic states. It is therefore useful in the treatment of excessive growth hormone release that occurs with gigantism and acromegaly

A 40-year-old obese, white woman, mother of five children, gives a history of repeated episodes of right upper quadrant abdominal pain. The pain is brought about by the ingestion of fatty foods and is relieved by the administration of anticholinergic medications. The pain is spasmodic, radiates to the right shoulder and around to the back, and is accompanied by nausea and occasional vomiting. The patient has no pain at this time, but is anxious to avoid further episodes. She is afebrile, and physical examination is unremarkable. Which of the following is the most appropriate next step in management?

Sonogram of biliary tract and gallbladder

A 30 year-old female presents with left wrist pain after slipping on the ice while walking to her car. On examination, pain is noted on palpation over the anatomical snuff box. X-ray of her wrist shows no identifiable fracture. Which of the following is the most appropriate treatment in this patient?

Splint application Splint Application Tenderness in the snuff box should be treated as a suspected scaphoid fracture. The patient should be treated as if it is fractured and placed in a splint with a referral to an orthopedic specialist for further evaluation and repeat imaging.

A 16-year-old female is evaluated because of fever and headache. Her 2-year old brother had a rash-like illness 3 weeks previously. The patient has had a fever (38 C) and a moderately stiff neck. Lumbar puncture is performed and the spinal fluid contains 8 white cells/mm 3, with 70% lymphocytes. The glucose level is 70 mg%, with a concurrent blood glucose level of 90 mg%. Gram's stain is negative. Peripheral white blood cell count is 5,000 cells/mm3. Spinal fluid samples are sent for cultures. Which of the following is the most likely diagnosis?

Viral Meningitis

A 10 year-old African immigrant presents 2 weeks after arriving in the US describing dry eyes and difficulty with night vision. Examination reveals numerous, small white conjunctival patches bilaterally. This patient's symptoms are most likely due to which of the following deficiencies?

Vitamin A Vitamin A deficiency is one of the most common causes of blindness in developing countries. It is also seen secondary to malabsorption disorders and frequent mineral oil laxative use. Night blindness is one of the earliest symptoms.

Prolonged use of a proton pump inhibitor can lead to low levels of which of the following nutrients?

Vitamin B12 Vitamin B12 requires gastric acid for absorption in the stomach. Prolonged use of a proton pump inhibitor suppresses gastric acid production.

An 18 year-old female with diabetes presents to the emergency department with altered level of consciousness, deep breathing and fruity odor to her breath. Which of the following medications is indicated for this patient?

Regular insulin Regular insulin has a rapid onset of action when given intravenously and is the initial choice in patients with diabetic ketoacidosis. Following the initial dose, a continuous infusion often promotes a steady, slow fall of glucose levels to normal, which can then be stabilized by decreasing the insulin

What findings would you suspect on Echocardiogram (ASD)?

RAE & RVH Reveals evidence of RV volume overload, including abnormal motion of ventricular septum (i.e., right-to-left movement) during diastole.

EKG shows: Which of the following is the most appropriate long-term management in this patient?

Radiofrequency ablation Pre-excitation Syndrome Congenital(Genetic in 3%); Structurally normal heart; Cardiac Exam normal Accessory pathways bypass AV node; Predisposes to AF w/ very fast HR EKG: short PR interval; delta wave in upstroke of QRS complex Do NOT give medication that slows atrioventricular conduction Treatment of Choice Radiofrequency ablation

A 63-year-old woman with a history of stage II breast cancer 8 years prior is seen in your clinic. Her breast cancer was treated with lumpectomy, adjuvant chemotherapy, and localized radiotherapy. She completed 5 years of tamoxifen. She has no symptoms of recurrent disease. A routine bone densitometry study reveals moderate osteoporosis. She does have a family history of osteoporosis, and her mother suffered a hip fracture in her early 70's. Which of the following therapeutic options would be most appropriate?

Raloxifene Currently in the United States alendronate and raloxifene are approved for the prevention of osteoporosis. Alendronate, raloxifene, and calcitonin are approved for treatment of osteoporosis. Alendronate has the greatest positive effect on bone mineral density and reduces the incidence of vertebral and nonvertebral fractures. In addition, raloxifene and calcitonin also appear to reduce the incidence of vertebral fractures. Given the patient's prior diagnosis of breast cancer, estrogen therapy for the prevention and treatment of osteoporosis is usually contraindicated. The anti-estrogen affect of raloxifene may also prove beneficial in patients with a history of breast cancer, and the agents listed above would be the most appropriate option. In the clinical setting, use of either alendronate or raloxifene would be appropriate

A patient with obsessive-compulsive disorder would most likely have which of the following findings?

Raw, red hands Common manifestations of obsessive-compulsive disorder include phobias of germ and contaminants, which results in frequent hand washing leading to chafe and reddened hands. The other answers are inconsistent with obsessive-compulsive disorder.

A 34 year-old female construction worker presents with episodic blanching of her fingers when exposed to cold weather. The physical examination of her extremities and digits is normal at this time. Which of the following is the most likely diagnosis?

Raynaud phenomenon Raynauds phenomenon is caused by vascular spasm when exposed to cold or stressful situations

A patient with Type 2 diabetes uses a mixture of NPH and regular insulin twice daily. She consistently has mid- afternoon bouts of hypoglycemia, despite eating her meals as scheduled. Which modification is most appropriate to reduce her mid-afternoon hypoglycemic events?

Reduce morning dose of NPH insulin Reducing the morning dose of NPH insulin would be most appropriate as it exerts its greatest effect on the noontime meal

A 50 year-old male with a history of alcohol abuse presents complaining of nodular thickening of the palmar aspect of the left hand. Examination reveals flexion of the 3rd, 4th, and 5th metacarpophalangeal and proximal interphalangeal joints. Which of the following is indicated at this time?

Refer to a hand surgeon Dupuytren contracture Nonsurgical treatment is ineffective in reversing or halting Dupuytren contracture. The primary indication for surgery is a fixed contracture of more than 30 degrees at the metacarpophalangeal joint or any degree of flexion contracture at the proximal interphalangeal joint.

An elderly woman presents to your clinic complaining of unilateral facial pain and painful lesions. She also complains of blurred vision in the ipsilateral eye. On examination she has several vesicles on an erythematous base, some of the lesions with crusts. They are distributed in a dermatomal pattern and involve the skin overlying the maxillary region and the tip of her nose. Which of the following is the next most appropriate intervention in the care of this patient?

Referral to an ophthalmologist Immediate referral to an ophthalmologist is needed when herpes keratitis is suspected, as in this case. A fluorescein stain of the eye might reveal the typical dendritic corneal lesion.

An elderly patient in a long-term care facility develops influenza A pneumonia despite both vaccination and amantadine prophylaxis. What is the organism most likely to complicate influenzae pneumonia?

S. pneumoniae

A 12-year-old girl is referred to you following a screening for scoliosis at school by the school nurse. The nurse used a scoliometer that revealed a reading >5 degrees and felt radiographs were needed at this point. The girl is totally asymptomatic of any back pain or motion or activity limitations. You order a standing AP and lateral long cassette spine and use the Cobb method to measure your curvatures. You measure and find a thoraco-lumbar single curve of 25 degrees with

Repeat radiographs in 3 months For progressive curves in a patient with Risser stage of 3, it is recommended that you repeat radiographs in 3 months. A year is too long to wait in an actively growing child. Bracing is reserved for curves >30° in a patient with Risser stage of 3. Surgical referral is not needed until idiopathic curves reach 50° or for curves of 40° to 50° that are likely to progress. This patient should be followed up with every 3 months until skeletally mature.

A 65 y/o female patient complains of fatigue and SOB. Objective findings note an elevated JVP and a Kussmaul's sign. There is low voltage on the EKG with nonspecific repolarization changes. An echocardiogram shows impaired diastolic filling with preserved contractile function. You suspect which of the following?

Restrictive Cardiomyopathy

A 40 year-old male nonsmoker in good health undergoes a routine chest x-ray for an insurance physical. Results show an isolated, well-defined, coin lesion 1 cm in size. Which of the following is the next step in the evaluation of this problem?

Review old radiographs The first and most important step in the radiographic evaluation is to review old radiographs to estimate doubling time, an important marker for malignancy.

A 39-year-old female comes to your office with a 6-month history of malaise, paresthesias in both hands, and vague pain in both hands & wrists. She has also felt extremely fatigued. She tells you that the pains in her joints are much worse in the morning. She is also beginning to notice pain & swelling in both knees. The patient has a normal family history with no significant diseases noted. The patient is taking no drugs and has no allergies. On examination, there is a sense of "bogginess" & slight swelling in both hands, in both wrists, and in the small bones of the hands. Both knees also feel somewhat "swollen & boggy." There are no other joint abnormalities, and the rest of the physical exam is normal1. What is the most likely diagnosis in this patient?

Rheumatoid arthritis

A 50 year-old female presents with a two-month history of fatigue and morning stiffness involving both hands. It is now difficult for her to perform simple tasks at home and work. Examination reveals swelling and warmth of the metacarpophalangeal and proximal interphalangeal joints, sparing the distal interphalangeal joints. Elevation of which of the following laboratory test results would best support the suspected diagnosis?

Rheumatoid factor A high titer of rheumatoid factor is the most significant diagnostic finding.

A 22 year-old presents with over 6 months of increasing problems with hallucinations and delusions. The patient states that the extraterrestrials are sending messages to a chip implanted in his brain via channel 8 on television, and sometimes also via WYYZ on the radio. Most of the time they are telling him to avoid talking to his friends because they are planning to sell his kidneys to a man in India and his heart to a man in Ethiopia. A medical screening and drug screen revealed no obvious causes for the hallucinations. First line pharmacotherapeutic treatment of this patient includes which of the following drugs?

Risperidone (Risperdal) Risperidone is one of the first line options for the treatment of schizophrenia due to its effectiveness and lower side effect profile compared to other drugs.

A 43 year-old woman presents with episodes of facial flushing with increased skin temperature followed by the development of tiny papules and pustules primarily on her cheeks and chin. These symptoms worsen when she eats spicy foods. On examination, telangiectasia are noted along the nasal folds and tiny pustules on both cheeks. Which of the following is the most likely diagnosis?

Rosacea Rosacea is chronic, episodic flushing of the face followed by the development of tiny papules and pustules. Eating spicy foods or drinking hot liquids or alcohol can cause exacerbation of the symptoms

A two year-old male is brought into a free clinic with a fever of 102 degrees F for two days. Through an interpreter, the mother states that they are from Romania. Examination reveals that the buccal mucosa has a grainy appearance opposite the second molars. Which of the following is the most likely diagnosis?

Rubeola Rubeola or measles causes Koplik spots which are described as the salt-like crystals opposite the second molars.

A 55-year-old female, previously healthy and recovering from an episode of bronchitis, suddenly develops a "shaking chill" followed by the onset of a high fever(40C), pleuritic chest pain, and cough productive of purulent, rust-colored sputum. On examination, the patient appears ill. Her respiratory rate is 30 breaths/min, and chest splinting is present. Bronchial breath sounds are heard in the right lower lobe. CXR is seen here. What is the most likely organism responsible for the patient's illness?

S. pneumoniae

A 34 year-old woman who is new to the practice has a long history of medical complaints that include almost every organ system. She describes herself as being ill since she was a child. She takes a total of 25 prescriptions from multiple providers, including prednisone and multiple pain medications, as well as several over-the-counter medications. You find no abnormalities on comprehensive examination. What is the recommended plan for this patient?

Schedule monthly visits with you The patient has somatization disorder. It is important that one provider coordinates the medical care of patients with somatization disorder. Regular, short appointments are helpful, and brief physical examinations should be performed to assess somatic complaints. Diagnostic evaluations should be limited.

A 22 year-old male presents to the behavioral health unit with his family who relate his 5 month history of delusions and hallucinations. Examination of the patient reveals a flattened affect, disorganized speech and poor hygiene. Which of the following is the most likely diagnosis?

Schizophreniform disorder Schizophreniform disorder is characterized by the same features as schizophrenia except for the total duration of illness being from 1-6 months. Additionally, with schizophreniform, there is no impairment in social or occupation functioning.

A 41 year-old female presents to you for medical screening advice. Her 44 year-old sister passed away recently 18 months after diagnosis of metastatic colon cancer. Which of the following is the most appropriate advice for this patient?

Screening colonoscopy now and repeat every 3-5 years if normal Hereditary factors are believed to contribute to up to 30% of colorectal cancers. Relative risk is 3.8 times if the family member's cancer was diagnosed at less than 45 years of age. Recommended screening in a single first degree relative with colorectal cancer diagnosed before age 60 is beginning colonoscopy at age 40 or ten years younger than age at diagnosis of youngest affected first-degree relative. Then if negative, every 5 years

A 41 year-old male with a history of intravenous drug abuse presents to your office with acute, nontraumatic right knee pain, chills, and sweats starting 2 days ago. On physical examination, his temperature is 102.9 degrees F. The right knee is erythematous, edematous and tender to palpation and range of motion. Plain knee x-ray reveals soft tissue swelling. Which of the following is most likely the diagnosis?

Septic arthritis This patient's signs and symptoms are most consistent with septic arthritis. IV drug abuse places this patient at even greater risk.

A 26 year-old presents with two days of a generalized, non-pruritic rash with concurrent low grade fever and sore throat. He is otherwise in good health with no history of dermatologic problems other than acne and a non-painful ulceration at the base of his penis seven months ago that resolved spontaneously. Examination reveals a macularpapular rash spread diffusely over the body including the palms and soles of the feet. Shallow ulcers are noted on the buccal mucosa while the pharynx is moderately erythematous. Which of the following is the most appropriate initial diagnostic study?

Serum FTA-Absorption test With secondary syphilis, 100% of persons test positive with serum FTA-Absorption test.

A 31 year old nonsmoker male has a 2 month history of progressive dyspnea without associated cough or wheezing. Emphysema is diagnosed, based upon pulmonary function studies demonstrating airway obstruction. Which of the following is the most appropriate diagnostic test to be performed?

Serum alpha-antitrypsin level

A 23 year-old African American female presents with concern regarding "lightening" of the skin in her knees, face, clavicles, and wrists. These areas appear to be increasing in size. She was recently diagnosed with pernicious anemia. In order to confirm her diagnosis, which of the following tests is indicated?

Skin biopsy Skin biopsy in a patient with vitiligo will show complete absence of melanocytes and will confirm the diagnosis.

Which of the following is the most common extra-pulmonary finding in a patient with sarcoidosis?

Skin lesions Although sarcoidosis can affect nearly all organ systems dermatologic and ocular signs and symptoms dominate after pulmonary issues.

An 81 year-old long term care facility resident presents to your office with complaint of generalized itching over the last several days. She denies use of any new products or ingestions of new foods or medications. Examination is significant for generalized excoriations and burrows in the web spaces of the hands and feet. Which of the following tests will confirm your diagnosis of this patient?

Skin scrapings Skin scraping is the diagnostic tool for suspected scabies. A 15 blade is used to scrape multiple, unexcoriated, lesions and mounted on a slide for microscopy. Diagnosis is confirmed by the presence of the organism, ova or feces

A 16-year-old high school student presents with the sudden onset of sharp right-sided chest pain associated with shortness of breath. She denies any history of trauma. On physical examination, the patient is afebrile with a respiratory rate of 28/min. Her blood pressure is 100/70 mm Hg and his heart rate is 120/min. Neck examination reveals no tracheal deviation. On lung auscultation, the patient has decreased fremitus, hyperresonance, and diminished breath sounds over the right posterior hemithorax. Which of the following is the most likely diagnosis?

Spontaneous pneumothorax Clinical manifestations include sudden onset of dyspnea and pleuritic chest pain.

A 60 year-old male presents with a slowly developing facial lesion first noticed 4-5 months ago. He describes it as non-painful and non-pruritic but notes it to be extremely scaly. He denies a history of similar lesions or dermatologic disease. Examination reveals a one centimeter, firm nodule at the right temple with heavy keratinization. There is no fluctuance or skin discoloration. Which of the following is the most likely diagnosis?

Squamous cell cancer This is a very typical scenario for a squamous cell cancer occurring in a sun exposed area, with slow development and heavy keratinization

The most frequent malignant neoplasm involving the larynx is:

Squamous cell carcinoma

A 56-year-old African American man presents with a 15-lb weight loss over the last 6 wk. he states that food "gets stuck" in the middle of his chest. Initially the patient had difficulty swallowing solids, but he symptoms progressed to the point where he has similar problems when swallowing liquids. He also complains of odynophagia. He denies hoarseness. He is a smoker and admits to heavily drinking alcohol. PE reveals a L fixed supraclavicular node. Which of the following is the most likely diagnosis?

Squamous cell carcinoma of the esophagus Progressive (solids > liquids) difficulty swallowing or dysphagia accompanied by rapid weight loss and odynophagia(painful swallowing) often indicates esophageal cancer. Risk factors for esophageal cancer include tobacco and alcohol use, chronic gastric reflux causing a Barrett's esophagus, achalasia, and lye ingestion. Tumor involvement of the recurrent laryngeal nerve would cause hoarseness. The fixed supraclavicular node (Virchow node) is consistent with the diagnosis. Adenocarcinoma is more common in white patients, while squamous cell carcinoma is more common in black patients.

You are completing a physical on a patient and his blood pressure is 160/100. According to The Seventh Report of the Joint National Committee on Prevention, Detection, Evaluation, and Treatment of High Blood Pressure. What category would this blood pressure qualify as? What category would this blood pressure qualify as?

Stage 2 hypertension

A 66 year-old woman with type 2 diabetes comes to the office because she has had a painful red patch that has been spreading on her right lower leg over the past 3 days. Her temperature is 100.4 degrees F. Physical examination of her right lower leg reveals a 3 cm tender, warm, erythematous, and edematous plaque. Which of the following pathogens is most likely causing this patient's symptoms?

Staphylococcus aureus S. aureus and group A beta-hemolytic streptococci are the most common causes of cellulitis

A 61-year-old man with coronary artery disease complains of progressive orthopnea and pedal edema. He is hospitalized with a blood pressure of 190/105mm Hg. The cardiac enzymes and EKG are normal. IV furosemide has been administered. What is the best next step?

Start an ACE inhibitor

A 10 year-old girl has been treated for a UTI with Bactrim for 4 days. Patient was feeling better until last night when she developed a low-grade fever and cough. She presents with multiple flat, irregular target type lesions and macules on the hands, trunk and extremities. There are multiple lesions in the mouth and genital mucus membranes. This presentation most likely represents:

Steven's Johnson syndrome This severe form of allergic erythema multiforme which involves mucus membranes is known as the Stevens Johnson Syndrome, commonly caused by drug allergies, especially to Bactrim. This could progress to Toxic Epidermal Necrolysis (TEN) and this patient needs to be treated with corticosteroids and followed closely Generalized eruption of lesions that can initially have a target-like appearance but then become confluent, brightly erythematous, and bullous and leads to epidermal loss. 90% of patients have mucocutaneous erosions (mouth, lips, conjunctiva, genital and anal skin) Patients present with fever, photophobia, sore throat By definition involves less than 10% BSA in SJS, 10-30% in overlap cases, and greater than 30% BSA in TEN Note the target lesions on the hands of this patient, as well as the mucosal involvement on the lips

Which of the following therapies is most likely to produce the greatest benefit to a patient with chronic stable emphysema and a resting oxygen saturation of 86 %

Supplemental O2 used continuously For patients with hypoxemia, supplemental oxygen has a significant impact on mortality, with a greater benefit with contyinuous usgage rather than intermittent or nocturnal use only

A 4-year-old girl presents to the Emergency Department with her parents who witnessed her falling on an outstretched right dominant arm with her elbow fully extended from a slide (about 4 feet fall) 30 minutes ago. There was no loss of consciousness, but there was extreme pain, and she immediately grabbed her right forearm and her right elbow. There was an obvious deformity at the elbow. Which of the following is the most likely diagnosis?

Supracondylar humerus fracture Supracondylar fractures of the distal humerus are the most common elbow fracture in children between the ages of 2 and 12. The typical mechanism of injury for this type of fracture is a fall on an outstretched arm, usually from a height.

A healthy 75-yrar-old man undergoing an ultrasound examination for suspected gallbladder disease is found incidentally to have a 6.0-cm abdominal aneurysm of the aorta. Which of the following is the best management for this patient?

Surgical repair of the aneurysm AAA Complications Propensity for enlargement & rupture AAA enlarge at average rate of 0.3 cm/yr Some double in size over a few months Risk of rupture diameter dependent J = P x r/t P = intraluminal pressure, r = aneurysm radius, t = wall thickness Thrombus formation along wall which may either embolize or stimulate aneurysm growth Treatment 4 cm AAA annual risk of rupture < 5 % 6 cm AAA annual risk of rupture 15 % Repair recommended when risk of rupture > risk of surgery; occurs with a AAA > 4.0- 5.5 cm or if tender

A 45-year-old woman with new-onset AI is found to have an aortic dissection of the ascending aorta and aortic arch by echocardiography. She is relatively asymptomatic. Which of the following is the best management?

Surgical repair of the dissection Surgery is urgently required in the event of aortic root or other proximal(type A) dissection. Unrecognized and hence untreated aortic dissection can quickly lead to exsanguination and death. Medical therapy such as beta-blockers can help to decrease the risk of dissection while getting the patient urgently to the operating room

A 7 year-old white male presents with a life-long history of frequent, and commonly complicated, bouts of bronchiectasis and other respiratory tract infections. His past history includes poor weight gain, low exercise endurance and pancreatitis. Examination reveals a barrel chest and hyperresonance to percussion while a mixed obstructive-restrictive pattern is seen on pulmonary function testing. Peribronchial cuffing and increased interstitial markings are noted on chest radiograph. Which of the following tests would confirm the diagnosis?

Sweat chloride test The sweat chloride test, though not the only diagnostic study, is a commonly ordered test to help solidify the diagnosis of cystic fibrosis. Purified protein derivative is used to assess for tuberculosis, serum ACE levels can be used to diagnose sarcoidosis and alpha-1 antitrypsin levels are checked for deficient states.

The 4 cardinal symptoms of aortic stenosis are:

Syncope, dyspnea, angina, sudden death The 4 cardinal symptoms of aortic stenosis are: A. Syncope, dyspnea, angina, sudden death Non-Surgical Avoid strenuous activity in asymptomatic phase Avoid hypotension Treat associated CHF in standard fashion but avoid hypotension (caution use of afterload & preload reducing agents) Do not perform a stress test Surgical Aortic Valve Replacement Indications Severe AS, Symptoms resulting from AS Exercise/dobutamine/adenosine stress tests would be contraindicated in these patients Most patients should undergo cardiac cath before surgery to determine whether concomitant CAD is present so they can have CABG at the same time.

A 35yo women presents to you complaining of abdominal pain and a malar rash that began shortly after the initiation birth control pills. She also complains of diffuse arthalgias of wrists and ankles. On laboratory evaluation she has a positive ANA in high titer, a positive ds DNA antibody, and an elevated ESR with a low compliment level. Based on the above the most likely diagnosis is:

Systemic Lupus Erythematosus Clinical Manifestations of SLE % of pts. are women, usually of child-bearing age More common in blacks Course of disease is often characterized by periods of exacerbation & relative quiescence May involve virtually any organ system & have a wide range of disease severity Constitutional -fatigue, fever, malaise, weight loss Cutaneous-rashes(especially malar "butterfly" rash), photosensitivity, vasculitis, alopecia, oral; ulcers Arthritis-inflammatory, symmetric, nonerosive Hematologic-anemia(may be hemolytic), neutropenia, thrombocytopenia, lymphadenopathy, splenomegaly, venous or arterial thrombosis Cardiopulmonary-pleuritis, pericarditis, myocarditis, endocarditis. & increased risk of MI usually due to accelerated atherosclerosis Renal- nephritis-classification is primarily histologic GI- peritonitis, vasculitis Neurologic- OBS, seizures, psychosis, cerebritis

A 50-year-old woman has had Raynaud's phenomenon of the hands for 15 years. The condition has become worse during the last year, and she has developed arthralgia's and arthritis involving the hands and wrists as well as mild sclerodactyly and difficulty swallowing solid foods. Physical Exam of the face shows shiny, taut skin and no facial wrinkles Laboratory studies reveal a positive serum antinuclear antibody assay at a dilution of 1:160. Anticentromere antibodies are present in high titers; antiribonucleoprotein antibodies are not detectable. The most likely diagnosis of this woman's disorder is

Systemic sclerosis(scleroderma) Systemic sclerosis can be classified into two variants depending on whether scleroderma is present only in the fingers (sclerodactyly) or whether it is also present proximal to the metacarpophalangeal joints. The former disorder is associated with a constellation of findings labeled the CREST syndrome: calcinosis, Raynaud's phenomenon, esophageal dysmotility, sclerodactyly, and telangiectasia. Although once thought not to be associated with significant internal organ involvement, the CREST variant of systemic sclerosis has occurred in association with the development of pulmonary arterial hypertension or biliary cirrhosis. The fluorescent antinuclear antibody (ANA) test is positive in 40 to 80% of persons with systemic sclerosis. Antibodies are produced to deoxyribonucleoprotein, nucleolar, centromere, and topoisomerase 1 antigens. MCTD is the overlap of three rheumatic disease syndromes: SLE, polymyositis, and the CREST variant of systemic sclerosis. It is associated with high titers of ANAs directed against the extractable nuclear antigen ribonucleoprotein. Arthritis and a positive ANA test are not sufficient to make a diagnosis of SLE. Overlap syndromes are diseases that fulfill diagnostic criteria for two rheumatic diseases

A 70-year-old woman presents with blurring of vision in the left eye since waking earlier in the morning. She reports 2 months of fevers, sweats, anorexia, and a 4.5-kg (10-lb) weight loss. She also reports increasingly severe left temporal headaches over the same time period.Her physical examination reveals scalp tenderness over the left temporal region. Her laboratories reveal a normochromic, normocytic anemia, mildly elevated alkaline phosphatase, and an erythrocyte sedimentation rate of 92. What is the MOST likely diagnosis

Temporal Arteritis(Giant Cell) Inflammation of medium-and large-sized arteries Involves temporal artery but systemic and large vessel involvement may occur Symptoms include headache, jaw/tongue claudication, scalp tenderness, fever, musculoskeletal symptoms(PMR) Clinical Findings Headache & painful, palpably enlarged & tender temporal artery. No specific laboratory findings, though ESR is often > 100

What nail finding is most consistent with cirrhotic liver disease?

Terry's nails Terry's nails occur when the nail plate turns white with a ground glass appearance, a distal band of reddish brown, and obliteration of the lunula. Terry's nails are seen in liver disease, usually cirrhosis.

A newborn is being evaluated for perioral cyanosis while feeding associated with sweating. T is 37.8 100 F , BP 80/45, HR 180, and RR 40. A grade 3/6 harsh SEM with a single loud S2 is heard at the LUSB. ECG shows RVH with RAD . CXR shows a boot- shaped heart and decreased pulmonary vascular markings. Which of the following is the most likely diagnosis?

Tetralogy of Fallot This is a common presentation for tetralogy of fallot. Clinical Presentation Infants have paroxysmal dyspnea with LOC & central cyanosis Older children develop clubbing of fingers & toes Retarded growth in childhood PE Findings Parasternal heave & precordial prominence. SEM over 3rd & 4th ICS, sometimes radiating to L side of neck; Thrill may be present; Single S2 is heard; Clubbing.; Arterial & Venous pulses are normal.

A 52 year-old patient presents with fatigue, complaints of paleness, anorexia, nausea, and weight loss. The patient also complains of numbness in his hands and feet and a recent occurrence of foot drop. He has a past history of diabetes and hypertension. Based on his clinical presentation, which of the following disorders is most likely to be responsible for this clinical picture?

chronic renal failure

A 22 year-old male presents to the emergency department complaining of right hand pain after punching a brick wall. His pain is noted at the ulnar aspect of his hand and worsens with touch and movement. On examination, you notice obvious swelling and tenderness over the dorsum of the right hand proximal to the metacarpal phalangeal (MCP) joint of the fifth digit. Skin is intact. X-ray reveals a fracture of the proximal fifth metacarpal with good alignment. Which of the following is the treatment of choice?

Ulnar gutter splint to immobilize fracture site An ulnar gutter splint is the most appropriate care in the emergency room. Immobilize joint above and below

A 56 y/o male with a 30 pack-year smoking history presents with substernal CP. The pain is described as a pressure that radiates to his jaw. The pain has lasted consistently for 30 minutes with variable relief. His current medications include atorvastatin (Lipitor) and glyburide (Micronase). Which of the following aggravating or relieving factors about the pain would support the diagnosis?

Unrelieved by nitroglycerin Patient is having a myocardial infarction which is unrelieved by rest or nitroglycerin. Plaque rupture or erosion with overlying thrombus is the initiating mechanism Pain longer than 15-30 min, building up to maximum Dull or pressure-like pain in the midsternal or peristernal Associated symptoms; Nausea, diaphoresis, SOB, fatigue; Neck pain, arm pain

A 3 week-old infant is evaluated for persistent projectile vomiting described as breast milk without bile or blood. The abdomen is distended before vomiting and a small, mid-epigastric mass is palpable after vomiting. Which of the following is the most appropriate diagnostic study for the evaluation of this patient?

Upper GI contrast radiographs This infant has pyloric stenosis and an upper GI series will reveal a narrowed distal stomach with double tract of barium.

A 62 year-old male presents with complaints of dyspepsia, early satiety, and dysphagia. What diagnostic study would be indicated in his initial workup?

Upper endoscopy Upper endoscopy is indicated for patients over the age of 55 presenting with new-onset symptoms of dyspepsia in order to evaluate for gastric cancer or other serious organic disease. Upper endoscopy is the diagnostic study of choice to diagnose gastroduodenal ulcers, erosive esophagitis, and upper GI malignancy

A 75-year old man is noted to have chest pain with exertion and has been passing out recently. On exam he is noted to have a harsh systolic murmur. Which of the following is the best therapy for this condition?

Valve replacement The symptoms of AS classically progress through angina, syncope and finally, CHF, which has the sores prognosis for survival. The patients systolic murmur is consistent with aortic stenosis. An evaluation should include echocardiography to confirm the diagnosis, and then aortic valve replacement

A 40-year old woman is awakened at 2 AM with anterior CP. ECG taken during the discomfort shows ST segment elevation in leads V1-V4. Five minutes after SL nitroglycerin discomfort ends and repeat ECG is normal. Which of the following is the most likely diagnosis?

Variant angina pectoris Prinzmetal's variant(Variant angina pectoris) Unknown etiology Woman >>> Men Pathophysiology Coronary artery spasm Spasm occurs during early morning hours ST elevation during angina discomfort Caused by occlusive spasm superimposed on a nonsevere coronary artery stenosis. Pain occurs mainly at rest, but may occur during exercise. Pain may awaken patient in the morning. May be associated with Raynaud's and migraine HA. Diagnosed when transient ST segment elevation is documented isode of chest pain

A 65 year-old female who recently had an anterior MI returns to clinic for follow-up six weeks after. She has no CP , but reports decreased exercise tolerance EKG shows persistent ST elevation in leads V2- V4. Which of the following is the most likely diagnosis ?

Ventricular aneurysm

A 72-year-old man collapses while playing golf. He has a 5-year history of angina and type 2 DM . Paramedics arrive in 10 minutes. Examination shows no respirations or BP . CPR is attempted for 10 minutes without success. Which of the following is the most likely cause of death in this patient?

Ventricular fibrillation

A 43-year-old man presents with fever and arthritis. During the past 2 months he has been treated four times for a maxillary sinus infection. He also c/o the recent onset of hematuria. Which of the following is the most likely diagnosis?

Wegener's granulomatosis Wegener's granulomatosis involves the upper airways(nasopharynx and sinuses) and the lungs, kidneys and joints. The diagnosis is made by the clinical picture, a positive antineutrophil cytoplasmic antibody with a cytoplasmic staining pattern(C-ANCA) and biopsy showing necrotizing granulomas. The disease causes a systemic necrotizing arteritis and is fatal without treatment

A 56 year-old white post-menopausal female had a recent surveillance DEXA bone densitometry. The T-score is -2.7 for her right hip. In counseling this patient, in addition to medications, which of the following would you recommend?

Weight bearing exercises Weight bearing exercises are an appropriate adjunct to medication in a patient with osteoporosis. Water aerobics are non-weight bearing. Sun exposure and vitamin D supplementation are useful in preventing and treating osteomalacia.

A 72 year-old woman presents to your clinic complaining of constipation. Which of the following presentations would be most concerning?

Weight loss Alarm symptoms are concerning for colorectal malignancy and include weight loss, anemia, hematochezia, or positive fecal occult blood test. These symptoms are particularly worrisome in a patient with a family history of colorectal cancer.

A 67 year-old female presents with progressive pain in her left knee that is worse with activity and relieved with rest. She notes stiffness of the knee that last about 20 minutes after activity is resumed. She exercises regularly. No known drug allergies. On physical exam she is 5' 5", 225 pounds. Her left knee exam reveals mild effusion without erythema or warmth. Radiographs of the left knee reveal medial joint space narrowing and subchondral bone sclerosis. Her sodium is 138 mEq/L, potassium 4.3 mEq/L, bicarbonate 24 mEq/L, chloride 104 mEq/L, BUN 23 mg/dl and creatinine 1.8 mg/dl. Which of the following medications is most appropriate for this patient's worsening pain?

acetaminophen Acetaminophen is recommended as first-line pharmacotherapy in patients with osteoarthritis

A 6-YO child is struck by a car while riding a bicycle. He is reported to be unconscious for 2 minutes following the accident. He is conscious and alert upon arrival to the ER, but within 45 minutes he begins to vomit and shortly thereafter becomes completely unresponsive. Which of the following most likely explains this child's injury?

acute epidural hematoma Secondary to traumatic biomechanical forces on the brain as a direct blow to the head or indirectly transmitted to the head from implosive anywhere on the body.

A 24-year-old woman comes to your office complaining of anxiety. The patient had witnessed a traumatic event 3 days earlier that made her feel fearful. She has not been able to tell her family about this experience. She now feels like she is numb and in a dazed, dreamlike state with poor concentration, and difficulty sleeping. She experienced a flashback of the event yesterday. What is the most likely diagnosis?

acute stress disorder Sxs occur w/in one month of traumatic event and last from 2 days to 4 weeks

A patient diagnosed with Barrett's esophagus is at an increased risk for the development of what type of cancer?

adenocarcinoma The most serious complication of Barrett's esophagus is esophageal adenocarcinoma.

A 23 year-old female presents with ongoing arthralgias with intermittent flares of arthritis. She is found to have a malar rash and an abnormal urinalysis. Serum ANA and anti-double-stranded DNA antibodies are present. Which of the following tests should be ordered to assess her risk for thrombotic events and future risk of spontaneous abortion?

anti-phospholipid antibodies Anti-phospholipid antibodies are present in 25% of SLE patients and may cause thrombotic events and spontaneous abortion. Anticardiolipin Antibody in SLE Approximately 50% have an anticardiolipin antibody, which is associated with a prolonged PTT& false-positive serologic tests for syphilis. This so-called lupus anticoagulant may be manifested by thrombocytopenia, venous or arterial clotting, & recurrent fetal loss Though thrombotic problems are most common, if antibody is associated with hypoprothrombinemia, severe thrombocytopenia, or antibodies to clotting factors (usually VIII or IX), bleeding may result. Confirmation that PTT is prolonged on basis of a lupus anticoagulant may be proved by failure of normal plasma to correct defect.

A 55 year-old male presents with complaint of sudden ripping CP that radiates into the abdomen. On examination the patient is found to have diminished peripheral pulses and a diastolic murmur. EKG reveals LVH . Which of the following is the most likely diagnosis?

aortic dissection Aortic dissection is characterized by a ripping or tearing type pain with radiation to the neck, back or abdomen. Left ventricular hypertrophy is often seen on EKG secondary to longstanding hypertension. A diastolic murmur is often present secondary to aortic insufficiency.

A 32 year-old mother of three presents with complaints of headache, abdominal pain, and general anxiety. Upon questioning, she admits that her husband has pressured her into sexual activity and has humiliated her in front of friends and family by implying that she is stupid and worthless. He gives her a weekly allowance, but does not allow her use of credit cards. She feels trapped and hopeless, and does not see any way out of this relationship. She finds herself crying often when the children are at school. You should:

ask if she has considered killing her husband Victims of domestic abuse may see no way out of a relationship besides homicide to protect themselves and/or their children. Risk of injury or homicide requires further psychiatric evaluation immediately.

A 70-year-old man of Irish descent returns to his physician for a routine check of his blood pressure. He is a vigorous, retired executive who except for mild hypertension is healthy. After his examination, as he is getting dressed, he states that his wife has been nagging him to mention a spot on his nose The most likely diagnosis for this lesion is

basal cell carcinoma

In addition to verapamil (Calan), which of the following medications is considered the treatment of choice for patients with hypertrophic cardiomyopathy?

beta-blockers Patients with hypertrophic cardiomyopathy are treated with both calcium channel blockers and beta-blockers because these agents improve symptoms, especially dyspnea and chest pain. These agents slow the heart rate, which prolongs diastole and allows for increased passive ventricular filling. When the ventricle is allowed to fill, there is less outflow obstruction.

Which of the following are characteristics of Cushing's syndrome?

central obesity, glucose intolerance, and easy bruising These are classic signs of Cushing's syndrome.

Which of the following leukemia's is most closely associated with the Philadelphia chromosome?

chronic myelocytic Philadelphia chromosome is noted most commonly in patients with chronic myelogenous leukemia.

A 45 year-old woman has a 7 month history of moderate abdominal pain that is exacerbated by eating. She has used alcohol heavily for the past 10 years. X-ray films of the abdomen show diffuse retroperitoneal calcifications. The most likely diagnosis is

chronic pancreatitis Most cases due to ETOH Chronic abdominal pain, diarrhea, weight loss Amylase/lipase may be elevated or normal Imaging studies - KUB: pancreatic calcifications Treatment: Symptomatic w/ pancreatic enzyme replacement.

A 23-year-old previously healthy female letter carrier works in a suburb in which the presence of rabid foxes and skunks has been documented. She is bitten by a bat, which then flies away. Initial examination reveals a clean break in the skin in the right upper forearm. She has no history of receiving treatment for rabies and is unsure about vaccination against tetanus. The PA should

clean the wound with a 20% soap solution, administer tetanus toxoid, administer human rabies immune globulin intramuscularly, and administer human diploid cell vaccine Post Exposure Prophylaxis Therapy 1. Vigorous cleaning of the wound with a 20% soap solution 2. Tetanus toxoid & antibiotics 3. Passive immunization with human rabies immune globulin dose of 10 units/kg into the wound and 10 units/kg IM into the gluteal region. 4. Actively immunize with an antirabies vaccine in five 1-mL doses given intramuscularly, preferably in the deltoid or anterior lateral thigh area over 28 days

A 12-y/o boy presents to the office with pain in his legs with activity gradually becoming worse over the past month. He is unable to ride a bicycle with his friends due to the pain in his legs. Exam of the heart reveals an ejection click and accentuation of S2. Femoral pulses are weak and delayed compared to the brachial pulses. BP obtained in both arms is elevated. CXR reveals rib notching. Which of the following is the most likely diagnosis?

coarctation of the aorta Coarctation is a discrete or long segment of narrowing adjacent to the left subclavian artery. As a result of the coarctation, systemic collaterals develop. X-ray findings occur from the dilated and pulsatile intercostal arteries and the "3" is due to the coarctation site with proximal and distal dilations.

A 12 year-old boy presents to the office with pain in his legs with activity gradually becoming worse over the past month. He is unable to ride a bicycle with his friends due to the pain in his legs. Examination of the heart reveals an ejection click and accentuation of the second heart sound. Femoral pulses are weak and delayed compared to the brachial pulses. Blood pressure obtained in both arms is elevated. Chest x-ray reveals rib notching. Which of the following is the most likely diagnosis?

coarctation of the aorta Coarctation is a discrete or long segment of narrowing adjacent to the left subclavian artery. As a result of the coarctation, systemic collaterals develop. X-ray findings occur from the dilated and pulsatile intercostal arteries and the "3" is due to the coarctation site with proximal and distal dilations.

A patient presents with a macrocytic anemia and paresthesia of the lower extremities. The treatment of choice is

cobalamin The patient presents with laboratory findings and symptoms of Vitamin B12 deficiency. The treatment of choice is cobalamin.

An 18 year-old patient has a tibia/fibula fracture following a motorcycle crash. Twelve hours later the patient presents with increased pain despite adequate doses of analgesics and immobilization. Which of the following is the most likely diagnosis?

compartment syndrome Compartment syndrome is characterized by a pathological increase of pressure within a closed space and results from edema or bleeding within the compartment. It may occur as an early local complication of fracture

A 43 year-old male with a history of a right medial meniscectomy and a strong family history of osteoarthritis presents to the clinic for a routine physical exam. He states he is very active and runs 20-25 miles a week and competes routinely in 5 km races. He is 5' 10" and 160 lbs, BP is 128/76 and P 72. His physical examination is unremarkable. Which of the following would you recommend to this patient to delay the onset of osteoarthritis?

consider swimming or biking instead of running Swimming and biking would promote excellent joint motion and muscle strength and void the high-impact of competitive short-distance running.

A 41 year-old female complains of 3 weeks of gradually worsening pain at the base of the thumb and radial aspect of the wrist. She and her husband have been renovating their home for the past 2 months and it has become increasingly difficult for her to hold a hammer. She denies numbness or tingling. She denies any history of previous trauma to the wrist. On examination, there is tenderness over the distal radial styloid and pain reproduced with ulnar deviation of a fist clenched over the abducted thumb. Which of the following is the most likely diagnosis?

deQuervain's tenosynovitis deQuervain's tenosynovitis typically results from repetitive activity involving pinching the thumb while moving the wrist. There is often pain and tenderness over the radial styloid and Finkelstein's is positive in this patient.

A 26-year-old woman with SLE is noted to have a prolonged partial thromboplastin time. This abnormality is associated with

deep venous thrombosis Patients with SLE may have a host of autoantibodies. Virtually all have antinuclear antibodies directed at multiple nuclear and cytoplasmic antigens Approximately 50% have an anticardiolipin antibody, which is associated with a prolonged partial thromboplastin time and false-positive serologic tests for syphilis. This so-called lupus anticoagulant may be manifested by thrombocytopenia, venous or arterial clotting, recurrent fetal loss, and valvular heart disease. Though thrombotic problems are most common, if the antibody is associated with hypoprothrombinemia, severe thrombocytopenia, or antibodies to clotting factors (usually VIII or IX), bleeding may result. Confirmation that the partial thromboplastin time is prolonged on the basis of a lupus anticoagulant may be proved by failure of normal plasma to correct the defect.

A 22-year-old woman with a history of ulcerative colitis presents with jaundice, pruritus, and intermittent right upper quadrant abdominal pain. The most likely finding on ERCP

diffuse strictures(sclerosing cholangitis) This patient is presenting with cholestasis. Pain would be suggestive of intermittent biliary obstruction and would not be as typical of intrahepatic cholestasis, given the normal hepatic transaminases, which would rule out hepatitis. Patients with ulcerative colitis are at particular risk for certain extraintestinal manifestations such as sclerosing cholangitis. Other complications of intermittent biliary obstruction include complete biliary obstruction, secondary biliary cirrhosis, hepatic failure, or portal hypertension.

A 26 year-old female required 12 units packed red blood cells during a trauma resuscitation and surgical repair of liver and splenic lacerations. The patient is now 6 hours postoperative and has blood oozing from the suture line and IV sites. There is bloody urine in the Foley bag. Laboratory evaluation demonstrates a platelet count of 10,000/microliter, prolonged prothrombin level, and the presence of fibrin split products. Which of the following is the most likely diagnosis?

disseminated intravascular coagulation Disseminated intravascular coagulation is characterized by bleeding from many sites as all coagulation factors are consumed and then broken down, leading to decreased fibrinogen level and platelet count, prolonged PT and PTT, and presence of fibrin split products

A 62-year-old African-American male is seen for his yearly physical exam. He has no complaints. He denies any current medications or medical problems, but the occupational medicine nurse has taken his blood pressure several times in the past year and told him it was high. He denies any tobacco or alcohol use. His blood pressure is 156/92 today. What is the most likely cause of his elevated blood pressure?

essential hypertension The most common cause of hypertension (HTN) is essential, also known as primary hypertension, making up about 95% of patients with hypertension. Secondary causes of hypertension are less common and include sleep apnea, primary aldosteronism, pheochromocytoma, and renal artery stenosis. There is no reason to suspect these other causes in this patient, who is otherwise healthy with stage 1 hypertension based on the stated history.

You are called by one of your patients, a 59-year-old woman with a history of breast cancer, metastatic to the lungs and bones, who has now developed confusion, lethargy, nausea, and vomiting. Further questioning indicates that she also appears to have polyuria and constipation. She completed chemotherapy 6 weeks ago and since then has been on hormonal therapy. She was started on sustained release morphine sulfate and a NSAID one month ago because of worsening bone and back pain. Based on this history, you decide to admit to her to the hospital Your initial workup and treatment are based on a suspected diagnosis of:

hypercalcemia Secretion of PTH-like substance increases the serum calcium

A 53 year-old man with a history of hypertension is being treated with atenolol (Tenormin). He currently presents complaining of chronic fatigue, insomnia, decreased appetite, and difficulty concentrating for the past 3 weeks. His wife also notes that he no longer goes bowling with his friends and has lost interest in any sexual intimacy. Physical examination is unremarkable. Which of the following is the most likely diagnosis?

major depression Diagnostic criteria for a major depressive disorder include a loss of pleasure in usual activities, vegetative or physical changes (poor appetite, loss of energy), and cognitive changes such as difficulty in concentrating.

A 76-year-old man has developed over the past 3 months a large number of occasionally pruritic lesions on his trunk(see above image) These lesions are probably a manifestation of

malignancy These fleshy hyperpigmented papules, seborrheic keratoses, are very common, especially in older adults. They may occasionally be pruritic and tender (but only if secondarily infected). Early "flat" lesions can be confused with solar lentigo, whereas larger pigmented lesions may be mistaken for pigmented basal cell carcinoma or melanoma. Either electrocautery or cryotherapy may be used to remove lesions. Usually they are quite benign and not associated with any systemic condition; however, should seborrheic keratoses appear rapidly and in large numbers, especially if associated with acrochordon (skin tags) and acanthosis nigrilons, then a suspicion for internal malignancy is raised (sign of Leser-Trelat).

A 46 year-old female is being evaluated for a new-onset HTN that was discovered on screening at her workplace. The patient had several readings revealing systolic and diastolic HTN . Patient is currently on no medications. PE is unremarkable. A complete laboratory evaluation revealed hypokalemia as the only abnormality. Which of the following is the most likely diagnosis for this patient?

primary aldosteronism Primary aldosteronism has an increased aldosterone secretion, which causes the retention of sodium and the loss of potassium. This should be the primary consideration for this patient.

A 42 year-old female has a diagnosis of migraine headaches. She had been using ergotamine to abort her headaches, but she is now having one or two headaches per week that are interfering with work. Of the following, the most appropriate preventive therapy is

propranolol (Inderal). Propanolol is useful in preventing migraine headaches and may be maintained indefinitely.

A 30 year-old male has a history of weakness without pain on the left side of his face for the past 4 days. Physical examination of the face reveals unilateral weakness to the left side, but not complete paralysis. The left eye does not close completely. The most appropriate initial treatment is to

reassure and provide patient education Bell's palsy will resolve completely without treatment in approximately 60% of all cases.

A patient with diffuse cutaneous scleroderma (systemic sclerosis) who had been stable for several years is recently noted to have hypertension. This patient is at significant risk of dying from

renal failure Patients with the more malignant variant of systemic sclerosis (scleroderma) have diffuse cutaneous disease characterized by skin thickening in the extremities, face, and trunk. It is this subset of patients, in contrast to those with limited cutaneous disease who often have the CREST syndrome, who are at risk for developing kidney and other visceral disease. Hypertension heralds the onset of a renal crisis manifested by malignant hypertension, encephalopathy, retinopathy, seizures, and left ventricular failure. The renin-angiotensin system is markedly activated; therefore, angiotensin- converting enzyme inhibitors are particularly effective. Even patients who require dialysis may reverse course and have a slow return of renal function after the passage of several months. Patients with systemic sclerosis may also develop esophageal dysfunction, hypomotility of the small intestine (which can produce pain and malabsorption), pulmonary fibrosis sometimes progressing to pulmonary hypertension, and heart failure due to myocardial fibrosis.

A 12 year-old female presents for a routine sports physical. The physical exam reveals asymmetry of the posterior chest wall on forward bending. This is most consistent with which of the following?

scoliosis Asymmetry of the posterior chest wall on forward bending is the most striking and consistent abnormality in patients with idiopathic scoliosis

Which of the following should be the first consideration in the management of acute irritant contact dermatitis?

separation of the patient from the sensitizing agent Unless the source of the irritation is avoided, the dermatitis will not resolve. Topical steroids and compresses can decrease the inflammation, but the patient needs to be separated from the etiologic agent

A 37-year-old woman under your care is diagnosed with bipolar I disorder. As part of her drug regimen, you prescribe lithium carbonate as long-term maintenance therapy. Which of the following would be most appropriate to perform or order prior to the initiation of lithium carbonate?

serum creatinine Prior to beginning lithium, clinicians should obtain a urinalysis, blood urea nitrogen (BUN), creatinine, thyroid function studies, calcium, pregnancy test for women of childbearing potential, and an electrocardiogram (ECG) for patients over age 40. BUN and creatinine should be checked every two to three months during the first six months of therapy, and every 6 to 12 months thereafter. Thyroid function should be checked once or twice during the first six months, and every 6 to 12 months thereafter.

Polysaccharide pneumococcal vaccine should be given to a patient with which of the following diseases?

sickle cell disease Patients with sickle cell disease are functionally asplenic and, therefore, require pneumococcal vaccination to prevent invasive disease

A 14 year-old male who is overweight presents with complaints of left knee and anteromedial thigh pain for the past month. He states the pain gets better with rest and denies any known trauma. On examination of the gait, a slight limp is noted. X-ray films of the left knee are normal. The most likely diagnosis is

slipped capital femoral epiphysis. Slipped capital femoral epiphysis is most common in overweight adolescent males who present with complaints of pain that is referred to the thigh or medial side of the knee associated with a limp. X-ray films of knee are normal since condition involves hip.

In designing a hormone replacement program for patients with coexistent thyroid and adrenal failure,

thyroid replacement must not be initiated until treatment with glucocorticoid has been instituted When coexistent adrenal insufficiency is suspected, it is important that thyroid replacement not be initiated until treatment with a glucocorticoid has begun. Adrenocortical insufficiency can be precipitated by an increase in the clearance rate of glucocorticoids engendered by correction of the hypothyroid state.

43-year-old nurse whose job requires frequent hand washing has noted a small erosive skin lesion between the third interdigital web of the right hand The best therapy for this condition would be

topical clotrimazole Dx of cutaneous cadidiasis can be made clinically or on the basis of demonstration of yeast on KOH preparation. Treatment involves removing predisposing factors such as chronic wetness, antibiotics, or improving glucose control in diabetics and use of effective topical agents such as nystatin or azoles; occasionally, the addition of hydrocortisone cream is required to decrease the associated inflammatory response. Systemic therapy with fluconazole may be required in immunosuppressed patients or individuals whose disease fails to respond to topical therapy.

A 28 year-old female with diabetes mellitus type 2 sustains a partial thickness burn to her left upper arm and her chest when hot grease spilled on her at home. The burn to her arm is circumferential and the estimated total body surface burned is 18%. She has no allergies. The most appropriate treatment of this patient would include

transfer to a burn center Reasons for transfer to a burn center include a partial thickness burn covering greater than 10% of total body surface area. In addition, burns in patients with pre-existing medical conditions, such as diabetes, that could complicate their management, prolong recovery, or affect their outcome, is also a reason for transfer to a burn center.

A 32 year-old medical transcriptionist presents with burning and tingling in her right wrist and hand for the past month. On physical exam, Phalen's test is positive; however, there is no atrophy of the thenar eminence. Which of the following is the initial step in management of this patient?

wrist splints The treatment of carpal tunnel syndrome is aimed at relieving the pressure on the median nerve. This is best accomplished by having the patient wear a wrist splint during the activities that increase the pressure on the median nerve


संबंधित स्टडी सेट्स

Nursing Care of Clients with Cognitive Disorders

View Set

Aleks Chemistry - Setting up a unit conversion

View Set

Sectional Anatomy Head and Neck Test 1

View Set

lofi playlist to study/chill/summon satan to

View Set

Sales and Marketing Chapters 4-7

View Set

Mastering Astronomy 101- Chapter 3

View Set

Chapter 8-Recombinant DNA Technology

View Set

SHS 431 Final: Secondary Developmental Language disorders

View Set